11.01.2015 Views

mat_olm_book

mat_olm_book

mat_olm_book

SHOW MORE
SHOW LESS

You also want an ePaper? Increase the reach of your titles

YUMPU automatically turns print PDFs into web optimized ePapers that Google loves.

MATEMATİK<br />

OLİMPİYATI ÇALIŞMA<br />

KİTAPÇIĞI<br />

www.sbelian.wordpress.com


Mate<strong>mat</strong>˙ık Ol˙ımp˙ıyatları<br />

Çalışma K˙ıtapçığı<br />

www.sbelian.wordpress.com<br />

6 Temmuz 2010


İçindekiler<br />

1 Giriş 5<br />

1.1 İlksöz . . . . . . . . . . . . . . . . . . . . . . . . . . . . . . . . 5<br />

2 KONULAR 6<br />

2.1 Denklem S˙ıstemler˙ı . . . . . . . . . . . . . . . . . . . . . . . . . 6<br />

2.1.1 Çalışma Soruları . . . . . . . . . . . . . . . . . . . . . . 9<br />

2.1.2 Çözümler . . . . . . . . . . . . . . . . . . . . . . . . . . 11<br />

2.2 Repun˙ıtler . . . . . . . . . . . . . . . . . . . . . . . . . . . . . . 16<br />

2.2.1 Çalışma Soruları . . . . . . . . . . . . . . . . . . . . . . 19<br />

2.2.2 Çözümler . . . . . . . . . . . . . . . . . . . . . . . . . . 20<br />

2.3 Soph˙ıe Germa˙ın Özdeşl˙ığ˙ı . . . . . . . . . . . . . . . . . . . . . 23<br />

2.3.1 Çalışma Soruları . . . . . . . . . . . . . . . . . . . . . . 24<br />

2.3.2 Çözümler . . . . . . . . . . . . . . . . . . . . . . . . . . 26<br />

2.4 Tamkareler Poz˙ıt˙ıft˙ır . . . . . . . . . . . . . . . . . . . . . . . . 30<br />

2.4.1 Çalışma Soruları . . . . . . . . . . . . . . . . . . . . . . 31<br />

2.4.2 Çözümler . . . . . . . . . . . . . . . . . . . . . . . . . . 34<br />

2.5 Eş˙ıts˙ızl˙ıkler I . . . . . . . . . . . . . . . . . . . . . . . . . . . . 40<br />

2.5.1 Çalışma Soruları . . . . . . . . . . . . . . . . . . . . . . 49<br />

2.6 Eş˙ıts˙ızl˙ıkler II . . . . . . . . . . . . . . . . . . . . . . . . . . . . 50<br />

2.6.1 Çalışma Soruları . . . . . . . . . . . . . . . . . . . . . . 58<br />

2.7 İnd˙ırgemel˙ı D˙ız˙ıler . . . . . . . . . . . . . . . . . . . . . . . . . 59<br />

2.7.1 Birinci Dereceden İndirgemeler . . . . . . . . . . . . . . 59<br />

2


2.7.2 İkinci Dereceden İndirgemeler . . . . . . . . . . . . . . . 63<br />

2.7.3 Alıştırmalar . . . . . . . . . . . . . . . . . . . . . . . . . 65<br />

2.8 Yen˙ıden Düzenleme [Rearrangement] Eş˙ıts˙ızl˙ığ˙ı . . . . . . . . . 66<br />

2.9 Tr˙ıgonometr˙ık Değ˙ışken Değ˙ışt˙ırme . . . . . . . . . . . . . . . . 75<br />

2.9.1 Çalışma Soruları . . . . . . . . . . . . . . . . . . . . . . 78<br />

2.9.2 Çözümler . . . . . . . . . . . . . . . . . . . . . . . . . . 80<br />

2.10 Ceb˙ırde Teleskop˙ık Toplamlar ve Çarpımlar . . . . . . . . . . . 90<br />

2.10.1 Çalışma Soruları . . . . . . . . . . . . . . . . . . . . . . 93<br />

2.10.2 Çözümler . . . . . . . . . . . . . . . . . . . . . . . . . . 96<br />

2.11 Tamdeğer Fonks˙ıyon Problemler˙ı . . . . . . . . . . . . . . . . . 102<br />

2.11.1 Çalışma Soruları . . . . . . . . . . . . . . . . . . . . . . 107<br />

2.11.2 Çözümler . . . . . . . . . . . . . . . . . . . . . . . . . . 108<br />

2.12 Bölüneb˙ılme ve Asal Sayılar . . . . . . . . . . . . . . . . . . . . 110<br />

2.12.1 Çalışma Soruları . . . . . . . . . . . . . . . . . . . . . . 114<br />

2.12.2 Çözümler . . . . . . . . . . . . . . . . . . . . . . . . . . 116<br />

2.13 Tr˙ıgonometr˙ıde Sonsuz Toplam ve Farklar . . . . . . . . . . . . 122<br />

2.13.1 Çalışma Soruları . . . . . . . . . . . . . . . . . . . . . . 124<br />

2.13.2 Çözümler . . . . . . . . . . . . . . . . . . . . . . . . . . 126<br />

2.14 Kuş, Güverc˙ın, Yuva... . . . . . . . . . . . . . . . . . . . . . . . 133<br />

2.14.1 Çalışma Soruları . . . . . . . . . . . . . . . . . . . . . . 135<br />

2.14.2 Çözümler . . . . . . . . . . . . . . . . . . . . . . . . . . 136<br />

2.15 Üstel D˙ıyofant Denklemler˙ı . . . . . . . . . . . . . . . . . . . . 137<br />

2.15.1 Çalışma Soruları . . . . . . . . . . . . . . . . . . . . . . 139<br />

2.15.2 Çözümler . . . . . . . . . . . . . . . . . . . . . . . . . . 140<br />

2.16 Kalan Sınıfları [Res˙ıdues] . . . . . . . . . . . . . . . . . . . . . 143<br />

2.16.1 Çalışma Soruları . . . . . . . . . . . . . . . . . . . . . . 146<br />

2.16.2 Çözümler . . . . . . . . . . . . . . . . . . . . . . . . . . 147<br />

2.17 V˙ıete Teoremi . . . . . . . . . . . . . . . . . . . . . . . . . . . . 151<br />

2.17.1 Newton-Girard Formulæ . . . . . . . . . . . . . . . . . . 152<br />

2.17.2 Lagrange İnterpolasyon Tekn˙ığ˙ı . . . . . . . . . . . . . . 155<br />

2.17.3 Çalışma Soruları . . . . . . . . . . . . . . . . . . . . . . 157<br />

3


2.17.4 Çözümler . . . . . . . . . . . . . . . . . . . . . . . . . . 159<br />

2.18 Bağıntı Sayıları . . . . . . . . . . . . . . . . . . . . . . . . . . . 163<br />

2.18.1 Çalışma Soruları . . . . . . . . . . . . . . . . . . . . . . 167<br />

2.18.2 Çözümler . . . . . . . . . . . . . . . . . . . . . . . . . . 167<br />

2.19 L˙ıneer Denklemler˙ın Tamsayı Çözümler˙ı . . . . . . . . . . . . . 168<br />

2.19.1 Çalışma Soruları . . . . . . . . . . . . . . . . . . . . . . 172<br />

2.19.2 Çözümler . . . . . . . . . . . . . . . . . . . . . . . . . . 173<br />

2.20 Fonks˙ıyonel Denklemler . . . . . . . . . . . . . . . . . . . . . . 173<br />

2.20.1 Tek Değişkenliler - Temel Teknikler . . . . . . . . . . . 173<br />

2.20.2 Çok Değişkenliler . . . . . . . . . . . . . . . . . . . . . . 176<br />

2.20.3 Çalışma Soruları . . . . . . . . . . . . . . . . . . . . . . 180<br />

2.20.4 Çözümler . . . . . . . . . . . . . . . . . . . . . . . . . . 180<br />

4


Bölüm 1<br />

Giriş<br />

1.1 İlksöz<br />

Belkide internetin hayatımıza kattığı en büyük artı değer legal paylaşımlar<br />

yapmak ve bilgiyi birbirimizle paylaşmaya ortam hazırlamak <strong>olm</strong>uştur. Bizde<br />

bu paylaşımın bir parçası olarak bu zamana kadar bir çok ders notunu ve<br />

çözüm paketini sizlerle paylaşmıştık. Şimdi de, tüm bu ders notlarının birleşimi<br />

olan bu mini kitabı sizlerin paylaşımına açıyoruz. Sizlerde çekinmeden<br />

bu kitapçığı gerek fotokopi ile gerekse elektronik yollarla birbirinizle paylaşın.<br />

Yararlanabildiğiniz kadar yararlanın.<br />

Bu çalışmanın hazırlanmasında öncelikle www.sbelian.wordpress. com sayfamıza<br />

teveccüh gösteren tüm dostlarımıza ve faydalı olabilmek için çalışmalarımıza<br />

yardım eden LATEX 2ε dostlarına teşekkür ederiz.<br />

Sbelian<br />

Σ<br />

Haziran 2010<br />

5


Bölüm 2<br />

KONULAR<br />

2.1 Denklem S˙ıstemler˙ı<br />

Bu bölümde bazı standart <strong>olm</strong>ayan denklem sistemlerinin çözümlerini yapacağız.<br />

Ancak ilerleyen örneklerde sizinde farkedeceğiniz üzere, kullanacağımız<br />

yöntemler genelde bazı cebirsel manipülasyonlardan oluşuyor. Bu yöntemleri<br />

kullanarak hem çözümlere daha kolay ulaşacağız hemde her bir soruda<br />

farklı bir tekniği öğrenmiş olacağız.<br />

Örnek.<br />

x + y 2 + z 2 = 3<br />

y + z 2 + x 3 = 3<br />

z + x 2 + y 3 = 3<br />

denklem sisteminin pozitif çözüm üçlüsünün yanlızca (1, 1, 1) olduğunu kanıtlayınız.<br />

Çözüm.<br />

Eğer ilk iki denklemin farkını alırsak<br />

x(1 − x 2 ) + y(y − 1) + z 2 (z − 1) = 0<br />

denklemini elde ederiz. Benzer şekilde ikinci ve üçüncü denklemlerin farkını<br />

alırsak<br />

y(1 − y 2 ) + z(z − 1) + x 2 (x − 1) = 0<br />

6


2.1. DENKLEM SİSTEMLERİ 7<br />

denklemini elde ederiz. Eğer bu denklemi z ile çarpıp bir önceki denklemden<br />

çıkarırsak<br />

x(x − 1)(1 + x + xz) = y(y − 1)(1 + z + yz)<br />

eşitliğini elde ederiz. Benzer işlemleri yaparak<br />

y(y − 1)(1 + y + yx) = z(z − 1)(1 + x + xz)<br />

eşitliğinede ulaşırız. Son yazdığımız iki eşitlikte eğer x, y, z pozitifse x = y =<br />

z = 1, x, y, z < 1 veya x, y, z > 1 olacaktır. Son iki yazdığımız olasılığın<br />

x + y 2 + z 3 = 3<br />

eşitliğini sağlayamayacağı açıktır. Demek ki tek çözüm<br />

olacaktır.<br />

Örnek.<br />

a + b + c + d = 12<br />

(x, y, z) = (1, 1, 1)<br />

abcd = 27 + ab + ac + ad + bc + bd + cd<br />

denklem sistemini sağlayan tüm (a, b, c, d) pozitif sayı dörtlülerini bulunuz.<br />

Çözüm. Eğer soruda verilen sistemdeki ikinci eşitliğe Aritmetik Orta -<br />

Geometrik Orta eşitsizliğini uygularsak<br />

abcd ≥ 27 + 6 √ abcd<br />

olacaktır. Eğer bu eşitsizliği düzenlersek yeni denklemimiz √ abcd değişkenine<br />

bağlı<br />

( √ abcd + 3)( √ abcd − 9) ≥ 0<br />

eşitsizliğini elde ederiz. Burada<br />

√<br />

abcd ≥ 9


8 BÖLÜM 2. KONULAR<br />

olduğu açıktır. Eğer bu durumu sistemdeki ilk denklemle beraber kullanırsak<br />

a + b + c + d<br />

4<br />

≤ 4√ abcd ⇒ 3 ≤ 3<br />

olduğundan eşitlik durumu söz konusudur. Buna göre tek çözüm<br />

olacaktır.<br />

a = b = c = d = 3<br />

Örnek.<br />

√<br />

3x<br />

(1 + 1 )<br />

x + y<br />

√<br />

7y<br />

(1 − 1 )<br />

x + y<br />

= 2<br />

= 4 √ 2<br />

denklem sistemini pozitif reel sayılarda çözünüz.<br />

Çözüm. Bu soruda çözüme daha kolay işlemlerle ulaşmak için bazı değişken<br />

eğiştirmeler yapmak yerinde olacaktır. Buna göre eğer √ x = u ve √ y = v<br />

alırsak sistemimiz<br />

(<br />

)<br />

1<br />

u 1 +<br />

u 2 + v 2 = √ 2 3<br />

(<br />

)<br />

1<br />

v 1 −<br />

u 2 + v 2 = 4√ 2<br />

√<br />

7<br />

şeklini alır. Burada u 2 +v 2 aslında z = u+iv karmaşık sayısının normunun karesidir.<br />

Buna göre ikinci denklemi i karmaşık sayısı ile çarpıp birinci denkleme<br />

eklersek<br />

⎛<br />

u + iv + u − iv<br />

u 2 + v 2 = ⎝<br />

2<br />

√<br />

3 + i<br />

4 √ ⎠<br />

2<br />

√<br />

7<br />

⎞<br />

eşitliğini elde ederiz. Burada<br />

u − iv<br />

u 2 + v 2


2.1. DENKLEM SİSTEMLERİ 9<br />

ifadesi<br />

olacaktır. Yani<br />

z<br />

|z| 2 =<br />

z<br />

(zz) = 1 z<br />

(<br />

)<br />

z + 1 z = 2<br />

√ + i 4√ 2<br />

√<br />

3 7<br />

eşitliği elde edilir. Eğer bu denklemi düzenlersek<br />

(<br />

)<br />

z 2 2<br />

− √ + i 4√ 2<br />

√ z + 1 = 0<br />

3 7<br />

denklemini ve bu denklemin çözümü olan<br />

( 1√3<br />

± √ 2 ) (<br />

2 √ 2<br />

+ i √ ± √ )<br />

2<br />

21 7<br />

sayısını elde ederiz. Buna göre soruda verilen sistemin çözümü<br />

x =<br />

( 1√3<br />

± √ 2 ) (<br />

2<br />

2 √ 2<br />

ve y = √ ± √ ) 2<br />

2<br />

21 7<br />

olacaktır.<br />

2.1.1 Çalışma Soruları<br />

1.<br />

x + 2 x = 2y<br />

y + 2 y<br />

z + 2 z<br />

= 2z<br />

= 2x<br />

denklem sistemini çözünüz.


10 BÖLÜM 2. KONULAR<br />

2. [x], x sayısının tam kısmını, {x} ise ondalık kısmını temsil etmek üzere<br />

x = [x] + {x} olarak veriliyor. Buna göre<br />

x + [y] + {z} = 1, 1<br />

z + [x] + {y} = 2, 2<br />

y + {x} + [z] = 3, 3<br />

denklem sistemini sağlayan x, y, z değişkenlerini bulunuz.<br />

3. Aşağıdaki denklem sistemini pozitif reel sayılarda çözünüz.<br />

xy + yz + zx = 12<br />

xyz = 2 + x + y + z<br />

4.<br />

4x 2<br />

4x 2 + 1<br />

4y 2<br />

4y 2 + 1<br />

4z 2<br />

4z 2 + 1<br />

= y<br />

= z<br />

= x<br />

Denklem sisteminin tüm reel çözümlerini bulunuz.<br />

5.<br />

3 = x + y + z = x 3 + y 3 + z 3<br />

eşitliğini sağlayan tüm tamsayı üçlülerini bulunuz.<br />

6.<br />

6(x − y −1 ) = 3(y − z −1 ) = 2(z − x −1 ) = xyz − (xyz) −1<br />

eşitliğini sağlayan tüm sıfıran farklı x, y, z reel sayılarını bulunuz.


2.1. DENKLEM SİSTEMLERİ 11<br />

7. a, b, c birbirinden ve sıfırdan farklı reel sayılar <strong>olm</strong>ak üzere verilen<br />

8.<br />

9.<br />

x 2 − yz = a<br />

y 2 − zx = b<br />

z 2 − xy = c<br />

denklem sisteminin tüm (x, y, z) reel üçlülerini bulunuz.<br />

(x + y) 3 = z<br />

(y + z) 3 = x<br />

(z + x) 3 = y<br />

denklem sistemini sağlayan tüm (x, y, z) reel sayı üçlüleini bulunuz.<br />

x 3 − 9(y 2 − 3y + 3) = 0<br />

y 3 − 9(z 2 − 3z + 3) = 0<br />

z 3 − 9(x 2 − 3x + 3) = 0<br />

denklem sisteminin tüm çözümlerini bulunuz.<br />

10. a, b, x, y reel sayılar <strong>olm</strong>ak üzere<br />

ax + by = 3<br />

ax 2 + by 2 = 7<br />

ax 3 + by 3 = 16<br />

ax 4 + by 4 = 42<br />

olarak veriliyor. Buna göre, ax 5 + by 5 toplamının eşitini bulunuz.<br />

2.1.2 Çözümler<br />

1. Eğer çözümde aritmetik orta - geometrik orta eşitsizliğini kullanırsak<br />

x + 1 √<br />

x ≥ 2 x · 2<br />

x = 2√ 2


12 BÖLÜM 2. KONULAR<br />

ise 2y ≥ 2 √ 2 ve y ≥ √ 2 olacaktır. Benzer şekilde y ≥ √ 2 ve z ≥ √ 2<br />

olacaktır. Eğer sistemdeki 3 denklemi −1 ile çarpıp toplarsak<br />

( 1<br />

x + y + z = 2<br />

x + 1 y + 1 )<br />

z<br />

eşitliği elde edilir. Bu denklemin sol kısmı<br />

x + y + x ≥ 3 √ 2<br />

iken sağ kısım<br />

( 1<br />

2<br />

x + 1 y + 1 )<br />

≤ 3 √ 2<br />

z<br />

olacaktır. Demek ki<br />

( 1<br />

x + y + z = 2<br />

x + 1 y + 1 )<br />

= 3 √ 2<br />

z<br />

<strong>olm</strong>alıdır. Buradan istenen çözümler<br />

x = y = z = ± √ 2<br />

olarak bulunur.<br />

2. Eğer sistemdeki 3 eşitsizliği taraf tarafa toplarsak<br />

2x + 2y + 2z = 6.6<br />

olduğundan<br />

x + y + z = 3.3<br />

denklemini elde ederiz. Sistemdeki her bir denklemi son bulduğumuz<br />

eşitlikten çıkarırsak yeni sistemimiz<br />

{y} + [z] = 2.2<br />

{x} + [y] = 1.1<br />

{z} + [x] = 0


2.1. DENKLEM SİSTEMLERİ 13<br />

olacaktır. Bu sistemde ilk denklemden {y} = 0.2 ve [z] = 2 bulunur.<br />

İkinci denklemden [y] = 1, {0.1} ve üçüncü denklemden {z} = 0, [x] = 0<br />

bulunur. Buna göre, denklem sisteminin çözümü<br />

olacaktır.<br />

x = 0.1, y = 1.2, z = 2<br />

3. Varsayalım 3√ xyz = a olsun. Aritmetik orta - geometrik orta eşitsizliğinden<br />

12 = xy + yz + zx ≥ 3a 2<br />

ve<br />

a 3 = 2 + x + y + z ≥ 2 + 3a<br />

olacaktır. ilk eşitsizlikten 12 ≥ 3a 2 ise 4 ≥ a 2 bulunur. İkinci eşitsizliktense<br />

a ≥ 2 bulunur. Dolayısıyla a = 2 ve x = y = z olacaktır. Demek<br />

ki tek çözüm<br />

(x, y, z) = (2, 2, 2)<br />

olur.<br />

4. Çözüme, denklem sistemimizdeki her bir denklemi karşılayan bir fonksiyon<br />

bularak başlayalım. Varsayalım f fonksiyonu f : [0, ∞) → [0, ∞)<br />

<strong>olm</strong>ak üzere<br />

f(t) =<br />

4t2<br />

4t 2 + 1<br />

olarak verilsin. f fonksiyonun artan olduğu açıktır. Dolayısıyla eğer x < y<br />

ise f(x) < f(y) dolayısıyla y < z olacaktır. Eğer bu argümanı tekrar<br />

edersek z < x elde ederiz. Öyleyse x < y < z < x olacaktır ki bu durum<br />

imkansızdır. Benzer biçimde x > y ile başlarsak yine çelişki elde ederiz.<br />

Demek ki x = y = z <strong>olm</strong>alıdır. Buna göre,<br />

4t 2<br />

4t 2 + 1 = t<br />

denklemini çözersek t = 0 veya t = 1/2 olacaktır. Öyleyse sistemin<br />

çözümleri sadece<br />

olarak bulunur.<br />

(0, 0, 0) ve ( 1 2 , 1 2 , 1 2 )


14 BÖLÜM 2. KONULAR<br />

5. Eğer soruda verilen eşitlik üzerinde biraz oynarsak<br />

24 = (x + y + z) 3 − (x 3 + y 3 + z 3 ) = 3 ∑ x 2 y + 6xyz<br />

eşitliğine ulaşırız. Eğer sadeleştirme yapıp çarpanlarına ayırırsak<br />

8 = (x + y)(x + z)(y + z)<br />

olacağından<br />

8 = (3 − x)(3 − y)(x + y)<br />

eşitliği elde edilir. 8 sayısının çarpanlarını kontrol edersek çözümleri<br />

olarak bulabiliriz.<br />

6. Soruda verilen eşitliği kullanarak<br />

(1, 1, 1), (4, 4, −5), (4, −5, 4), (−5, 4, 4)<br />

(x − y −1 ) + (y − z −1 ) + (z − x −1 ) = xyz − (xyz) −1<br />

yazabiliriz. Eğer bu eşitliği yeniden düzenlersek de<br />

(x − y −1 )(y − z −1 )(z − x −1 ) = 0<br />

denklemini elde ederiz. Demek ki çarpanlardan biri sıfır <strong>olm</strong>alıdır. Ancak<br />

sorudaki eşitlikten hepsinin sıfır olduğu ortaya çıkar. Dolayısıyla<br />

bulunur.<br />

xy = yz = zx = 1 ve x = y = z = ±1<br />

7. Soruda verilen sistemdeki herbir eşitliğin karesini alıp diğer iki eşitliğin<br />

çarpımından çıkarırsak yeni sistemimimiz<br />

a 2 − bc = x(x 3 + y 3 + z 3 − 3xyz)<br />

b 2 − ac = y(x 3 + y 3 + z 3 − 3xyz)<br />

c 2 − ab = z(x 3 + y 3 + z 3 − 3xyz)


2.1. DENKLEM SİSTEMLERİ 15<br />

olacaktır. Burada<br />

olarak alırsak<br />

x 3 + y 3 + z 3 − 3xyz = k<br />

(a 2 − bc) 2 − (b 2 − ac) 2 (c 2 − ab) = k 2 (x 2 − yz) = k 2 a<br />

olur. Burada eşitliğin sol tarafını açıp düzenlersek<br />

k = ± √ a 3 + b 3 + c 3 − 3abc<br />

olacaktır. Buna göre sistemin çözümleri<br />

olacaktır.<br />

x = a2 − bc<br />

k<br />

, y = b2 − ac<br />

,<br />

k<br />

c 2 − ab<br />

k<br />

8. Eğer ikinci denklemi birinciden çıkarırsak<br />

(x − z) ( (x + y) 2 + (x + y)(y + z) + (y + z) 2) = z − x<br />

eşitliğini elde ederiz. Burada<br />

(x + y) 2 + (x + y)(y + z) + (y + z) 2 > 0<br />

olduğuna göre, x = z olacaktır. Simetriden dolayı y = z ve elimizde<br />

8x 3 = x denklemi oluşacaktır. Bu denklemin kökleri x = 0 ve x = ± 2<br />

2 √ 2<br />

olacaktır. Buna göre sistemin çözümleri<br />

olur.<br />

x = y = z = 0, x = y = z = ± 1<br />

2 √ 2<br />

9. Soruda verilen sistemi düzenlersek, yeni sistemimiz<br />

(y − 3) 3 = y 3 − x 3<br />

(z − 3) 3 = z 3 − x 3<br />

(x − 3) 3 = x 3 − z 3


16 BÖLÜM 2. KONULAR<br />

olacaktır. Bu eşitliği toplarsak<br />

(x − 3) 3 + (y − 3) 3 + (z − 3) 3 = 0<br />

eşitliği elde edilir. Genelliği bozmadan varsayalım x ≥ 3 olsun. Sistemdeki<br />

üçüncü denklemden<br />

z 3 − 27 = 9x(x − 3)<br />

olacaktır, dolayısıyla z ≥ 3 olur. Benzer biçimde y ≥ 3 olacaktır. Ancak<br />

üçü birden 3’ten büyük olamaz. Buna göre tek çözüm x = y = z = 3<br />

olacaktır.<br />

10. n = 2 ve n = 3 için<br />

(ax n + by n )(x + y) − (ax n−1 + by n−1 )xy = ax n+1 + by n+1<br />

eşitliğinin sağlandığı görülebilir. Buna göre,<br />

7(x + y) − 3xy = 16<br />

ve<br />

16(x + y) − 7xy = 42<br />

olacaktır. Bu iki denklemi çözersek x+y = −14 ve xy = −38 olur. n = 4<br />

için başta belirlediğimiz reküransı yeniden uygularsak<br />

olacaktır.<br />

ax 5 + by 5 = (42)(−14) − (16)(−38) = −588 + 608 = 20<br />

2.2 Repun˙ıtler<br />

Basamaklarını oluşturan rakamların hepsi 1 olan doğal sayılara repunit<br />

diyeceğiz. Öyleki<br />

111 · · · 111 } {{ }<br />

n basamaklı


2.2. REPUNİTLER 17<br />

sayısı bir repunit sayı olarak sorularımızda yerini alacaktır. Şimdi ilk örneğimizle<br />

başlayalım. Sorumuz 2005 Bulgaristan Mate<strong>mat</strong>ik Olimpiyatları’nda<br />

sorulmuştur.<br />

Örnek.<br />

x, y, z ∈ Z <strong>olm</strong>ak üzere verilen<br />

x 2 + 2y 2 + 98z 2 = 111 · · · 111 } {{ }<br />

666 tane 1<br />

eşitliğini sağlayan tamsayı üçlülerinin bulunmadığını kanıtlayınız.<br />

Çözüm I. Çözüme çelişki ile ulaşmaya çalışalım. varsayalım soruda verilen<br />

denklemi sağlayan (x, y, z) tamsayı üçlüsü varolsun. Buna göre eşitliğin<br />

sol tarafındaki repuniti çarpanlarına ayırırsak<br />

111111 · (1 + 10 6 + · · · + 10 6·110 ) = 106 − 1<br />

(1 + 10 6 + · · · + 10 6·110 )<br />

9<br />

olacaktır. Küçük Fer<strong>mat</strong> teoremine göre, 10 6 − 1 sayısı 7 ile bölünebilir. Buna<br />

göre eşitliğimize (mod7) altında bakabiliriz. Tamkare bir sayının (mod7) altındaki<br />

kalan sınıfı {0, 1, 2, 3, 4} ve 98 sayısı 7 ile kalansız bölünebildiğine göre,<br />

eşitliğin sağ tarafındaki ifademizde x ve y sayıları 7 ile kalansız bölünmelidir.<br />

Bu durumda eşitliğin sol tarafı 7 2 ile tam bölünecektir. Ancak eşitliğin sağ<br />

kısmındaki ifademiz<br />

7 · 15873 · (1 + 10 6 + · · · + 10 6·110 )<br />

olduğundan bu ifadenin (mod7) altındaki ikinci çarpanı 4 olacaktır. Üçüncü<br />

çarpanı (mod7) altında 0 <strong>olm</strong>adığına göre, eşitliğin sağ tarafı 49 ile bölünemez.<br />

Çelişki vardır. Demekki (x, y, z) tamsayı üçlüleri yoktur.<br />

Eğer soruda verilen eşitliğin sağ tarafına (mod) altında ba-<br />

Çözüm II.<br />

karsak<br />

veya<br />

x 2 + 2y 2 + 2x 2 ≡ 111 (mod 8)<br />

x 2 + 2y 2 + 2z 2 ≡ 7 (mod 8)


18 BÖLÜM 2. KONULAR<br />

olduğunu görebiliriz. Bir tamsayının karesinin ( mod 8) altındaki kalanları {0, 1, 4}<br />

olabilir. Eğer iki katlarını alırsak kalan sınıfı {0, 2} olacaktır. Ancak elde edilen<br />

{0, 1, 4} kümesinin elemanlarını {0, 2} ve {0, 2} kümesinin elemanlarına ekleyerek<br />

(mod8) altında 7 sayısına ulaşamayız. Demek ki, soruda verilen denklemi<br />

sağlayan (x, y, z) tamsayıları yoktur.<br />

Sıradaki sorumuzda Rusyada yayınlanan Potansia Magazine isimli dergiden<br />

alıntıdır.<br />

Örnek. Repunitleri yine repunitlere götüren tüm ikinci dereceden tamsayı<br />

katsayılı polinomları bulunuz.<br />

Çözüm. varsayalım ikinci dereceden polinomumuz f(x) = ax 2 + bx + c<br />

olsun. Soruda verilen şartlara göre,<br />

yani<br />

olacaktır. Eğer<br />

f(111 } {{ · · · 1}<br />

) = } 111 {{ · · · 1}<br />

m tane n tane<br />

( 10 m )<br />

− 1<br />

f<br />

9<br />

olarak alırsak,<br />

( ( ) x − 1 2 ( ) )<br />

x − 1<br />

9 a + b + 1<br />

9<br />

9<br />

olacaktır. Demek ki<br />

g(x) = a 9 x2 +<br />

= 10n − 1<br />

9<br />

( ) x − 1<br />

g(x) = 9f + 1<br />

9<br />

( a<br />

= 9<br />

9 2 (x2 − 2x + 1) + b )<br />

9 (x − 1) + c + 1<br />

= a 9 (x2 − 2x + 1) + b(x − 1) + c + 1<br />

= a (<br />

9 x2 + b − 2a )<br />

x + (9c + 1 − b + a 9<br />

9 )<br />

(<br />

b − 2a )<br />

x + (9c + 1 − b + a 9<br />

9 )


2.2. REPUNİTLER 19<br />

olacaktır. Buna göre, aslında g(10 m ) = 10 n olacaktır. yani g polinomu 10’un<br />

kuvvetlerini yine 10’un kuvvetlerine götürecektir. Buna göre,<br />

10 −2m · g(10 m ) = 10 n−2m = a (<br />

9 + b − 2a ) (<br />

10 −m + 9c + 1 − b + a )<br />

10 −2m<br />

9<br />

9<br />

olacaktır. İlk eşitlikten görüldüğü üzere 10−2m ·g(10 m ) sayısı 10’un kuvvetidir.<br />

Eğer m değerini yeterince büyük alırsak 10 −2m g(10 m ) ifadesi a/9 değerine<br />

yakınsak. Buradan da, a/9 = 10 k ve 10 −2m g(10 m ) = 10 k olacaktır. Dolayısıyla<br />

(<br />

b − 2a 9<br />

)<br />

10 −m +<br />

(<br />

9c + 1 − b + a 9<br />

)<br />

10 −2m = 0<br />

olacağından<br />

b − 2a 9 = 9c + 1 − b + a 9 = 0<br />

olur. Buradan b = 2 · 10 k ve c = 10k −1<br />

9<br />

olacağından istenen polinomlar<br />

olur.<br />

2.2.1 Çalışma Soruları<br />

f(x) = 9 · 10 k · x 2 + 2 · 10 k · x + 10k − 1<br />

9<br />

1. (a.) Çift sayıda basamağı olan, beş tabanındaki repunitlerin ardışık pozitif<br />

iki tamsayının çarpımı olduğunu gösteriniz.<br />

(b.) Dokuz tabanındaki tüm repunitlerin Üçgensel Sayı 1 olduğunu kanıtlayınız.<br />

2.<br />

111 } {{ · · · 1}<br />

= } 222 {{ · · · 2}<br />

+(333 } {{ · · · 3}<br />

2n tane n tane n tane<br />

eşitliğini kanıtlayınız.<br />

) 2<br />

1 Bir a k üçgensel sayısı, 1’den k’ya kadar olan tamsayıların toplamıdır. Üçgensel sayı<br />

denilmesinin temel sebebi, eşkenar bir üçgenin üzerine eşit aralıklarla yerleştirilibilecek eşit<br />

yarıçaplı kürelerin sayısını vermesidir. Öyleki, ilk bir kaç üçgensel sayı 1, 3, 6, 10, 15, 21, 28<br />

olarak kolaylıkla bulunabilir.<br />

a n = 1 + 2 + 3 + · · · + (n − 1) + n = n(n+1) = n2 +n<br />

= ( )<br />

n+1<br />

2 2 2 genel formuda n. üçgensel sayıyı<br />

vermektedir.


20 BÖLÜM 2. KONULAR<br />

3. 19 ile bölünebilen ve tüm basamakları 1 olan en küçük pozitif tamsayıyı<br />

bulunuz.<br />

4. Asal bir repunit sayının basamak sayısınında asal olduğunu kanıtlayınız.<br />

Ayrıca, bu önermenin tersi doğrumudur<br />

5. Tüm basamakları 1 olan, 81 basamaklı bir sayı 81 ile kalansız bölünebilir<br />

mi<br />

6. (a.) 111 · · · 1 } {{ }<br />

n basamaklı<br />

sayının 41 ile ancak ve ancak 5|n olduğunda, bölünebileceğini<br />

kanıtlayınız.<br />

(b.) 111 · · · 1 } {{ }<br />

n basamaklı<br />

sayının 91 ile ancak ve ancak 6|n olduğunda, bölünebileceğini<br />

kanıtlayınız.<br />

7. 1’den büyük hiç bir repunitin tam kare olamayacağını kanıtlayınız.<br />

8. Son basamağı 1, 3, 7 veya 9 olan her n tamsayısı için, n ile bölünebilen<br />

bir repunit bulunduğunu kanıtlayınız.<br />

9. Herhangi iki elemanı aralarında asal olan sonsuz büyüklükte bir repunitler<br />

dizisi olduğunu kanıtlayınız.<br />

10. Sonsuz çoklukta n değeri için, basamaklarında sıfır bulunmayan n basamaklı<br />

bir sayının basamakları toplamına bölünebileceğini kanıtlayınız.<br />

2.2.2 Çözümler<br />

1. (a.)<br />

(111 } {{ · · · 1}<br />

) 5 = 1 + 5 + · · · + 5 2n−1<br />

2n tane<br />

= 52n − 1<br />

= 5n − 1<br />

· 5n + 1<br />

4 2 2<br />

eşitliğine göre, 5n −1<br />

2<br />

ve 5n +1<br />

2<br />

ardışık tamsayılardır.


2.2. REPUNİTLER 21<br />

(b.)<br />

(111 } {{ · · · 1}<br />

) 9 = 1 + 9 + · · · + 9 n−1<br />

n tane<br />

= 9n − 1<br />

= 1 8 2 · 3n − 1<br />

2<br />

· 3n + 1<br />

2<br />

olduğuna göre bu sayı bir üçgensel sayıdır. Çünkü, 3n −1<br />

ardışık tamsayılardır.<br />

2. Soruda verilen eşitliği kullanalım. Buna göre,<br />

2<br />

ve 3n +1<br />

2<br />

} 111 {{ · · · 1}<br />

− 222 } {{ · · · 2}<br />

2n tane n tane<br />

= } 111 {{ · · · 1}<br />

000 } {{ · · · 0}<br />

− 111 } {{ · · · 1}<br />

n tane n tane n tane<br />

= 111 · · · 1 ·(10 n − 1) = 111 · · · 1<br />

} {{ }<br />

n tane<br />

} {{ }<br />

n tane<br />

= } 333 {{ · · · 3}<br />

· 333 } {{ · · · 3}<br />

= (333 } {{· 3}<br />

n tane n tane n tane<br />

) 2<br />

· 999 · 9 } {{ }<br />

n tane<br />

olur.<br />

3. Varsayalım A sorudaki şartları sağlayan sayımız olsun. Eğer basamak<br />

sayısını n olarak alırsak A = 10n −1<br />

9<br />

olacaktır. 9 ve 19 sayıları aralarında<br />

asal olduklarına göre, en küçük n değeri için, 10 n − 1 sayısının 19 ile<br />

bölünmesi gerekmektedir. Küçük Fer<strong>mat</strong> teoremine göre, 10 18 − 1 sayısı<br />

19 ile bölünebilir. Eğer daha küçk bir n değeri varsa, 10 n ≡ 1 (mod19) ve<br />

n|18 <strong>olm</strong>alıdır. Bu durumda 2, 3, 6 ve 9 değerlerini kontrol etmemiz gerekir.<br />

Ancak bu sayılardan hiçbirisinin denkliği sağlamadığı görülecektir.<br />

Demek ki en küçük sayımız 19 basamaklıdır.<br />

4. m, n > 1 <strong>olm</strong>ak üzere repunit sayımız m × n basamaklıysa, bu sayıyı<br />

111 } {{ · · · 1}<br />

×1 000 } {{ · · · 0}<br />

1 000 } {{ · · · 0}<br />

1 · · · 1 000 } {{ · · · 0}<br />

1<br />

n tane n−1 tane n−1 tane n−1 tane<br />

şeklinde çarpanlara ayırabiliriz. Burada m − 1 tane sıfırlardan oluşmuş<br />

grup vardır. Dolayısıyla bu şekildekii bir repunit asal olamaz. Demek ki,


22 BÖLÜM 2. KONULAR<br />

asal bir repunit için, basamak sayısıda asal <strong>olm</strong>alıdır. Ancak tersi doğru<br />

değildir. Mesela 111 = 3 × 37 ve<br />

sayıları asal değildirler.<br />

111 } {{ · · · 1}<br />

= 21649 × 513239<br />

11 tane<br />

5. Soruda verilen sayıyı çarpanlarına ayırmaya çalışalım. Buna göre,<br />

111 · · · 1 } {{ }<br />

81 tane<br />

= } 111 {{ · · · 1}<br />

9 tane<br />

× 100 · · · 0100 · · · 010 · · · 01<br />

} {{ }<br />

9 tane 1 64 tane 0<br />

olur. Çarpanların her ikiside 9 ile bölünebilir. Dolayısıyla sayımız 81 ile<br />

bölünebilir.<br />

6. (a.) Varsayalım n = 5k + r, r ∈ {0, 1, 2, 3, 4} olsun. Buna göre,<br />

111 · · · 1 } {{ }<br />

n tane<br />

= } 111 {{ · · · 1}<br />

000 } {{ · · · 0}<br />

5k tane r tane<br />

+ 111 · · · 1 } {{ }<br />

r tane<br />

= 11111×100001 · · · 00001+111 · · · 1 } {{ }<br />

r tane<br />

olacaktır. 11111 = 41 × 271 olduğuna göre, ifade mod41 altında<br />

111 } {{ · · · 1}<br />

’e eşittir. Fakat 1, 11, 111, 1111 sayıları 41 ile bölünemez.<br />

r tane<br />

n tane<br />

Dolayısıyla 111 } {{ · · · 1}<br />

sayısının 41 ile bölünebilmesi için ancak ve ancak<br />

r = 0 yani 5|n <strong>olm</strong>alıdır.<br />

(b.) Önceki şıkta kullanılan aynı yöntemle çözüme gidilir. Sadece 111111 =<br />

91 × 1221 olacaktır.<br />

7. Varsayalım sayımız<br />

A = } 111 {{ · · · 1}<br />

= 10n − 1<br />

9<br />

n tane<br />

bir tamkare olsun. Buna göre, A ≡ 111 · · · 1 ≡ 11 ≡ 3 (mod4) olur.<br />

Ancak bu imkansızdır çünkü bir tamkarenin (mod4) altıdaki denkleri<br />

sadece {0, 1} olabilir.


2.3. SOPHİE GERMAİN ÖZDEŞLİĞİ 23<br />

8. Eğer dikkat edilirse 1, 3, 7 ve 9 sayılarının 10 ile aralarında asal oldukları<br />

kolayca görülebilir. Varsayalım elimizde 1, 11, 111, · · · , 111 } {{ · · · 1}<br />

sayılarımız<br />

olsun. Güvercin Yuvası İlkesine göre, bu (n + 1) sayı arasından iki<br />

n+1 tane<br />

tanesi n ile bölündüğünde aynı kalanları verir. Bu sayıların farkı ise n ile<br />

bölünebilir ve fark a · b formundadır. Burada a bir repunit ve b ise 10’un<br />

kuvvetidir. (n, 10) = 1 olduğuna göre, repunit a sayısı 10 ile bölünmelidir.<br />

ispat tamamlanır.<br />

9. Euclide’in sosuz çoklukta asalın varlığını kanıtladığı ispatını sorumuza<br />

adapte edelim. Varsayalım a 1 = 1 olsun ve dizinin elemanlarını a n ’e kadar<br />

seçelim. Bir önceki problemden a 1 · a 2 · · · · · a n çarpımını bölen bir<br />

m repunit sayısı vardır. 10m + 1 sayısıda bir repunittir ve m ile aralarında<br />

asaldırlar. Dolayısıyla 1 ≤ k ≤ n <strong>olm</strong>ak üzere a k ’larda aralarında<br />

asaldırlar. Eğer a n+1 = 10m + 1 seçersek kanıt tamamlanır.<br />

10. Tümevarımla ispatlamaya çalışalım. Buna göre 3 n basamaklı bir repunitin<br />

3 n ile bölünebileceğini göstermemiz yeterli olacaktır. n = 1 için<br />

111 = 3 × 37 olur. Varsayalım n basamak içinde doğru olsun. Buna göre,<br />

111 } {{ · · · 1}<br />

= } 111 {{ · · · 1}<br />

×1 000 } {{ · · · 0}<br />

1 000 } {{ · · · 0}<br />

1<br />

3 n+1 tane 3 n tane 3 n −1 tane 3 n −1 tane<br />

olacaktır. İlk çarpanın 3n ile bölünebildiği açıktır. İkinci çarpanda 3 ile<br />

kalansız bölünebileceğine göre, (n + 1) için ispatı tamamlarız.<br />

2.3 Soph˙ıe Germa˙ın Özdeşl˙ığ˙ı<br />

x 2 + 1 polinomunun R üzerinde indirgenemeyen yada çarpanlarına ayrılamayan<br />

bir polinom olduğunu biliyoruz. Ancak, her nekadar benzer gibi görünsede,<br />

x 4 + 1 polinomu için durum aynı değildir. Öyleki,<br />

x 4 + 1 = ( x 4 + 2x 2 + 1 ) − 2x 2<br />

= ( x 2 + 1 ) 2<br />

(√ ) 2<br />

− 2x<br />

(<br />

= x 2 + √ ) (<br />

2x + 1 x 2 − √ )<br />

2x + 1


24 BÖLÜM 2. KONULAR<br />

olacaktır. Benzer biçimde<br />

x 4 + 4y 4 = ( x 2 + 2xy + 2y 2) ( x 2 − 2xy + 2y 2)<br />

özdeşliğide elde edilebilir. Eğer katsayıyı değitirirsek,<br />

x 4 + 1 4 y4 =<br />

(x 2 + xy + 1 ) (x<br />

2 y2 2 − xy + 1 )<br />

2 y2<br />

olacaktır. Bu son yazdığımız iki özdeşlik sorularn çözümünde oldukça kullanışlıdır<br />

ve literatürdeki adıda Sophie Germain Identity olarak geçer.<br />

Örnek.<br />

olarak verilen toplamı bulunuz.<br />

n∑<br />

k=1<br />

4k<br />

4k 4 + 1<br />

Çözüm.<br />

Eğer bildiğimiz özdeşlikleri kullanırsak,<br />

n∑<br />

k=1<br />

4k<br />

4k 4 + 1<br />

=<br />

=<br />

n∑ (2k 2 + 2k + 1) − (2k 2 − 2k + 1)<br />

(2k 2 + 2k + 1)(2k 2 − 2k + 1)<br />

n∑<br />

(<br />

)<br />

1<br />

(2k 2 + 2k + 1) − 1<br />

(2(k + 1) 2 + 2(k + 1) + 1)<br />

k=1<br />

k=1<br />

= 1 −<br />

1<br />

2n 2 + 2n + 1<br />

olacaktır. Yukarıda çözümlerini verdiğimiz örnekler konunun anlaşılması için<br />

yeterlidir. Şimdi bu bilgiler ışığında çalışma sorularını çözmeye çalışınız.<br />

2.3.1 Çalışma Soruları<br />

1. n > 2, n ∈ Z için 2 2n −2 + 1 sayısının asal <strong>olm</strong>adığını kanıtlayınız.<br />

2.<br />

x n+1 + x n−1 = √ 2x n<br />

dizisinin periyodik olduğunu kanıtlayınız.


2.3. SOPHİE GERMAİN ÖZDEŞLİĞİ 25<br />

3.<br />

n∑<br />

k=1<br />

k 2 − 1/2<br />

k 4 + 1/4<br />

toplamını hesaplayınız.<br />

4.<br />

(<br />

1 4 + 1 ) ( (<br />

)<br />

4 3 4 + 4) 1 · · · · · · (2n − 1) 4 + 1 4<br />

(<br />

2 4 + 1 ) ( ( )<br />

4 4 4 + 4) 1 · · · · · · (2n) 4 + 1 4<br />

ifadesinin eşitini bulunuz.<br />

5. n 4 + a sayısının asal <strong>olm</strong>amasını sağlayan sonsuz sayıda pozitif a değeri<br />

olduğunu kanıtlayınız.<br />

6. n 4 + 4 n sayısının ancak ve ancak n = 1 durumunda asal olduğunu kanıtlayınız.<br />

7. P (x) = x 4 +6x 2 −4x+1 polinomu veriliyor. Buna göre, P (x 4 ) polinomunun<br />

katsayıları tamsayı olan ve derecesi 1’den büyük olan iki polinomun<br />

çarpımı şeklinde yazılabileceğini kanıtlayınız.<br />

8. n 12 + 64 polinomunun her biri 1’den büyük 4 farklı çarpanın çarpımı<br />

olarak yazılabileceğini kanıtlayınız.<br />

9. m, n pozitif tamsayılardır, m bir çift sayı ise<br />

m∑<br />

(−4) k n 4(m−k)<br />

k=0<br />

toplamının asal <strong>olm</strong>adığını kanıtlayınız.<br />

10. P (x) = x n−4 + 4n polinomunun, sabit polinomdan farklı 4 polinomun<br />

çarpımı şeklinde yazılmasını sağlayan en küçük n değeri kaçtır


26 BÖLÜM 2. KONULAR<br />

2.3.2 Çözümler<br />

1. Soruda verilen ifadeyi<br />

olarak yazabiliriz . Buna göre,<br />

2 2n −2 + 1 = 1 + m4<br />

4 , m = 22n−2<br />

1 + m4<br />

4 = (<br />

1 + m + 1 2 m2 ) (<br />

1 − m + 1 2 m2 )<br />

olacağından sayımız asal değildir.<br />

2. x 4 + 1 polinomunu çarpanlarına ayırırsak,<br />

x 4 + 1 =<br />

(<br />

x 2 − √ ) (<br />

2x + 1 x 2 + √ )<br />

2x + 1<br />

olacaktır. Buna α ve β sayıları x 2 + √ 2x + 1 karakteristiğinin kökleri<br />

ise bunlar aynı zamanda x 4 + 1 polinomunun kökleridir. Benzer biçimde<br />

x 2 + √ 2x + 1 içinde aynı durum söz konusu ise, x n = a · α n + b · β n<br />

eşitliğinden, dizinin periyodunun 8 olduğu bulunur.<br />

3. k 4 + 1 4 = ( k 2 − k + 1 2) (<br />

k 2 + k + 1 2)<br />

olduğuna göre,<br />

n∑<br />

k=1<br />

k 2 − 1/2<br />

k 4 + 1/4 =<br />

n ∑<br />

k=1<br />

( k − 1/2<br />

k 2 − k + 1/2 − k + 1/2 )<br />

k 2 =<br />

+ k + 1/2<br />

(<br />

n∑ k − 1/2<br />

k 2 − k + 1/2 − (k + 1) −<br />

(k + 1) 2 − (k +<br />

k=1<br />

olacağından toplam<br />

olarak bulunacaktır.<br />

1 −<br />

(2n + 1)<br />

(2n 2 + 2n + 1)<br />

4. Soruda verilen ifadeyi genelleştirirsek,<br />

n∏ (2k − 1) 4 + 1/4<br />

k=1<br />

(2k) 4 + 1/4


2.3. SOPHİE GERMAİN ÖZDEŞLİĞİ 27<br />

ifadesini elde ederiz. Eğer bu ifadenin pay ve paydasını çarpanlına ayırırsak,<br />

(<br />

) (<br />

)<br />

n∏ (2k − 1) 2 + (2k − 1) + 1/2 (2k − 1) 2 − (2k − 1) + 1/2<br />

(<br />

) (<br />

)<br />

(2k) 2 + 2k + 1/2 (2k) 2 − 2k + 1/2<br />

k=1<br />

durumunu elde ederiz. Ancak<br />

m 2 − m + 1/2 = (m − 1) 2 + (m − 1) + 1/2<br />

olduğundan pay ve paydadaki çarpanlardan birer tanesi sadeleşir. Sadece<br />

1 2 − 1 + 1/2 pay kısmında ve (2n) 2 + 2n + 1/2 paydada kalır. Dolayısıyla<br />

istenilen cevap<br />

1<br />

8n 2 + 4n + 1<br />

olacaktır.<br />

5. Eğer a = 4k 4 , k > 1 olarak alırsak,<br />

n 4 + 4k 4 = ( n 2 + 2nk + 2k 2) ( n 2 − 2nk + 2k 2)<br />

olacaktır. Buna göre, n 2 + 2nk + 2k 2 > k > 1 ve n 2 − 2nk + 2k 2 =<br />

(n − k) 2 + k 2 > k 2 > 1 olduğundan n 4 + 4k 4 asal olamaz ve sonsuz<br />

sayıda a değeri seçilebilir.<br />

6. Eğer n çift bir sayı ise n 4 +4 n çift olacaktır. Eğer n tek sayı ve n = 2k+1<br />

ise,<br />

(2k + 1) 4 + 4 (2k+1) = (2k + 1) 4 + 4 · 4 2k = (2k + 1) 4 + 4 (2) 4k<br />

olacaktır. Eğer (2k + 1) = a ve 2 k = b olarak alırsak, ifademiz<br />

a 4 + 4b 4 = ( a 2 + 2ab + 2b 2) ( a 2 − 2ab + 2b 2)<br />

olacaktır. Buna göre ifademiz,<br />

((2k + 1) 2 + 2 (2k + 1) 2 k + 2 · 2 2k) ( (2k + 1) 2 − 2 (2k + 1) 2 k + 2 · 2 2k)


28 BÖLÜM 2. KONULAR<br />

olacaktır. Buradan da,<br />

(n 2 + 2 k+1 n + 2 2k+1) ( n 2 − 2 k+1 n + 2 2k+1)<br />

olacaktır. Eğer n > 1 ise her iki çarpanda 1’den büyük olacaktır. Dolayısıyla<br />

n = 1 durumu tek asal olduğu durumdur.<br />

7. P ( x 4) = x 16 + 6x 8 − 4x 4 + 1 = ( x 4 − 1 ) 4 + 4<br />

(<br />

x<br />

3 ) 4 olacaktır. Buna göre,<br />

A = x 4 − 1 ve B = x 3 olarak alırsak P (x 4 ) polinomu<br />

[ (x 4 − 1 ) 2<br />

+ 2<br />

(<br />

x 4 − 1 ) x 3 + 2 ( x 3) 2 ] [ (<br />

x 4 − 1 ) 2<br />

− 2<br />

(<br />

x 4 − 1 ) x 3 + 2 ( x 3) 2 ]<br />

olur. İspat tamamlanır.<br />

8. Sophie Germain özdeşliğini kullanırsak<br />

n 12 + 64 = ( n 6 − 4n 3 + 8 ) ( n 6 + 4n 3 + 8 )<br />

olarak yazabiliriz. Diğer yandan, n 12 +64 ifadesi ( n 4 + 4 ) ( n 8 − 4n 4 + 16 )<br />

biçiminde de yazılabilir. Burada ( n 4 + 4 ) = ( n 2 − 2n + 2 ) ( n 2 + 2n + 2 )<br />

olduğuna göre ifademiz n 12 = ( n 2 − 2n + 2 ) ( n 2 + 2n + 2 ) ( n 8 − 4n 4 + 16 )<br />

olacaktır. Şimdi de (n 6 − 4n 3 + 8) ve (n 6 + 4n 3 + 8) çarpanlarını inceleyelim.<br />

Buna göre,<br />

n 6 + 4n 3 + 8 = ( n 2 − 2n + 2 ) ( n 4 + 2n 3 + 2n 2 + 4n + 4 )<br />

ve<br />

olacağına göre<br />

n 6 − 4n 3 + 8 = ( n 2 + 2n + 2 ) ( n 4 − 2n 3 + 2n 2 − 4n + 4 )<br />

n 12 +64 = ( n 2 − 2n + 2 ) ( n 2 + 2n + 2 ) ( n 4 + 2n 3 + 2n 2 + 4n + 4 ) ( n 4 − 2n 3 + 2n 2 −<br />

olacaktır. Çarpanlardan herbiri artan fonksiyonlar olduğuna göre, hepsi<br />

birbirinden farklıdır.<br />

9. Geometrik dizi toplamını kullanırsak,<br />

m∑<br />

(−4) k n 4(m−k) =n 4m<br />

k=0<br />

n∑<br />

k=0<br />

(<br />

− 4 n) k<br />

=<br />

(<br />

n<br />

4 ) m+1 + 4<br />

m+1<br />

n 4 + 4<br />

=<br />

(<br />

n<br />

m+1 ) 4 + 4<br />

(<br />

2<br />

m/2 ) 4<br />

n 4 + 4


2.3. SOPHİE GERMAİN ÖZDEŞLİĞİ 29<br />

eşitliğini elde edebiliriz. Sophie Germain özdeşliğini kullanarak kesrin<br />

pay kısmı çarpanlarına ayrılabilir. Buna göre ifade,<br />

(n 2(m+1) + 2 m 2 +1 · n m+1 + 2 m+1) ( n 2(m+1) − 2 m 2 +1 · n m+1 + 2 m+1)<br />

olacaktır. m ≥ 2 olduğundan, payda kısmı paydaki iki çarpandan da küçük<br />

olacaktır. Sadeleştirmelerden sonra bile, kalan sayı hala 1’den büyük<br />

iki sayının çarpımı olacaktır. Buna göre, ispat tamamlanır.<br />

10. Çözüme başlamadan önce Einstein Kriteri’ni bir okuyalım.<br />

Einstein Kriteri. P (x) = a n x n + a n−1 x n−1 + · · · + a 0 , tamsayı katsayılı<br />

bir polinom veriliyor. Varsayalım p asal sayı <strong>olm</strong>ak üzere, p ∤<br />

a n , p|a k , k = 1, 2, · · · , n − 1 ve p 2 ∤ a 0 olsun. Buna göre, P (x) polinomu<br />

Z[x] üzerinde indirgenebilir.<br />

Biz en küçük değerin 16 olduğunu göstereceğiz, ancak ilk 15 değerin <strong>olm</strong>adığını<br />

kanıtlamamız gerekiyor. Bunun içinde Einstein Kriteri’ni kullanacağız.<br />

n = 10, 11, 12, 13, 14, 15 için kriterimiz 5, 11, 3, 13, 7 ve 5 asallarını uygulayarak<br />

kontrol edebiliriz.<br />

n = 8 veya 9 durumunda, eğer polinomumuz istenilen gibi çarpanlarına<br />

ayrılabiliyorsa, çarpanlar lineer <strong>olm</strong>alıdır. Ancak kontrol edilebilirki bu<br />

polinomların tamsayı kökleri <strong>olm</strong>ayacaktır. Demek ki, n ≥ 16 <strong>olm</strong>alıdır.<br />

Bu durumda da, zaten sorumuz (8.) örmekte olduğu gibi çözülebilir.<br />

Sophie Germain 2 (1776-1831)<br />

Babası zengin bir ipek tüccarıdır. Toplumda liberal reformların konuşulup<br />

planlandığı bir ailenin kızı olarak 1776 yılında doğar. Daha 13 yaşındayken<br />

Arşimet’in ölüm hikayesini okuduktan sonra <strong>mat</strong>e<strong>mat</strong>ikçi <strong>olm</strong>aya karar verir.Kendi<br />

kendine Latince ve Yunanca öğrenir. Ailesinin muhalefetine rağmen,<br />

anne ve babası uyduktan sonra Newton ve Euler’i okur. Felsefeye merak sarar.<br />

Bu kadar inatçı bir çocukla baş edemeyen babası sonunda Sophie’yi hayatı<br />

boyunca desteklemeye karar verir. Mate<strong>mat</strong>ikteki zekasını ilk kez meşhur <strong>mat</strong>e<strong>mat</strong>ikçi<br />

Lagrange keşfeder. Lagrange için hazırladığı bir ödevi kadın olduğundan<br />

önem verilmeyeceği kaygısıyla “M. LeBlanc” diye sahte bir isimle verir.<br />

2 Hürriyet gazetesi yazarı, Erkan Kumcu’nun 9 Şubat 2003 tarihli yazısından alınmıştır.


30 BÖLÜM 2. KONULAR<br />

Lagrange bu dehanın Sophie Germain olduğunu daha sonra öğrenir. Sophie’nin<br />

<strong>mat</strong>e<strong>mat</strong>ik alanında en büyük destekçilerinden biri Lagrange olur. Sophie Germain’i<br />

en çok etkileyen <strong>mat</strong>e<strong>mat</strong>ikçilerden biri de çoğu kesimlerin fikir birliği<br />

içinde <strong>mat</strong>e<strong>mat</strong>iğin prensi diye adlandırılan Gauss oldu. Ona da çeşitli <strong>mat</strong>e<strong>mat</strong>ik<br />

konularında bir çok mektup yazdı. Aynı kaygıyla, mektuplarına uzun<br />

süre M. LeBlanc olarak imza attı.<br />

Gauss, M. LeBlanc’ın Sophie Germain olduğunu<br />

Fransızlar Gauss’un oturduğu şehri işgal edip Sophie’nin<br />

aile dostu olan bir Fransız generalden Gauss<br />

için ayrıcalık istediğinde öğrenir. Sophie Germain’in<br />

<strong>mat</strong>e<strong>mat</strong>ikteki meşhur Fer<strong>mat</strong> Teoremi’nin<br />

çözümüne yaptığı katkılar bilinen en iyi yönüdür.<br />

Yaptığı katkıların önemi kendinden ancak 100 yıl<br />

sonra Kummer tarafından bir adım ileri götürülebildiği<br />

düşünülürse daha iyi anlaşılır. Zamanın çok<br />

prestijli yarışmalarına katılmıştır. Poisson gibi <strong>mat</strong>e<strong>mat</strong>ik<br />

ve istatistiğin önde gelen isimleriyle yarışmıştır.<br />

Başarılı olamamıştır. Hak ettiği dereceler hiçbir<br />

zaman kendine verilmemiştir. Geçmişte M. LeBlanc<br />

ismini kullanmakla ne kadar haklı olduğunu tüm <strong>mat</strong>e<strong>mat</strong>ik dünyası adete<br />

Sophie’ye ispat etmiştir.Poisson, Gaspard de Prony ve Laplace’dan oluşan bir<br />

jürinin seçiciliğinde katıldığı bir yarışmada sunduğu makale bazı teknik hatalar<br />

nedeniyle kabul dahi edilmemiş ve kendisine çalışmasının neden kabul edilmediği<br />

söylenmemiştir bile. Olaydan 55 yıl sonra Gaspard de Prony’nin yazdığı<br />

makalelerinden birinin Sophie Germain’in yazdığı makalenin düzeltilmiş şekli<br />

olduğu anlaşılmıştır.Bir çok deha gibi, Sophie Germain de çok genç yaşta öldü.<br />

2.4 Tamkareler Poz˙ıt˙ıft˙ır<br />

Bu bölümde, cebirde oldukça basit bir eşitsizlik olan<br />

x 2 ≥ 0,<br />

∀x ∈ R<br />

eşitsizliğinin bazı uygulamalarını yapacağız. Eşitlik durumu yanlız ve yanlız<br />

x = 0 durumunda vardır. Şimdi örneklerle konuyu kavramaya çalışalım.


2.4. TAMKARELER POZİTİFTİR 31<br />

Örnek.<br />

4x − x 4 ≤ 3, x ∈ R eşitsizliğini kanıtlayınız.<br />

Çözüm. Bu örnek eşitsizlikleri öğrenmeye başlayan genç bir öğrenci<br />

tarafından ortaya atılmıştır. Henüz herhangi bir analiz kitabından eşitsizlikler<br />

konusunu çalışmadığını düşünürsek fena bir örnek değildir. Buna göre,<br />

x 4 − 4x + 3 0 ⇒ x 4 − 2x 2 + 1 + 2x 2 − 4x + 2 0<br />

olacaktır. Buradan,<br />

(<br />

x 2 − 1 ) 2<br />

+ 2 (x − 1) 2 0<br />

olacaktır ki, ispat tamamlanır.<br />

Örnek. f ( x 2) −(f (x)) 2 1/4 eşitsizliğini sağlayan f : R → R ve birebir<br />

olan tüm f fonksiyonlarını bulunuz.<br />

Çözüm. x = 0 alırsak, f(0) − (f(0)) 2 ≥ 1/4 ve (f (0) − 1/2) 2 0<br />

olacaktır ki, bu durum imkansızdır ancak f(0) = 1/2 olabilir. Ancak x = 1<br />

alırsak bu seferde f(1) = 1/2 olacaktır. Ancak bu durum f fonksiyonun birebir<br />

<strong>olm</strong>ası durumu ile çelişir.<br />

Şimdi alıştırmaları çözerek konuyu daha iyi kavramaya çalışınız. Alıştırmalardaki<br />

yöntemlere benzer yöntemleri uygulayarak çözüme gitmeye gayret<br />

ediniz.<br />

2.4.1 Çalışma Soruları<br />

1. n tane reel sayının ikişerli çarpımları toplamı ve toplamları sıfırdır. Buna<br />

göre, bu sayıların küpleri toplamınında sıfır olduğunu kanıtlayınız.<br />

2. a, b, c, d birer reel sayı <strong>olm</strong>ak üzere, a−b 2 , b−c 2 , c−d 2 ve d−a 2 sayılarının<br />

tümünün birden 1/4’ten büyük olamayacağını kanıtlayınız.<br />

3. x, y, z pozitif reel sayılarının hepsi 4 ten küçüktür. Buna göre,<br />

1<br />

x + 1<br />

4 − y , 1<br />

y + 1<br />

4 − x , 1<br />

z + 1<br />

4 − x<br />

sayılarından en az bir tanesinin 1’den büyük veya eşit olduğunu kanıtlayınız.


32 BÖLÜM 2. KONULAR<br />

4. Aşağıdaki denklem sisteminin tüm reel çözümlerini bulunuz.<br />

x + y = √ 4z − 1<br />

y + z = √ 4x − 1<br />

z + x = √ 4y − 1<br />

5. x, t ∈ (0, 1) <strong>olm</strong>ak üzere, öyle bir a ≠ 1 pozitif sayısı vardır ki,<br />

log x a + log y a = 4 log xy a<br />

eşitliği sağlanmaktadır. Buna göre, x = y olduğunu kanıtlayınız.<br />

6.<br />

x 4 + y 4 + z 4 − 4xyz = −1<br />

eşitliğini sağlayan tüm (x, y, z) reel üçlülerini bulunuz.<br />

7.<br />

2xy − z 2 ≥ 1<br />

z − |x + y| ≥ −1<br />

denklem sistemini sağlayan tüm (x, y, z) reel üçlülerini bulunuz.<br />

8. x 4 + ax 3 + 2x 2 + bx + 1 denkleminin reel çözümü yoksa a 2 + b 2 ≥ 8<br />

olacağını gösteriniz.<br />

9. a, b ve c reel sayılar <strong>olm</strong>ak üzere a 2 + c 2 4b olarak veriliyor. Buna göre<br />

x ∈ R <strong>olm</strong>ak üzere<br />

10.<br />

eşitsizliğini kanıtlayınız.<br />

x 4 + ax 3 + bx 2 + cx + 1 0<br />

x 2 + y 2 + z 2 − xy − yz − xz 3 (x − y)2<br />

4<br />

eşitsizliğinin tüm (x, y, z) reel değerleri içi sağlandığını kanıtlayınız.


2.4. TAMKARELER POZİTİFTİR 33<br />

11.<br />

√<br />

x1 − 1 2 + 2 √ x 2 − 2 2 + · · · · · · + n √ x n − n 2 = 1 2 (x 1 + x 2 + · · · + x n )<br />

eşitliğini sağlayan tüm (x 1 , x 2 , · · · , x n ) reel sayılarını bulunuz.<br />

12. a. a, b, c negatif <strong>olm</strong>ayan reel sayılardır. Buna göre,<br />

eşitsizliğini kanıtlayınız.<br />

ab + bc + ca √ 3abc (a + b + c)<br />

b. a, b, c negatif <strong>olm</strong>ayan reel sayılardır, öyleki a + b + c = 1 veriliyor.<br />

Buna göre,<br />

eşitsizliğini kanıtlayınız.<br />

13. k, m, n ∈ R <strong>olm</strong>ak üzere verilen<br />

a 2 + b 2 + c 2 + √ 12abc 1<br />

f (km) + f (kn) − f (k) · f (mn) 1<br />

eşitsizliğini sağlayan tüm f : N → R fonksiyonlarını bulunuz.<br />

14. Bir dik paralelyüzün kenar uzunlukları a, b, c ve köşegen uzunluğu d ise<br />

eşitsizliğini kanıtlayınız.<br />

a 2 b 2 + b 2 c 2 + c 2 a 2 abcd √ 3<br />

15. a 1 , a 2 , · · · , a n reel sayılar <strong>olm</strong>ak üzere<br />

n∑<br />

eşitsizliğini kanıtlayınız.<br />

i=1 j=1<br />

n∑<br />

i · j · cos (a i − a j ) 0


34 BÖLÜM 2. KONULAR<br />

2.4.2 Çözümler<br />

1. Varsayalım sayılarımız a 1 , a 2 , · · · , a n olsun. Soruda verilen şartlara göre,<br />

a 2 1+a 2 2+· · ·+a 2 n = (a 1 + a 2 + · · · + a n ) 2 −2 (a 1 a 2 + a 1 a 3 + · · · + a n−1 a n ) = 0<br />

olduğuna göre, a 1 = a 2 = · · · = a n = 0 olacağına göre,a 3 1 +a3 2 +· · ·+a3 n =<br />

0 olacaktır.<br />

2. Varsayalım tümü birden,<br />

a − b 2 > 1 4 , b − c2 > 1 4 , c − d2 > 1 4 , d − a2 > 1 4<br />

olsun. Eğer bu dört eşitsizliği toplarsak,<br />

a + b + c + d − ( a 2 + b 2 + c 2 + d 2) > 1<br />

olacaktır. Buradan,<br />

( ) 1 2 ( ) 1 2 ( ) 1 2 ( ) 1 2<br />

2 − a +<br />

2 − b +<br />

2 − c +<br />

2 − d < 0<br />

olacaktır ki, bu durum imkansızdır.<br />

3. Soruya göre, varsayalım hepsi birden 1’den küçük olsun. Buna göre,<br />

( 1<br />

x + 1 ) ( 1<br />

+<br />

4 − x y + 1 ) ( 1<br />

+<br />

4 − y z + 1 )<br />

< 3<br />

4 − z<br />

olur. Diğer taraftan<br />

( 1<br />

a + 1 )<br />

1, a < 4<br />

4 − a<br />

olacağından (a − 2) 2 ≥ 2 olacaktır. İspat tamamlanır.<br />

4. Çözüm I. Bir önceki soruda olduğu gibi, üç denklemide taraf tarafa<br />

toplayıp, kareler tolamını elde etmeye çalışalım. Buna göre,<br />

2x + 2y + 2z − √ 4x − 1 − √ 4y − 1 − √ 4z − 1 = 0


2.4. TAMKARELER POZİTİFTİR 35<br />

olacaktır. Bu eşitliği öyle kareler toplamı şeklinde yazalım ki, bunlardan<br />

biri x, biri y ve biride z değişkenine bağlı olsun. Eğer tüm denklemi 2 ile<br />

bölersek elimizde<br />

√<br />

x −<br />

x − 1 4<br />

oluşacaktır. Eğer bu denkleme 1/4 ekler çıkarırsak<br />

x − 1 4 − √<br />

x − 1 4 + 1 4 = (√<br />

x − 1 4 − 1 2<br />

) 2<br />

oluşacaktır. Şimdi bu yöntemi diğerleri içi uygularsak,<br />

(√<br />

2 (√<br />

2 (√<br />

2<br />

x − 1 4 2) − 1 + y − 1 4 2) − 1 + z − 1 4 2) − 1 = 0<br />

olacaktır. Buna göre, x = y = z = 1/2 denklem sisteminin tek çözümü<br />

olarak bulunur.<br />

Çözüm II. Eğer denklemlerin karelerini alıp toplarsak,<br />

(<br />

) (<br />

) (<br />

)<br />

0 = (x + y) 2 − 4z + 1 + (y + z) 2 − 4x + 1 + (z + x) 2 − 4y + 1<br />

= (x + y − 1) 2 + (y + z − 1) 2 + (z + x − 1) 2<br />

olacağından x + y = y + z = z + x = 1 veya x = y = z = 1/2 bulunur.<br />

5. a > 0 ve a ≠ 1 olduğuna göre eşitlik<br />

veya<br />

1<br />

log a x + 1<br />

log a y = 4<br />

log a xy<br />

log a x + log a y<br />

log a x log a y = 4<br />

log a x + log a y<br />

olarak yazılabilir. Eğer içler dışlar çarpımı yapılırsa,<br />

(log a x + log a y) 2 = 4 log a x log a y


36 BÖLÜM 2. KONULAR<br />

eşitliği elde edilir. Buradan da,<br />

olacağından x = y olduğu açıktır.<br />

(log a x − log a y) 2 = 0<br />

6. Eğer eşitliğin sol tarafını ilk iki terimi içeren bir tamkare olarak yazmaya<br />

çalışırsak<br />

−1 = ( x 2 − y 2) 2<br />

+ 2x 2 y 2 + z 2 − 4xy<br />

olacaktır. Bu noktadan sonra denkleme 2z 2 ekler ve çıkarırsak bir tamkare<br />

daha elde ederiz. Zaten bu basamaktan sonra yeni denklemimiz üç<br />

karenin toplamı<br />

(<br />

x 2 − y 2) 2<br />

+<br />

(<br />

z 2 − 1 ) 2<br />

+ 2 (xy − z) 2 = 0<br />

olacaktır. Eşitlik ancak hepsinin sıfır <strong>olm</strong>ası durumunda geçerlidir. Buna<br />

göre, z = ±1 olacağından istenen üçlüler<br />

olacaktır.<br />

(1, 1, 1) , (−1, −1, 1) , (−1, 1, −1) , (1, −1, −1)<br />

7. Çözüm I. ikinci denklemden z ≥ |x + y| − 1 olacağından, bunu ilk<br />

denklemde yerine koyarsak<br />

olacaktır. Buna göre,<br />

2xy − (1 − |x + y|) 2 1<br />

2xy − (1 − |x + y|) 2 = 2xy − |x + y| 2 + 2 |x + y| − 1<br />

= 2xy − x 2 − y 2 − 2xy + 2 (±x ± y) − 1<br />

= −x 2 − y 2 + (±x ± y) − 1<br />

eğer işaret seçimimizi uygularsak, yeni denklem aşağıdaki gibi olacaktır.<br />

Buna göre,<br />

0 x 2 + y 2 − 2 (±x ± y) + 1 + 1 = (1 ± x) 2 + (1 ± y) 2


2.4. TAMKARELER POZİTİFTİR 37<br />

olacağından her iki karede sıfır <strong>olm</strong>alıdır. Yani x = y = ±1 değerlerini<br />

alacaklardır. Birinci denklemden xy > 0 olacağına göre x ve y aynı<br />

işaretli <strong>olm</strong>alıdır. Yani x = y = 1 veya x = y = −1 <strong>olm</strong>alıdır. Eğer<br />

bu değerleri yerine koyarsak x = y = z = 1 ve x = y = 1, z = 1 çözümler<br />

olacaktır.<br />

Çözüm II.<br />

alırsak<br />

Eğer ikinci denklemi z + 1 ≥ |x + y| olarak yazıp karesini<br />

(x + y) 2 ≥ (x + y) 2<br />

olacaktır. Birinci eşitsizliği 2 ile çarpıp son yazdığımız eşitsizliği toplarsak<br />

0 ≥ (x − y) 2 + (z − 1) 2<br />

olacaktır. Demek ki x = y ve z = 1 <strong>olm</strong>alıdır. Eğer bu eşitlikleri yerine<br />

koyarsak 2x 2 ≥ 2 ve 2 ≥ 2|x| ise |x| = 1 <strong>olm</strong>alıdır.<br />

8.<br />

x 4 + ax 3 + 2x 2 + bx + 1 =<br />

(x 2 + a ) ( 2<br />

2 x + 1 + b ) 2<br />

2 x + 1 (<br />

8 − a 2 − b 2) x 2<br />

4<br />

eşitliğinden bu ifadenin pozitif olduğunu söyleyebiliriz. Ancak, 8 − a 2 −<br />

b 2 ≤ 0 ise yani a 2 + b 2 ≥ 8 olursa durum bozulur.<br />

9.<br />

x 4 + ax 3 + bx 2 + cx + 1 =<br />

(x 2 + a ) 2<br />

2 x +<br />

(b − a2 + c 2 ) ( c<br />

) 2<br />

x 2 +<br />

4<br />

2 x + 1<br />

olduğuna ve b ≥ (a 2 + c 2 )/4 ise ifade daima sıfırdan büyük eşittir.<br />

10. Eğer eşitsizliği düzenlersek<br />

olacağından<br />

1<br />

[<br />

(x − y) 2 + (y − z) 2 + (z − x) 2] 3 (x − y)2<br />

2<br />

4<br />

[<br />

2 (y − z) 2 + (z − x) 2] (x − y) 2


38 BÖLÜM 2. KONULAR<br />

olacaktır. Eğer a = y − z ve b = z − x alırsak<br />

2 ( a 2 + b 2) (a + b) 2<br />

olur ki, doğrudur, ispat tamamlanır.<br />

11. Yine bu sorudada amacımız, verilen denklemi kareler toplamı biçiminde<br />

yazmak olacaktır.Buna göre denklemi 2 ile genişletip, hepsini sol tarafa<br />

toplarsak<br />

(x 1 + x 2 + · · · + x n ) − 2 √ x 1 − 1 − 4 √ x 2 − 2 2 − · · · − 2n √ x n − n 2<br />

ifadesini elde ederiz. Buna göre,<br />

(<br />

x1 − 1 − 2 √ x 1 − 1 + 1 ) +(<br />

x 2 − 2 2 − 4 √ x 2 − 2 2 + 2 2) (<br />

+· · ·+ x n − n 2 − 2n √ x n − n<br />

ifadesi elde edilir. Eğer bu ifade de kareler toplamı olarak yazılırsa<br />

(√<br />

x1 − 1 − 1 ) (√ )<br />

2 2 (√ 2<br />

+ x2 − 2 2 − 2 + · · · + xn − n 2 − n)<br />

ifadesi elde edilir. Eğer dikkat edilirse tüm toplamın sıfır olabilmesi için<br />

tamkare ifadelerin de sıfır <strong>olm</strong>ası gerektiğini görmek zor değildir. Buna<br />

göre,<br />

x 1 = 2, x 2 = 8, · · · , x n = 2n 2<br />

olacaktır.<br />

12. a. Eşitsizliğin iki tarafınında karesini alırsak<br />

a 2 b 2 + b 2 c 2 + c 2 a 2 + 2abc (a + b + c) 3abc (a + b + c)<br />

olduğuna göre,<br />

1<br />

[<br />

(ab − bc) 2 + (bc − ca) 2 + (ca − ab) 2] 0<br />

2<br />

olacaktır. İspat tamamlanır.


2.4. TAMKARELER POZİTİFTİR 39<br />

b. a + b + c = 1 ise soruda verilen eşitsizliği,<br />

√<br />

12abc (a + b + c) 2 (a + b + c) 2 − ( a 2 + b 2 + c 2)<br />

olarak yazabiliriz. Buna göre eşitsizliğimiz<br />

√<br />

12abc (a + b + c) 2 (ab + bc + ac)<br />

olacaktır. Zaten benzer eşitsizliği (a.) şıkkında çözmüştük. Aynı<br />

yöntemle siz tamamlayabilirsiniz.<br />

13. Bu bölümün konu anlatım bölümünde çözdüğümğz örnekte olduğu gibi<br />

buradada m, n ve k için bazı değerleri kontrol edeceğiz. Eğer m = n =<br />

k = 0 alırsak<br />

2f (0) − f 2 (0) 1<br />

olacağından 0 (f(0) − 1) 2 olacaktırki, buradan f(0) = 1 olarak bulunur.<br />

Eğer m = n = k = 1 alırsak, bu seferde f(1) = 1 olacaktır. Eğer<br />

m = n = 0 alınırsa 2f(k) ≥ 1 olacağından f(k) ≤ 1 olarak bulunur.<br />

Benzer biçimde k = 1 ve m = 0 alırsak 1 + f(n) − 1 ≥ 1 ise f(n) ≥ 1<br />

olacaktır. Demek ki fonksiyonumuz f(x) = 1 fonksiyonu <strong>olm</strong>alıdır.<br />

14. Elimizde dik bir paralelyüz olduğuna göre<br />

olacağından eşitsizlik<br />

d = √ a 2 + b 2 + c 2<br />

(<br />

a 2 b 2 + b 2 c 2 + c 2 a 2) 2<br />

3a 2 b 2 c 2 ( a 2 + b 2 + c 2)<br />

olacaktır. Eğer parantezleri açar ve gruplama yaparsak,<br />

c 2<br />

2<br />

(<br />

a 2 − b 2) 2 a 4 (<br />

+ b 2 − c 2) 2 b 4<br />

+<br />

2<br />

2<br />

(<br />

c 2 − a 2) 2<br />

0<br />

olacağından soruda verilen eşitsizlik kanıtlanmış olur.


40 BÖLÜM 2. KONULAR<br />

15. Kosinüs için fark formülünü kullanırsak,<br />

n∑ n∑<br />

ij cos (a i − a j ) =<br />

i=1 j=1<br />

=<br />

n∑ n∑<br />

ij (cos a i cos a j + ij sin a i sin a j )<br />

i=1 j=1<br />

n∑<br />

i cos a i<br />

i=1<br />

n∑<br />

n∑<br />

j cos a j + i sin a i<br />

j=1<br />

i=1 j=1<br />

n∑<br />

j sin a j<br />

=<br />

( n∑<br />

) 2<br />

i cos a i +<br />

i=1<br />

j sin a j<br />

) 2<br />

0<br />

( n∑<br />

i=1<br />

olduğundan ispat tamamlanmış olur.<br />

2.5 Eş˙ıts˙ızl˙ıkler I<br />

Eşitsizlikleri çözerken sıklıkla sayıları ve <strong>mat</strong>e<strong>mat</strong>iksel ifadeleri karşılaştırırız.<br />

Yada bize verilen bir <strong>mat</strong>e<strong>mat</strong>iksel ifadenin en büyük yada en küçük<br />

değerini bulmaya çalışırız. Bu ders notumuzda eşitsizlik sorularında sıklıkla<br />

karşımıza çıkan temel tip eşitsizlikleri ve uygulamalarını göreceğiz.<br />

Aritmetik Orta-Geometrik Orta-Harmonik Orta Eşitsizliği. a 1 , a 2 , · · · , a n ><br />

0 ve a i ∈ R için,<br />

AO = a 1 + · · · + a n<br />

n<br />

≥ GO = n√ n<br />

a 1 · a 2 · · · a n ≥ HO =<br />

1<br />

a 1<br />

+ 1 a 2<br />

+ · · · + 1<br />

a n<br />

olacaktır. Eşitlik <strong>olm</strong>ası durumunda ise a 1 = a 2 = · · · = a n olacaktır.<br />

Örnek.(1)A0 − GO eşitsiliği ile, x > 0 <strong>olm</strong>ak üzere,<br />

x + 1 √<br />

x ≥ 2 x · 1<br />

x = 2<br />

olacaktır. Eşitlik ise x = 1 durumunda sağlanacaktır.<br />

(2) AO − HO eşitsizliğinden eğer a 1 , a 2 , · · · , a n > 0 ise<br />

(a 1 + a 2 + · · · + a n )( 1 a 1<br />

+ 1 a 2<br />

+ · · · + 1<br />

a n<br />

) ≥ n 2


2.5. EŞİTSİZLİKLER I 41<br />

olacaktır.<br />

(3) a, b, c > 0 ve abc = 1 ise (a + b + c)(ab + cb + ac) çarpımının en küçük<br />

değerini bulunuz.<br />

Çözüm.AO − GO eşitsizliğinden,<br />

a + b + c<br />

3<br />

≥ 3√ abc ve<br />

ab + bc + ac<br />

3<br />

≥ 3√ ab · bc · ac<br />

ise<br />

(a + b + c)(ab + ac + bc) ≥ 9<br />

olacaktır. Buna göre istenen en küçük değer 9 olacaktır.<br />

(4) n ∈ Z + ise<br />

eşitsizliğini kanıtlayınız.<br />

(1 + 1 n )n ≤ (1 + 1<br />

n + 1 )n+1<br />

Çözüm. Varsayalım a 1 = a 2 = · · · = a n = 1 + 1 n ve a n+1 = 1 olarak alalım.<br />

Buna göre, AO − GO eşitsizliğinden<br />

AO = n(1 + 1 n ) + 1<br />

n + 1<br />

eşitsizliğinden soruda istenen durum elde edilir.<br />

= 1 + 1<br />

n + 1 ≥ GO = n+1 √<br />

(1 + 1 n )n · 1<br />

(5)(1964, IMO) a, b, c bir üçgenin kenar uzunluklarıdır. Buna göre,<br />

eşitizliğini kanıtlayınız.<br />

Çözüm. Varsayalım,<br />

a 2 (b + c − a) + b 2 (c + a − b) + c 2 (a + b − c) ≤ 3abc<br />

x = a + b − c<br />

3<br />

, y = b + c − a , z = c + a − b<br />

2<br />

2<br />

olarak alalım. Buradan a = x+z, b = x+y, c = y+z olacağından eşitsizliğimiz<br />

(x + z) 2 2y + (x + y) 2 2z + (y + z) 2 2x ≤ 3(z + x)(x + y)(y + z)


42 BÖLÜM 2. KONULAR<br />

olacaktır. Eğer son bulduğumuz son eşitsizliği düzenlersek,<br />

x 2 y + y 2 z + z 2 x + xy 2 + yz 2 + zx 2 ≥ 6xyz<br />

eşitsizliğini elde ederiz. Buradan eşitsizliğin sol tarafına AO − GO eşitsizliği<br />

uygulanırsa soruda istenen eşitsizlik kanıtlanmış olur.<br />

(6)(2008, AUMO 3 ) n doğal sayısının kaç tane değeri için,<br />

x 1 + x 2 + · · · + x n = 9<br />

1<br />

+ 1 + · · · + 1<br />

x 1 x 2 x n<br />

= 1<br />

denklem siteminin pozitif reel sayılarda çözümü vardır<br />

Çözüm. AO − HO eşitsizliğini kullanırsak,<br />

n<br />

1<br />

x 1<br />

+ 1 x 2<br />

+ · · · + 1<br />

x n<br />

≤ x 1 + x 2 + · · · + x n<br />

n<br />

⇒ n 1 ≤ 9 n ⇒ n2 ≤ 9<br />

olduğuna göre, n = 1, 2, 3 değerlerini alabilir. Ancak, n = 1 değeri için doğrulanmadığı<br />

açıktır. Öyleyse sadece 2 ve 3 için çözümlüdür.<br />

Cauchy-Schwartz Eşitsizliği. a 1 , a 2 , · · · , a n ve b 1 , b 2 , · · · , b n reel sayıları<br />

için<br />

(a 1 b 1 + a 2 b 2 + · · · + a n b n ) 2 ≤ (a 2 1 + a 2 2 + · · · + a 2 n)(b 2 1 + b 2 2 + · · · + b 2 n)<br />

eşitsizliği vardır. Eşitlik durumu a i b j = a i b j , i, j = 1, · · · , n için vardır.<br />

(7) 0 ≤ θ < 2π için<br />

a · cos θ + b · sin θ<br />

ifadesinin alabileceği en büyük ve en küçük değerleri bulunuz.<br />

Çözüm. C.S. eşitsizliğinden<br />

(a cos θ + b sin θ) 2 ≤ (a 2 + b 2 )(cos 2 θ + sin 2 θ)<br />

3 Antalya Üniversitesi Mate<strong>mat</strong>ik Olimpiyatları, 2008


2.5. EŞİTSİZLİKLER I 43<br />

eşitsizliğinden,<br />

aralığı bulunur.<br />

− √ a 2 + b 2 ≤ a cos θ + b sin θ ≤ √ a 2 + b 2<br />

(8)(1978, USAMO 4 ) a, b, c, d, e reel sayıları için a + b + c + d + e = 8 ve<br />

a 2 + b 2 + c 2 + d 2 + e 2 = 16 ise, e’nin alabileceği en büyük değeri bulunuz.<br />

Çözüm. C.S. eşitsizliğinden,<br />

ise<br />

(a + b + c + d) 2 ≤ (1 + 1 + 1 + 1)(a 2 + b 2 + c 2 + d 2 )<br />

(8 − e) 2 ≤ 4(16 − e 2 )<br />

olacaktır. Buradan da, e(5e − 16) ≤ 0 ise 0 ≤ e ≤ 16/5 olacaktır. C.S. eşitsizliğinin,<br />

eşitlik duruu kullanılırsa a = b = c = d = 6/5 ve e max = 16/5 olacaktır.<br />

(9) a, b, c > 0 ve abc = 1 ise<br />

1<br />

a 3 (b + c) + 1<br />

b 3 (a + c) + 1<br />

c 3 (a + b) ≥ 3 2<br />

eşitizliğini kanıtlayınız.<br />

Çözüm. x = 1 a = bc, y = 1 b = ac, z = 1 c<br />

= ab alırsak, eşitsizliğin son hali<br />

olacaktır. Burada,<br />

x + y + z =<br />

x 2<br />

z + y +<br />

x<br />

√ z + y<br />

√ z + y +<br />

y2<br />

z + x +<br />

eşitliğine C.S.E. eşitsizliğini uygularsak,<br />

(x + y + z) 2 ≤ ( x2<br />

z + y +<br />

y2<br />

x + z +<br />

4 United States Of America Math Olympiads, 1978<br />

z2<br />

x + y ≥ 3 2<br />

y<br />

√ x + z<br />

√<br />

x + z +<br />

z<br />

√ x + y<br />

√ x + y<br />

z2<br />

)((z + y) + (x + z) + (y + x))<br />

x + y


44 BÖLÜM 2. KONULAR<br />

olacaktır. Son eşitsizliği ve A.O. − G.O. eşitsizliklerini kullanırsak,<br />

bulunur.<br />

x 2<br />

z + y +<br />

y2<br />

z + x +<br />

z2<br />

x + y ≥ x + y + z<br />

3<br />

≥ 3 2 3√ xyz = 3 2<br />

Yeniden Düzenleme (Permütasyon) Eşitsizliği. a 1 ≥ a 2 ≥ · · · ≥ a n ve<br />

b 1 ≥ b 2 ≥ · · · ≥ b n ise,<br />

a 1 b 1 +a 2 b 2 +· · ·+a n b n ≥ a 1 b r1 +a 2 b r2 +· · ·+a n b rn ≥ a 1 b n +a 2 b n−1 +· · ·+a n b 1<br />

eşitsizliği vardır. Burada (b r1 , b r2 , · · · , b rn ) dizilimi (b 1 , b 2 , · · · , b n ) diziliminin<br />

bir permütasyonudur.<br />

(10)(1978, IMO 5 ) c 1 , c 2 , · · · , c n farklı pozitif tamsayılardır. Buna göre,<br />

eşitsizliğini kanıtlayınız.<br />

c 1 + c 2<br />

2 2 + · · · + c n<br />

n 2 ≥ 1 + 1 2 + · · · + 1 n<br />

Çözüm. Varsayalım (a 1 , a 2 , · · · , a n ) dizilimi c i ’lerin artan sırayla dizilimi olsun.<br />

a i ’ler farklı pozitif tamsayılar olduğuna göre, a 1 ≥ 1, a 2 ≥ 2,· · · , a n ≥ n<br />

diyebiliriz. Burada,<br />

a 1 < a 2 < · · · < a n<br />

ve<br />

ise Y.D.E.’ne göre<br />

olacaktır.<br />

1 > 1 2 2 > 1 3 2 > · · · > 1 n 2<br />

c 1 + c 2<br />

2 2 + · · · + c n<br />

n 2 ≥ a 1 + a 2<br />

2 2 + · · · + a n<br />

n 2 ≥ 1 + 2 2 2 + · · · + n n 2<br />

(11) Örnek (9)’u Y.D.E. kullanarak yapınız.<br />

5 International Math Olympiads, 1978


2.5. EŞİTSİZLİKLER I 45<br />

Çözüm. x, y, z için tanımlamalarımız (9)’daki gibi olsun. Genelliği kaybetmeden<br />

x ≥ y ≥ z alalım. xyz = 1 ve x 2 ≥ y 2 ≥ z 2 olduğuna göre 1/(z + y) ≥<br />

1/(x + z) ≥ 1/(y + x) üçlüsüde ikinci dizilimimiz olsun. Bu noktada Y.D.E.’yi<br />

iki defa uygularsak,<br />

x 2<br />

z + y +<br />

x 2<br />

z + y +<br />

y2<br />

z + x +<br />

y2<br />

z + x +<br />

z2<br />

x + y<br />

z2<br />

x + y<br />

eşitsizliklerini taraf tarafa toplarsak,<br />

≥<br />

≥<br />

x 2<br />

x + y +<br />

x 2<br />

z + x +<br />

y2<br />

z + y +<br />

y2<br />

x + y +<br />

z2<br />

x + z<br />

z2<br />

z + y<br />

x 2<br />

z + y +<br />

y2<br />

z + x +<br />

eşitsizliğini elde ederiz.<br />

z2<br />

x + y ≥ 1 + x 2<br />

2 (y2 y + x + z2 + y 2<br />

y + z + z2 + x 2<br />

z + x )<br />

a 2 + b 2 ≥ a2 + b 2<br />

eşitsizliğini, sağ tarafın payları için kullandıktan sonra A.O.−G.O. eşitsizliğini<br />

kullanırsak,<br />

x 2<br />

z + y +<br />

y2<br />

z + x +<br />

z2<br />

x + y ≥ 1 2 (y + x<br />

2<br />

eşitsizliği elde edilir.<br />

+ z + y<br />

2<br />

2<br />

+ x + z ) = x + y + z<br />

2 2<br />

≥ 3 2 3√ xyz = 3 2<br />

Chebyshev Eşitsizliği. Eğer a 1 ≥ a 2 ≥ · · · ≥ a n ve b 1 ≥ b 2 ≥ · · · ≥ b n ise<br />

eşitsizliği vardır.<br />

a 1 b 1 + a 2 b 2 + · · · + a n b n ≥ (a 1 + · · · + a n )(b 1 + · · · + b n )<br />

n<br />

(12)(1974, USAMO 6 ) a, b, c > 0 ise<br />

a a b b c c ≥ (abc) (a+b+c)/3<br />

6 United States Of America Math Olympiads, 1974


46 BÖLÜM 2. KONULAR<br />

olduğunu kanıtlayınız.<br />

Çözüm. Üçlülerimizi (a, b, c) ve (log a, log b, log c) olarak seçersek,<br />

olacaktır.<br />

a log a + b log b + c log c ≥ (a + b + c)(log a + log b + log c) 1 3<br />

log a a b b c c ≥<br />

(a + b + c)<br />

log(abc)<br />

3<br />

log a a b b c c ≥ log(abc) (a+b+c)/3<br />

a a b b c c ≥ (abc) (a+b+c)/3<br />

(13) 0 ≤ a k < 1, k = 1, 2, 3, · · · , n ve S = a 1 + a 2 + · · · + a n ise<br />

eşitsizliğini kanıtlayınız.<br />

n∑<br />

k=1<br />

a k<br />

≥ nS<br />

1 − a k n − S<br />

Çözüm. Genelliği kaybetmeden a 1 ≥ a 2 ≥ · · · a n ≥ 0 alabiliriz. Buna göre,<br />

0 < 1 − a 1 ≤ 1 − a 2 ≤ · · · ≤ 1 − a n ve<br />

olacaktır. Chebysev eşitsizliğinden,<br />

a 1<br />

1 − a 1<br />

≥ a 2<br />

1 − a 2<br />

≥ · · · ≥ a n<br />

1 − a n<br />

S =<br />

≤<br />

a 1<br />

(1 − a 1 ) + a 2<br />

(1 − a 2 ) + · · · + a n<br />

(1 − a n )<br />

1 − a 1 1 − a 2 1 − a n 1 n∑ a k<br />

n∑<br />

(1 − a k ) = n − S n∑ a k<br />

olacaktır.<br />

n 1 − a k n 1 − a k<br />

k=1<br />

k=1<br />

k=1<br />

Mate<strong>mat</strong>ikte ve tabii ki istatistikte sıklıla ortalamalrı kullanmaya ihtiyaç<br />

duyarız. AO, GO, HO dışında kullandığımız Kuvvet Ortalaması ve Simetrik<br />

Ortalarda vardır. Aslında, bu iki ortalama AO ve GO’yıda özel birer durum<br />

olarak içerir.


2.5. EŞİTSİZLİKLER I 47<br />

Kuvvet Ortalaması Eşitsizliği. a 1 , a 2 , a 3 , · · · , a n > 0 ve s < t için<br />

M s = ( as 1 + as 2 + · · · + as n<br />

n<br />

) 1/s ≤ M t = ( at 1 + at 2 + · · · + at n<br />

) 1/t<br />

n<br />

eşitsizlikleri vardır. Eşitlik a 1 = a 2 = · · · = a n durumunda vardır.<br />

Not. Bu eşitsizlikte M 1 = A.O., M −1 = H.O. ve M 2 ise Karesel Ortalamadır.<br />

Karesel Orta istatistik ve fizikte kullanılır. Ayrıca, limitlerini alırsak, M +∞<br />

ifadesi MAX = max{a 1 , a 2 , · · · , a n } ve M 0 ifadesi Geometrik Orta ve de M −∞<br />

ifadesi MIN = min{a 1 , a 2 , · · · , a n } olacaktır. Dolayısıyla elimizde,<br />

eşitsizliği oluşacaktır.<br />

MAX ≥ KO ≥ AO ≥ GO ≥ HO ≥ MIN<br />

Maclaurin Simetrik Orta Eşitsizliği.a 1 , a 2 , · · · , a n > 0 için<br />

AO ≥ S 1 ≥ S 1/2<br />

2 ≥ · · · ≥ Sn<br />

1/n = GO<br />

olacaktır. Mesela, S j ifadesine n = 4 için bakalım<br />

olacaktır.<br />

S 1 = a 1 + a 2 + a 3 + a 4<br />

4<br />

S 2 = a 1a 2 + a 1 a 3 + a 1 a 4 + a 2 a 3 + a 2 a 4 + a 3 a 4<br />

6<br />

S 3 = a 1a 2 a 3 + a 1 a 2 a 4 + a 1 a 3 a 4 + a 2 a 3 a 4<br />

4<br />

S 4 = a 1 a 2 a 3 a 4<br />

(14) x, y, z pozitif sayılar olduğuna göre,<br />

√ √ √<br />

x 5 + y 5 + z 5 ≤ x 5 x 2 y<br />

yz + 2<br />

y5<br />

zx + z 2<br />

z5 xy<br />

eşitsizliğini kanıtlayınız.


48 BÖLÜM 2. KONULAR<br />

Çözüm. Varsayalım a = √ x, b = √ y, c = √ z olsun. Buna göre eşitsizliğimiz,<br />

a 10 + b 10 + c 10 ≤ a13 + b 13 + c 13<br />

olacaktır. Buna göre, KOE’yi kullanırsak<br />

a 13 + b 13 + c 13 = 3M 13<br />

13 = 3M 10<br />

13 M 3 13 ≥ 3M 10<br />

10 M 3 0 = (a 10 + b 10 + c 10 )abc<br />

olacaktır.<br />

(15) a, b, c > 0 ise,<br />

eşitsizliğini kanıtlayınız.<br />

Çözüm. Eşitsizliği düzenlersek,<br />

abc<br />

1<br />

a + 1 b + 1 c ≤ a8 + b 8 + c 8<br />

a 3 b 3 c 3<br />

a 8 + b 8 + c 8 ≥ a 3 b 3 c 3 ( 1 a + 1 b + 1 c ) = (abc)2 (bc + ac + ab)<br />

olacaktır. KOE ve SOE’yi kullanırsak,<br />

olacaktır.<br />

a 8 + b 8 + c 8 = 3M8 8 ≥ 3M1 8 = 3S1 8 = 3S1S 6 1<br />

2<br />

≥ (S 1/3<br />

3 ) 6 3(S 1/2<br />

2 ) 2 = (abc) 2 (bc + ac + ab)<br />

(16) a 1 , a 2 , · · · , a n ≥ 0 ve (1 + a 1 )(1 + a 2 ) · · · (1 + a n ) = 2 n ise,<br />

olduğunu gösteriniz.<br />

Çözüm. SO eşitsizliğinden,<br />

a 1 · a 2 · · · · · a n ≤ 1<br />

2 n = (1 + a 1 )(1 + a 2 ) · · · (1 + a n )<br />

( ( )<br />

n n<br />

= 1 + nS 1 + S 2 + · · · + S n−1 + S n<br />

2)<br />

n − 1<br />

( ( )<br />

n n<br />

≥ 1 + nSn 1/n + Sn 2)<br />

2/n + · · · + S n−1<br />

n<br />

n + S n = (1 + Sn<br />

1/n<br />

n − 1<br />

ise 2 ≥ 1 + S 1/n<br />

n<br />

ve 1 ≥ S n = a 1 · a 2 · · · · · a n olur.<br />

) n


2.5. EŞİTSİZLİKLER I 49<br />

2.5.1 Çalışma Soruları<br />

1. Konu anlatımında verilen tüm örnekleri çözümlerine bakmadan yapınız.<br />

2. x 1 , x 2 , · · · , x n > 0 için<br />

eşitsizliğini kanıtlayınız.<br />

x 2 1<br />

x 2<br />

+ x2 2<br />

x 3<br />

+ · · · + x2 n<br />

x 1<br />

≥ x 1 + x 2 + · · · + x n<br />

3. 0 < a, b, c < 1 ve a + b + c = 2 ise<br />

eşitsizliğini kanıtlayınız.<br />

8(1 − a)(1 − b)(1 − c) ≤ abc<br />

4. Eğer a, b, c, d > 0 ve c 2 + d 2 = (a 2 + b 2 ) 3 ise<br />

eşitsizliğini kanıtlayınız.<br />

a 3<br />

c + b3 d ≥ 1<br />

5. a 1 , a 2 , · · · , a n > 0 ve a 1 + a 2 + · · · + a n = 1 ise<br />

(a 1 + 1 a 1<br />

) 2 + (a 2 + 1 a 2<br />

) 2 + · · · + (a n + 1<br />

a n<br />

) 2<br />

ifadesinin en küçük değerini bulunuz.<br />

6. Eğer a, b, c, d > 0 ve S = a 2 + b 2 + c 2 + d 2 ise<br />

a 3 + b 3 + c 3<br />

a + b + c<br />

eşitsizliğini kanıtlayınız.<br />

+ a3 + b 3 + d 3<br />

a + b + d + a3 + c 3 + d 3<br />

a + c + d + b3 + c 3 + d 3<br />

b + c + d<br />

7. Eğer x 1 , x 2 , · · · , x n > 0 ve x 1 + x 2 + · · · + x n = 1 ise,<br />

n∑<br />

k=1<br />

eşitsizliğini kanıtlayınız.<br />

x k<br />

√ 1 − xk<br />

≥<br />

1<br />

√ n − 1<br />

n∑<br />

k=1<br />

√<br />

xk<br />

≥ S


50 BÖLÜM 2. KONULAR<br />

8. a, b, c bir üçgenin kenar uzunlukları olduğuna göre,<br />

eşitsizliğini kanıtlayınız.<br />

a 2 b(a − b) + b 2 c(b − c) + c 2 a(c − a) ≥ 0<br />

2.6 Eş˙ıts˙ızl˙ıkler II<br />

Bazı fonksiyon eşitsizliklerini kanıtını yaparken, fonksiyonunun belli aralıklardaki<br />

şeklide önemlidir. Bu ders notumuzda ele aldığımız eşitsizliklerin<br />

çözümleride bu temel prensiplere uyularak yapılmıştır.<br />

Isınma<br />

Tanım [Konveks vs. Konkav]. I aralığı üzerinde süreli bir f fonksiyonu,<br />

f( x 1 + x 2<br />

) ≤ f(x 1) + f(x 2 )<br />

2<br />

2<br />

x 1 , x 2 ∈ I<br />

eşitsizliğini sağlıyorsa f fonksiyonuna konveks denir. Eğer I aralığı üzerinde<br />

f fonksiyonu konveks ve eşitlik hali x 1 = x 2 oluyorsa, f tam konveks olur. I<br />

aralığında −f konveks ise f fonksiyonu konkav olur. Bu durumda,<br />

f( x 1 + x 2<br />

) ≥ f(x 1) + f(x 2 )<br />

2<br />

2<br />

x 1 , x 2 ∈ I<br />

olacaktır. Benzer biçimde eğer −f tam konveks ise f fonksiyonuda konveks<br />

olur.<br />

Tanım [İkinci Türev Testi]. Eğer I = (a, b) aralığında f ′′ (x) ≥ 0 oluyorsa,<br />

f fonksiyonu konvekstir. Eğer f ′′ (x) > 0 oluyorsa, f fonksiyonu tam konveks<br />

olur. Konkav ve tam konkav içinde tanım benzer biçimdedir. Sadece eşitsizlik<br />

yön değiştirir. Bir fonksiyonun konveksiliğini göstermek için sınır noktalarını<br />

içeren bir aralıkta ve bu aralıkta sürekli <strong>olm</strong>ası ile ikinci türev testinin negatif<br />

<strong>olm</strong>aması yeterlidir. İkinci türev testini kullanarak, aşağıda verilen fonksiyoların<br />

tam konvex olduğunu söyleyebiliriz.<br />

x p ∈ [0, ∞) , p > 1 yada x p ∈ (0, ∞), p < 0<br />

a x ∈ (−∞, ∞), a > 1 yada tan x ∈ [0, π/2)


2.6. EŞİTSİZLİKLER II 51<br />

Benzer biçimde aşağıda verilen fonksiyonlarda tam konkavdır.<br />

x p ∈ [0, ∞) , 0 < p < 1 yada log a x ∈ (0, ∞), a > 1<br />

cos x ∈ [−π/2, π/2), yada sin x ∈ [0, π/2π)<br />

Bu noktada, konveksliği ve konkavlığı eşitsizlik sorularında kullanabileceğimiz<br />

en güzel yer sanırız Jensen Eşitsizliği’dir.<br />

Mevzu<br />

Tanım [Jensen Eşitsizliği]. f fonksiyonu I üzerinde konveks ve x 1 , x 2 , · · · , x n ∈<br />

I ise<br />

( )<br />

x1 + x 2 + x 3<br />

f<br />

≤ f(x 1) + f(x 2 ) + · · · + f(x n )<br />

n<br />

n<br />

olacaktır. Burada eşitlik durumu ancak ve ancak x 1 = x 2 = · · · = x n eşitliğinde<br />

olur.<br />

Tanım [Genelleştirilmiş Jensen Eşitsizliği]. f konveks ve I aralığında<br />

sürekli <strong>olm</strong>ak üzere, x 1 , x 2 , · · · , x n ∈ I ve 0 < t 1 , t 2 , · · · , t n < 1, t 1 + t 2 + · · · +<br />

t n = 1 ise,<br />

f(t 1 x 1 + t 2 x 2 + · · · + t n x n ) ≤ t 1 f(x 1 ) + t 2 f(x 2 ) + · · · + t n f(x n )<br />

Şekil 2.1: Aralıkta Bir Konveks Fonksiyon


52 BÖLÜM 2. KONULAR<br />

olacaktır. Konkav fonksiyonlarda ise eşitsizlik yön değiştirir.<br />

Egzersiz<br />

[1.] Bir ABC üçgeninde,<br />

sin A + sin B + sin C ≤ 3√ 3<br />

2<br />

eşitsizliğini gösteriniz. Eşitlik durumu hangi durumda ortaya çıkar, açıklayınız.<br />

Çözüm. f(x) = sin x fonksiyonu [0, π] arasında konkavdır. Öyleyse,<br />

( ) ( )<br />

A + B + C<br />

A + B + C<br />

sin A+sin B+sin C = f(A)+f(B)+f(C) ≤ 3f<br />

= 3 sin<br />

= 3√ 3<br />

3<br />

2<br />

olacaktır. Eşitlik durumu ancak ve ancak A = B = C = π/3 yani ABC bir<br />

eşkenar üçgense gerçekleşir.<br />

[2.] a, b, c > 0 ve a + b + c = 1 ise,<br />

ifadesinin en küçük değerini bulunuz.<br />

(a + 1 a )10 + (b + 1 b )10 + (c + 1 c )10<br />

Çözüm. 0 < a, b, c < 1 olarak zaten verilmiş. f(x) = (x+ 1 x )10 ise f fonksiyonu<br />

I = (0, 1) aralığında konveksdir. Çünkü,<br />

Öyleyse JE’den<br />

f ′′ (x) = 90(a + 1 x )8 (x − 1 x 2 )2 + 10(x + 1 x )9 ( 2 x 3 ) > 0 olacaktır.<br />

f(a)+f(b)+f(c) = (a+ 1 a )10 +(b+ 1 b )10 +(c+ 1 c )10 ≥ 3f( a + b + c ) = 3f( 1 3<br />

3 ) = 1010<br />

3 9<br />

olarak bulunur.<br />

Çözüm.(Alternatif Yöntem) Soruda verilen eşitsizliği çözmenin bir diğer<br />

yöntemide Chebychev Eşitsizliğini kullanmak olabilirdi. Bu yöntemi size bırakıyoruz.<br />

7<br />

7 Chebychev Eşitsizliği ile alakalı olarak www.sbelian wordpress.com adresinden Yeniden<br />

Düzenleme Eşitsizliği Ders Notları’nı indirebilirsiniz.


2.6. EŞİTSİZLİKLER II 53<br />

[3.] Aritmetik Orta - Geometrik Orta eşitsizliği JE kullanarak göstermeye çalışalım.<br />

Buna göre, eğer a 1 , a 2 , a 3 , · · · , a n ise f(x) = log x’de (0, ∞) aralığında<br />

konkav olduğuna göre,<br />

log( a 1 + a 2 + · · · + a n<br />

) ≥ log a 1 + log a 2 + · · · + log a n<br />

n<br />

n<br />

ise istenen eşitsizlik kanıtlanmış olur.<br />

= log( n√ a 1 a 2 · · · a n )<br />

[4.](Hölder) p, q > 1, 1 p + 1 q = 1 ve a 1, a 2 , · · · , a n b 1 , b 2 , · · · , b n reel sayılarsa,<br />

∣ ( n∑ ∣∣∣∣ n∑<br />

) 1/p ( n∑<br />

) 1/q<br />

a<br />

∣ i b i ≤ |a i | p |b i | q<br />

i=1<br />

i=1<br />

i=1<br />

eşitsizliğini kanıtlayınız.<br />

Çözüm. Varsayalım<br />

A =<br />

n∑<br />

|a i | p ve B =<br />

i=1<br />

n∑<br />

|b i | q<br />

i=1<br />

olsun. Eğer A veya B sıfır ise, ya tüm a i ’ler yada tüm b i ’ler sıfırdır. Bu da zaten<br />

eşitsizliğin iki tarafınıda sıfır yapar. Buna göre biz A ≠ 0 ve B ≠ 0 durumunu<br />

inceleyelim. Varsayalım t 1 = 1 p ve t 2 = 1 q<br />

olsun. Öyleyse, 0 < t 1 , t 2 < 1 ve<br />

t 1 + t 2 = 1 olacaktır. Eğer<br />

x i = |a i| p<br />

A<br />

ve y i = |b i| q<br />

B<br />

ise<br />

n∑<br />

x i = 1 ve<br />

i=1<br />

n∑<br />

y i = 1<br />

olur. f(x) = e x fonksiyonu (−∞, +∞) aralığında konveks olduğundan Genelleştirilmiş<br />

Jensen Eşitsizliğini kullanabiliriz. Buna göre,<br />

x 1/p<br />

i<br />

· y 1/q<br />

i<br />

= f(t 1 ln x i + t 2 ln y i ) ≤ t 1 f(ln x 1 ) + t 2 f(ln y i ) = x i<br />

p + y i<br />

q<br />

i=1


54 BÖLÜM 2. KONULAR<br />

olacaktır. Buna göre,<br />

n∑<br />

i=1<br />

|a i ||b i |<br />

A 1/p B 1/q ≤ 1 p<br />

n∑<br />

x i + 1 q<br />

i=1<br />

n∑<br />

y i = 1<br />

i=1<br />

olacaktır. Bundan dolayıda,<br />

∣ n∑ ∣∣∣∣ n∑<br />

(∑ a<br />

∣ i b i ≤ |a i ||b i | ≤ A 1/p B 1/q = |ai | p) 1/p (∑<br />

|bi | q) 1/q<br />

bulunur<br />

i=1<br />

i=1<br />

Şimdi de, Jensen eşitsiliğinin bir başka uygulamasına geçelim.<br />

Tanım (Majorization 8 ). Eğer x 1 , · · · , x n ve y 1 , · · · , y n aşağıdaki şartları<br />

sağlıyorsa, yani<br />

x 1 ≥ x 2 ≥ · · · ≥ x n ,<br />

y 1 ≥ y 2 ≥ · · · ≥ y n<br />

x 1 ≥ y 1 ,<br />

ve<br />

x 1 + x 2 ≥ y 1 + y 2 , · · · , x 1 + x 2 + · · · + x n−1 ≥ y 1 + y 2 + · · · + y n−1<br />

x 1 + x 2 + · · · + x n = y 1 + y 2 + · · · + y n<br />

ise (x 1 , x 2 , · · · , x n ) majorize (y 1 , y 2 , · · · , y n ) denir ve<br />

ile gösterilir.<br />

(x 1 , x 2 , · · · , x n ) ≻ y 1 , y 2 , · · · , y n )<br />

Tanım (Majorization Eşitsizliği). I = [a, b] aralığında f fonksiyonu konveks<br />

ve<br />

(x 1 , x 2 , · · · , x n ) ≻ y 1 , y 2 , · · · , y n ), x i , y i ∈ I<br />

ise<br />

f(x 1 ) + f(x 2 ) + · · · + f(x n ) ≥ f(y 1 ) + f(y 2 ) + · · · + f(y n )<br />

olur. Yanlız ve yanlız x i = y i durumu için eşitlik vardır. Konkav fonksiyonlar<br />

içinse eşitsizlik yön değiştirir.<br />

8 http://en.wikipedia.org/wiki/Majorization


2.6. EŞİTSİZLİKLER II 55<br />

[5.] Dar açılı bir ABC üçgeni için,<br />

eşitsizliğini kanıtlayınız.<br />

1 ≤ cos A + cos B + cos C ≤ 3 2<br />

Çözüm. Genelliği kaybetmeden, varsayalım A ≥ B ≥ C olsun. Buna göre<br />

A ≥ π/3 ve C ≤ π/3 olacaktır. π/2 ≥ A ≥ π/3, π ≥ A + B = (−π − C) ≥ 2π 3<br />

olacağından<br />

( π 2 , π 2 , 0) ≻ (A, B, C) ≻ (π 3 , π 3 , π 3 )<br />

alabiliriz. f(x) = cos x fonksiyonu [0, π/2] aralığında konkav olduğuna göre,<br />

majorization teoreminden,<br />

1 = f( π 2 ) + f(π 2 ) + f(0) ≤ f(A) + f(B) + f(C) ≤ f(π 3 ) + f(π 3 ) + f(π 3 )<br />

ise soruda göstermemiz istenen,<br />

elde edilir.<br />

[6.] Eğer x 1 ≥ x 2 ≥ · · · ≥ x n ise<br />

1 ≤ cos A + cos B + cos C ≤ 3 2<br />

(x 1 , x 2 , · · · , x n ) ≻ (x, x, x, · · · , x)<br />

durumu vardır. Burada x değeri, x 1 , x 2 , · · · , x n değerlerinin aritmetik ortasıdır.(Bunu<br />

Majorization üzerine uygularsak Jensen Eşitsizliğini elde ederiz.)<br />

Buna göre, k = 1, 2, · · · , n − 1 için x 1 + x 2 + · · · + x k ≥ kx durumunu göstermemiz<br />

yeterli olacaktır. Buna göre,<br />

(n − k)(x 1 + x 2 + · · · + x k ) ≥ (n − k)kx k ≥ k(n − k)x k+1 ≥ k(x k+1 + · · · + x n )<br />

olacağından<br />

(n − k)(x 1 + · · · + x k ) ≥ k(x k+1 + · · · + x n )<br />

bulunur. Bu eşitsizliğin iki tarafınada k(x 1 + · · · + x k ) eklersek<br />

n(x 1 + · · · + x n ) ≥ k(x 1 + · · · + x n ) = knx


56 BÖLÜM 2. KONULAR<br />

olacağından<br />

olacaktır.<br />

x 1 + x 2 + · · · + x k ≥ kx<br />

[7.] −1 ≤ a, b, c ≤ 1 ve a + b + c = 1/2 ise a 12 + b 12 + c 12 en fazla kaç olur<br />

Çözüm. [−1, 1] aralığında f(x) = x 12 fonksiyonu konvekstir. Öyleki, f ′′ (x) =<br />

132x 10 ≥ 0 olacaktır. Eğer, 1 ≥ a ≥ b ≥ c ≥ −1 ve a + b + c = −1/2 ise major<br />

üçlülerimizi<br />

(1, − 1 , −1) ≻ (a, b, c)<br />

2<br />

olarak seçebiliriz. Çünkü, 1 ≥ a ve 1 2 = 1 − 1 2 ≥ −c − 1 2<br />

= a + b olacaktır. Buna<br />

göre, majorization eşitsizliğinden<br />

a 12 + b 12 + c 12 = f(a) + f(b) + f(c) ≤ f(1) + f(2) + f(−1) = 2 + 1<br />

2 12<br />

olacaktır. Zaten, a = 1, b = − 1 2 ve c = − 1 2<br />

[8.](1999,IMO 9 ) n ≥ 2 bir tamsayıdır. Buna göre,<br />

(a.)<br />

∑<br />

1≤i


2.6. EŞİTSİZLİKLER II 57<br />

olacaktır.<br />

n > 2 durumu için varsayalım<br />

x i<br />

a i =<br />

ve a 1 + a 2 + · · · + a n = 1 olsun.<br />

x 1 + x 2 + · · · + x n<br />

Buna göre, a i = [0, 1] olacaktır. Eğer, a i cinsinden yazarsak, ispatlanacak<br />

eşitsizlik<br />

∑<br />

a i a j (a 2 i + a 2 j) ≤ C<br />

1≤i 0<br />

olacaktır. Eşitsizliğimiz a i ’lere göre simetrik olduğundan, a 1 ≥ a 2 ≥ · · · ≥ a n<br />

alabiliriz.<br />

Buan göre, eğer a 1 ≤ 1/2 ise,<br />

( 1 2 , 1 2 , 0, 0, · · · , 0) ≻ (a 1, a 2 , · · · , a n )<br />

olacağından majorization eşitsizliğinden<br />

f(a 1 ) + f(a 2 ) + · · · + f(a n ) ≤ f( 1 2 ) + f(1 2 ) + f(0) + · · · + f(0) = 1 8<br />

olacaktır. Eğer, a 1 > 1/2 ise 1 − a 1 , a 2 , · · · , a n ∈ [0, 1/2) olacaktır.<br />

(1 − a 1 , 0, 0, · · · , 0) ≻ (a 2 , a 3 , · · · , a n )<br />

olduğundan majorization eşitsizliği ve n = 2 durumunu göz önünde bulundurursak,<br />

f(a 1 ) + f(a 2 ) + · · · + f(a n ) ≤ f(a 1 ) + f(1 − a 1 ) + f(0) + f(0) + · · · + f(0)<br />

= f(a 1 ) + f(1 − a 1 ) ≤ 1 8 olacaktır.<br />

Buna göre, eşitlik durumu ancak ve ancak iki değişken eşit ve geri kalan (n−2)<br />

değişken 0 ise vardır.


58 BÖLÜM 2. KONULAR<br />

2.6.1 Çalışma Soruları<br />

1. Jensen Eşitisizliğini kullanarak a a b b c c ≥ (abc) (a+b+c)/3 eşitsizliğini kanıtlayınız.(a, b, c<br />

pozitif reel sayılar.)<br />

2. x 1 , · · · , x n ∈ [0, 1] ve a 1 , · · · , a n > 0 <strong>olm</strong>ak üzere, a 1 +· · ·+a n = 1 olarak<br />

veriliyor. Buna göre,<br />

n∑<br />

i=1<br />

a i<br />

1 + x i<br />

≤<br />

1<br />

1 + x a 1<br />

1 · · · xa 1<br />

1<br />

eşitsizliğini kanıttlayınız. Eşitlik durumu hangi şartlar altında gerçekleşir<br />

3. Eğer a, b, c, d > 0 ve c 2 + d 2 = (a 2 + b 2 ) 3 ise<br />

a 3<br />

c + b3 d ≥ 1<br />

eşitsizliğini Hölder Eşitsizliği kullanarak kanıtlayınız.<br />

4. P noktası ABC üçgeni içersinde<br />

m(P AB) = m(P BC) = m(P CA) = α<br />

eşitliğini sağlayan bir nokta ise α açısının π/6 olduğunu kanıtlayınız.<br />

5. Dar açılı bir ABC üçgeninin iki açısı π/6’dan küçük veya eşittir. Buna<br />

göre,<br />

eşitsizliğini kanıtlayınız.<br />

sin A 2 sin B 2 sin C 2 ≥ sin π 4 sin π 6 sin π 12<br />

6. x, y, z > 1, xyz = 4096 ve max(x, y, z) < 32 olarak veriliyor. Buna göre,<br />

x + y + z toplamının en büyük ve en küçük değerlerini bulunuz.<br />

7. Eğer a, b ≥ 0 ise,<br />

3<br />

√a + 3√ a +<br />

√b 3 + 3√ b ≤<br />

√a 3 + 3√ √<br />

b + 3 b + 3√ a<br />

eşitsizliğini kanıtlayınız.


2.7. İNDİRGEMELİ DİZİLER 59<br />

2.7 İnd˙ırgemel˙ı D˙ız˙ıler<br />

2.7.1 Birinci Dereceden İndirgemeler<br />

Herhalde <strong>mat</strong>e<strong>mat</strong>ik olimpiyatları sınavlarına hazırlanıpta Fibonacci Sayıları’nı<br />

bilmeyen yoktur. Zaten Fibonacci sayılarıda f 0 = 0, f 1 = 1 <strong>olm</strong>ak<br />

üzere<br />

f n+1 = f n + f n−1 , n ≥ 1<br />

yinelemesi ile tanımlanır. Bir yinelemenin derecesi ise en büyük ve en küçük<br />

alt terimlerin farkına eşittir. Mesela,<br />

birinci derecedendir, ve<br />

u n+2 − u n+1 = 2<br />

u n+4 + 9u 2 n = n 5<br />

yinelemesi ise dördüncü derecedendir. Eğer bir yinelemede tüm ifadelerin üstleri<br />

bir ise bu yinelemeye Doğrusal Yineleme denir. Mesela,<br />

bir doğrusal yinelemedir. Ancak<br />

u n+2 − u n+1 = 2<br />

x 2 n + nx n−1 = 1 ve x n + 2 xn−1 = 3<br />

yinelemeleri lineer değillerdir. Eğer bir yinelemenin tüm terimlerinin kuvvetleri<br />

aynı kuvvettense, bu yinelemeye Homojen’dir denir. Mesela,<br />

yinelemesi homojendir. Ancak,<br />

x m+3 + 8x m+2 − 9x m = 0<br />

x m+3 + 8x m+2 − 9x m = m 2 − 3<br />

yinelemesi homojen değildir. Bir yinelemin sadece indis değişkenine göre tanımlanan<br />

denkleme ise o yinelemenin Kapalı Form’u denir. Kapalı form sayesinde<br />

rahatlıkla yinelemenin her hangi bir terimini bulabiliriz. Biz genel<br />

manada bu ders notunda ilk olarak,<br />

x n = ax n−1 + f(n), a ≠ 1


60 BÖLÜM 2. KONULAR<br />

formundaki (f burada bir polinomdur) yinelemelerin çözümleriyle ilgileneceğiz.<br />

İlgilenirken de aşağıda verdiğimiz basamakları takip edeceğiz.<br />

1. Önce x n = ax n−1 formunda verilen yinelemenin indislerini üste alarak<br />

yani, karakteristik denklem oluşturarak, x n = ax n−1 denklemini elde ederiz.<br />

Sadeleştirmeler yaptığımızda x = a olacaktır. Buna göre, x n = ax n−1<br />

homojen formundaki yinelememiz bize x n = Aa n denklemini verecektir.<br />

Burada A bir sabit sayıdır.<br />

2. Daha sonra bulunan x n = Aa n + g(n) formu test edilir. Burada g, f ile<br />

aynı dereceden bir polinomdur.<br />

Örnek 1. x 0 = 7 ve x n = 2x n−1 , n ≥ 1 ise x n kapalı formunu bulunuz.<br />

Çözüm. Alt indisleri üs olarak yazarsak karakteristik denklemimizi x n =<br />

2x n−1 olacaktır. Sadeleştirme yaparsak, x = 2 olacaktır. Buna göre bizim,<br />

x n = A2 n formunda çözüm yapmamız gerekmektedir. Burada, 7 = x 0 = A2 0<br />

ise A = 7 olacağından, kapalı form x n = 7(2) n olarak bulunur.<br />

Örnek 2. x 0 = 7 ve x n = 2x n−1 + 1, n ≥ 1 formunda verilen yinelemenin<br />

kapalı formunu bulunuz.<br />

Çözüm. Karakteristik denklemi yazıldığında x n = 2x n−1 veya x = 2 elde<br />

edilir. Buradan çözümlerden birinin x n = A(2) n olduğu açıktır. Ancak yinelemenin<br />

bir parçasıda f(n) = 1 polinomu olduğuna göre, kapalı denklemin x n =<br />

A2 n +B formunda <strong>olm</strong>ası gerekmektedir. Buradan, 7 = x 0 = A2 0 +B = A+B<br />

ve 15 = x 1 = 2A + B olduğuna göre bu iki denklemin çözümünden A = 8 ve<br />

B = −1 olacaktır. Öyleyse, soruda istenen kapalı form,<br />

olacaktır.<br />

x n = 8(2 n ) − 1 = 2 n+3 − 1<br />

Not. Örnek 2.’nin çözümünde dikkat edilirse oluşturula karakteristik denklem<br />

iki parçadan oluşuyor. Bunlardan ilki zaten alt indislerin üs olarak yazılmasıyla<br />

elde edilirken, ikincisi bir polinom ve bu polinomun derecesi yineleme içersinde<br />

ki polinomun derecesi ile aynı. Bundan sonraki çözümlerde de polinom seçimi<br />

benzer şekilde olacaktır.<br />

Örnek 3. x 0 = 2 ve x n = 9x n−1 − 56n + 63 ise x n kapalı formunu bulunuz.


2.7. İNDİRGEMELİ DİZİLER 61<br />

Çözüm. Karakterisitk denklem yazıldığında karakterisitik denklemimiz x n =<br />

9x n−1 veya x = 9 olacaktır. Buna göre, kapalı formun bir kısmı x n = A(9) n<br />

formunda olacaktır. Ancak, soruda verilen yinelemenin ikinci kısmını f(n) =<br />

−56n + 63 polinomu oluşturduğundan, çözüm olarak kullanacağımız polinom<br />

g(n) = Bn + C olacaktır. Buna göre, yinelememizin kapalı formu<br />

x n = A9 n + Bn + C<br />

olacaktır. x 0 , x 1 , x 2 için çözümlere bakıldığında,<br />

2 = A + C<br />

25 = 9A + B + C<br />

176 = 81A + 2B + C<br />

eşitlikleri için A = 2, B = 7 ve C = 0 olacaktır. Buna göre soruda istenen<br />

kapalı form, yada genel çözüm<br />

x n = 2(9 n ) + 7n<br />

olarak bulunur.<br />

Örnek 4. x 0 = 1 ve x n = 3x n−1 −2n 2 + 6n −3 ise x n kapalı formunu bulunuz.<br />

Çözüm. Yinelemenin karakterisitk denklemini yazdıktan sonra kolaylıkla x n =<br />

A(3) n indirgemesini elde edebiliriz. Ancak yinelememizin bir parçasıda f(n) =<br />

−2n 2 + 6n − 3 şeklinde ki ikinci dereceen bir polinom olduğundan özel çözümümüzde<br />

kullanacağımız g(n) polinomu Bn 2 + Cn + D <strong>olm</strong>alıdır. Buna göre,<br />

yinelememizin indirgenmiş hali,<br />

x n = A3 n + Bn 2 + Cn + D<br />

şekline olacaktır. Eğer bilinen x i , i = 0, 1, 2, 3 için katsayaları bulmaya çalışırsak,<br />

1 = A + D,<br />

4 = 3A + B + C + D,<br />

13 = 9A + 4B + 2C + D,<br />

36 = 27A + 9B + 3C + D


62 BÖLÜM 2. KONULAR<br />

denklemlerini elde ederiz. Buradan da, A = B = 1, C = D = 0 olacağından,<br />

istenen kapalı form<br />

x n = 3 n + n 2<br />

olacaktır.<br />

Örnek 5. x 0 = 2 ve x n = 2x n−1 + 3 n−1 ise kapalı formu bulunuz.<br />

Çözüm. Eğer gerekli işlemleri yaparsak, genel formun<br />

x n = A2 n + B3 n<br />

denklemi elde edilir. Burada, x 0 = 2 ve x 1 = 2(2) + 3 0 = 5 denklemlerinden,<br />

2 = A + B<br />

7 = 2A + 3B<br />

eşitliklerini elde edilir. Buna göre, A = 1 ve B = 1 olacağından istenen kapalı<br />

form<br />

x n = 2 n + 3 n<br />

olacaktır.<br />

Örnek 6. x 0 = 7 ve x n = x n−1 + n, n ≥ 1 ise x n için kapalı formu bulunuz.<br />

Çözüm. Şimdi bu çözümü siz yapmaya çalışın. Kapalı formu,<br />

olarak bulmanız gerekmektedir.<br />

x n = 7 +<br />

n(n + 1)<br />

2<br />

Şimdiye kadar çözdüklerimizde genel olarak, lineer yinelemeler hakimdi. Şimdiki<br />

örneğimizde de lineer <strong>olm</strong>ayan ancak lineerleştirilebilir, birinci drecen yinelemeler<br />

den birini çözeceğiz.<br />

Örnek 6. u 0 = 3 ve u 2 n+1 = u n, n ≥ 1 ise yinelemin kapalı formunu bulunuz.<br />

Çözüm. Varsayalım, v n = log u n olsun. Buna göre,<br />

v n = log u n = log u 1/2<br />

n−1 = 1 2 log u n−1 = v n − 1<br />

2


2.7. İNDİRGEMELİ DİZİLER 63<br />

olacaktır. Burada,<br />

olduğundan,<br />

olacaktır. Buradan,<br />

v n = v n−1<br />

2<br />

v n = v 0<br />

2 n<br />

log u n = log u 0<br />

2 n<br />

olduğundan istenilen kapalı form,<br />

olarak bulunur.<br />

u n = 3 1/2n<br />

2.7.2 İkinci Dereceden İndirgemeler<br />

Bir evvelki konumuzda birici dereceden yinelemeleri ele almıştık. Öyleki,<br />

her bir terim kendisinden bir önceki terime bağımlı olarak veriliyordu. Şimdi<br />

vereceğimiz formda ise durum artık biraz daha farklı. Öyleki artık karşılacağımız<br />

yinelemeler<br />

x n = ax n−1 + bx n−2<br />

şeklinde olacaktır. Bu tür yinelemelerin çözümleri içinde takip etmemiz gerekn<br />

bazı çözüm basamakları vardır. Buna göre,<br />

1. Önce alt indisleri üs olarak alıp karakteristik denklemi x n = ax n−1 +<br />

bx n−2 olduğundan kökleri r 1 ve r 2 olan x 2 − ax − b = 0 olarak bulunur.<br />

2. Eğer kökler birbirinden farklı ise genel form<br />

şeklinde olacaktır.<br />

3. Eğer kökler aynı ise genel form<br />

şeklinde olacaktır.<br />

x n = A(r 1 ) n + B(r 2 ) n<br />

x n = A(r 1 ) n + Bn(r 1 ) n


64 BÖLÜM 2. KONULAR<br />

Örnek 7. x 0 = 1, x 1 = −1 ve x n+2 + 5x n+1 + 6x n = 0 yinelemesinin kapalı<br />

formunu bulunuz.<br />

Çözüm. Soruda verilen yinelemenin karakteristik denklemi<br />

x 2 + 5x + 6 = (x + 3)(x + 2) = 0<br />

olarak elde edilir.Buna göre kapalı formumuz<br />

x n = A(−2) n + B(−3) n<br />

olacaktır. Buradan da, A = 2 ve B = −1 olacağından soruda istenilen kapalı<br />

form,<br />

x n = 2(−2) n − (−3) n<br />

olacaktır.<br />

Örnek 8. Fibonacci yinelemesi için kapalı formu, f 0 = 0, f 1 = 1 ve f n =<br />

f n−1 + f n−2 bilgilerini kullanarak bulunuz.<br />

Çözüm. Karakterisitk denklemimiz f 2 − f − 1 = 0 olacağından kapalı formumuz<br />

(<br />

1 + √ ) n (<br />

5 1 − √ ) n<br />

5<br />

f n = A<br />

+ B<br />

2<br />

2<br />

şeklinde olacaktır. Başlangıç değerleri kullanıldığında,<br />

1 = A<br />

(<br />

1 + √ ) n (<br />

5<br />

+ B<br />

2<br />

1 − √ 5<br />

2<br />

0 = A + B<br />

) n<br />

= 1 √<br />

5<br />

2 (A + B) +<br />

√<br />

5<br />

2 (A − B) = (A − B)<br />

2<br />

olacağından<br />

A = 1 √<br />

5<br />

, B = − 1 √<br />

5<br />

olacaktır. Sonuç olarakta Cauchy-Binet Formülü olarakta bilinen<br />

( ) n ( ) n<br />

f n = √ 1<br />

+ √ 1<br />

5 5<br />

kapalı form bulunacaktır.<br />

1 + √ 5<br />

2<br />

1 − √ 5<br />

2


2.7. İNDİRGEMELİ DİZİLER 65<br />

Örnek 8.(Tübitak Deformesi 10 ) x 0 = 1, x 1 = 4, x n = 4x n−1 − 4x n−2 ise<br />

kapalı formu bulunuz.<br />

Çözüm. Karakteristik denklemimiz x 2 − 4x + 4 = (x − 2) 2 = 0 olacaktır.<br />

Burada köklerin birbirine eşit olduğu açıktır. Buna göre kapalı formumuz,<br />

x n = A2 n + Bn2 n<br />

formunda olacaktır. Eğer başlangıç değerlerini kullanırsak,<br />

1 = A<br />

4 = 2A + 2B<br />

A = 1 ve B = 1 olarak bulunur. Buna göre, istenilen kapalı form<br />

olacaktır.<br />

x n = 2 n + n2 n<br />

2.7.3 Alıştırmalar<br />

Aşağıdaki sorulardan (1 − 5) için kapalı formları bulunuz.<br />

1. x 0 = 3, x n = x n−1+4<br />

3<br />

2. x 0 = 1, x n = 5x n−1 − 20n + 25<br />

3. x 0 = 1, x n = x n−1 + 12n<br />

4. x 0 = 5, x n = 2x n−1 + 9(5 n−1 )<br />

5. a 0 = 5, a j+1 = a 2 j + 2a j, j ≥ 0<br />

6. (AIME, 1984) x 19 = 94 ve<br />

x n + x n−1 = n 2 , n ≥ 1<br />

ise x 94 ’ün 1000 ile bölümünden kalan kaçtır<br />

Aşağıdaki (6 − 10) için ikinci dereceden yinelemelerin kapalı formlarını<br />

bulunuz.<br />

10 Benzer bir soru TÜBİTAK <strong>mat</strong>e<strong>mat</strong>ik olimpiyatlarında da sorulmuştur.


66 BÖLÜM 2. KONULAR<br />

7. x 0 = 0, x 1 = 1, x n = 10x n−1 − 21x n−2<br />

8. x 0 = 0, x 1 = 1, x n = 10x n−1 − 25x n−2<br />

9. x 0 = 0, x 1 = 1, x n = 10x n−1 − 21x n−2 + n<br />

10. x 0 = 0, x 1 = 1, x n = 10x n−1 − 21x n−2 + 2 n<br />

11. Bir düzlem üzerine çizilen n çember düzlemi parçalara ayırmaktadır.<br />

Buna göre, düzlem üzerindeki n çemberin ayırdığı parçaların sayısını<br />

veren denklemi bulunuz.<br />

12. Bir düzlem üzerine çizilen n doğrunun düzlem üzerinde ayırdığı parçaların<br />

sayısını veren denklemi bulunuz.<br />

2.8 Yen˙ıden Düzenleme [Rearrangement] Eş˙ıts˙ızl˙ığ˙ı<br />

Varsayalım elimizde iki reel üçlü (a 1 , a 2 , a 3 ) ve (b 1 , b 2 , b 3 ) olsun. Eğer ikinci<br />

üçlünün tüm permütasyonlarını düşünürsek elimizde 3! = 6 tane üçlünün olacağı<br />

açıktır. Biz bu üçlülerin kümesine P ve (x 1 , x 2 , x 3 ) ∈ P olsun diyelim.<br />

Buna göre varsayalım,<br />

S = a 1 x 1 + a 2 x 2 + a 3 x 3<br />

toplamıda elimizde bulunsun. Burada bizi ilgilendiren asıl soru S toplamının<br />

ne zaman en büyük, ne zaman en küçük olduğudur. Bu soruya cevap aramaya<br />

başlamadan evvel ileriki basamaklarda kullanacağımız bazı terimleri açıklayalım.<br />

Tanım. Varsayalım elimizde (a 1 , a 2 , a 3 ) ve (b 1 , b 2 , b 3 ) reel üçlüleri olsun. Buna<br />

göre,<br />

• Eğer iki üçlünün elemanlarıda artan veya iki üçlünün elemanlarıda azalan<br />

bir sırada yazılmışsa bu ikiliye Benzer Düzenli diyelim. Yani a 1 ≤<br />

a 2 ≤ a 3 ve b 1 ≤ b 2 ≤ b 3 veya a 1 ≥ a 2 ≥ a 3 ve b 1 ≥ b 2 ≥ b 3 durumları<br />

sağlansın.<br />

• Eğer iki üçlüden biri artan diğeri azalan sırada yazılmışsa bu üçlüyede<br />

Aykırı Düzenli diyelim.


2.8. YENİDEN DÜZENLEME [REARRANGEMENT] EŞİTSİZLİĞİ 67<br />

Örnek.<br />

a. (−1, 1, 3) ve (2, 5, 7) benzer düzenlidirler.<br />

b. Eğer 0 < a ≤ b ≤ c ise (a, b, c) ve ( 1 a , 1 b , 1 c<br />

) aykırı düzenlidir. Ama, (a, b, c)<br />

ve ( 1<br />

b+c , 1<br />

a+c , 1<br />

a+b<br />

) benzer düzenlidirler.<br />

c. Eğer 0 < a ≤ b ≤ c ve m ∈ R + ise (a, b, c) ve (a m , b m , c m ) benzer<br />

düzenliyken, (a, b, c) ve ( 1 1 1<br />

a<br />

, m b<br />

, m c<br />

) aykırı düzenlidir.<br />

m<br />

d. Eğer a ≤ b ≤ c ve n bir tek tamsayı ise (a, b, c) ve (a n , b n , c n ) benzer<br />

düzenlidir.<br />

Artık eşitsizliğimizi daha yakından tanıma zamanı geldi. Teorem [Rearragement<br />

Inequality]. (a 1 , a 2 , a 3 ) ve (b 1 , b 2 , b 3 ) iki reel üçlü <strong>olm</strong>ak üzere<br />

(x 1 , x 2 , x 3 ) üçlüsü (b 1 , b 2 , b 3 ) üçlüsünün bir permitasyonu olsun. Buna göre,<br />

• Eğer (a 1 , a 2 , a 3 ) ve (b 1 , b 2 , b 3 ) benzer düzenli ise<br />

olacaktır.<br />

a 1 b 1 + a 2 b 2 + a 3 b 3 ≥ a 1 x 1 + a 2 x 2 + a 3 x 3<br />

• Eğer (a 1 , a 2 , a 3 ) ve (b 1 , b 2 , b 3 ) aykırı düzenli ise<br />

olacaktır.<br />

a 1 b 1 + a 2 b 2 + a 3 b 3 ≤ a 1 x 1 + a 2 x 2 + a 3 x 3<br />

Kanıt. Varsayalım elimizde (a 1 , a 2 , a 3 ) ve (b 1 , b 2 , b 3 ) artan sırayla dizilmiş iki<br />

üçlü olsun. (x 1 , x 2 , x 3 ) üçlüsüde (b 1 , b 2 , b 3 ) üçlüsünün bir permütasyonu olsun<br />

ve x 1 ≥ x 2 olsun. S ve S ′ toplamlarınıda<br />

ve<br />

S = a 1 x 1 + a 2 x 2 + a 3 x 3<br />

S ′ = a 1 x 2 + a 2 x 1 + a 3 x 3<br />

olarak alalım. Burada S ′ toplamı, S toplamındaki x 1 ve x 2 ’nin yer değiştirmesiyle<br />

elde edildiği açıktır. Eğer bu iki toplamı farkını alırsak,<br />

S ′ − S = (x 1 − x<br />

} {{ } 2 )(a 2 − a 1 ) ≥ 0<br />

} {{ }<br />

+ +


68 BÖLÜM 2. KONULAR<br />

olacaktır. Demekki S ′ ≥ S’dir.<br />

Buna göre, x 1 ve x 2 ’nin yer değiştirmesi sadece S toplamının değerini artırmaktadır.<br />

Öyleyse, eğer tüm (x i , x j ) ikililerinin (x i ≥ x j , i < j) yerleri<br />

değiştirilirse toplam ancak en büyük olabilir. En büyük olacak toplamda zaten<br />

a 1 b 1 + a 2 b 2 + a 3 b 3 olacaktır. Benzer biçimde eğer (a 1 , a 2 , a 3 ) ve (b 1 , b 2 , b 3 )<br />

üçlülerinin ikiside azalan birer üçlü olarak seçilseydi, ispatın ikinci kısmıda<br />

buradan yapılabilirdi.<br />

Şimdi, bu yeni eşitsizliğimizi birkaç örnek üzerinde uygulayalım.<br />

Örnek.<br />

a, b, c ∈ R <strong>olm</strong>ak üzere<br />

i. a 2 + b 2 + c 2 ≥ ab + bc + ca<br />

ii a n + b n + c n ≥ a n−1 b + b n−1 c + c n−1 a<br />

eşitsizliklerini gösteriniz.<br />

Çözüm. Sorunun birinci şıkkı zaten ikinci şıkkın özel bir durumu olduğundan<br />

sadece ikinci şıkkı çözmemiz yeterli olacaktır. Buna göre eğer üçlülerimizi<br />

(a, b, c) ve (a n−1 , b n−1 , c n−1 ) benzer düzenlileri olarak belirlersek, istenen eşitsizlik<br />

yeniden düzenleme eşitsizliği ile,<br />

olarak bulunur.<br />

Örnek.<br />

aa n−1 + bb n−1 + cc n−1 ≥ ab n−1 + bc n−1 + ca n−1<br />

a, b, c > 0 oduğuna göre, aşağıdaki eşitsizlikleri kanıtlayınız.<br />

i.<br />

a+b+c<br />

abc<br />

≤ 1 a 2 + 1 b 2 + 1 c 2<br />

ii.<br />

a 2<br />

b 2<br />

iii.<br />

a 2<br />

b<br />

Çözüm.<br />

+ b2<br />

c 2 + c2<br />

a 2<br />

≥ b a + c b + a c<br />

+ b2<br />

c + c2 a ≥ a + b + c<br />

• a ≤ b ≤ c olarak kabul edelim ve ( 1 a , 1 b , 1 c ) ve ( 1 a , 1 b , 1 c<br />

) benzer düzenlilerini<br />

seçelim. Buna göre,<br />

1 1<br />

a a + 1 b<br />

1<br />

b + 1 1<br />

c c<br />

≥<br />

1 1<br />

a b + 1 1<br />

b c + 1 1<br />

c a<br />

1<br />

a 2 + 1 b 2 + 1 c 2 ≥ 1 ab + 1 bc + 1 ac


2.8. YENİDEN DÜZENLEME [REARRANGEMENT] EŞİTSİZLİĞİ 69<br />

olacaktır.<br />

• Eğer ( a b , b c , c a ) ve ( a b , b c , c a<br />

) benzer sıralılarını alırsak,<br />

elde edilecektir.<br />

a a<br />

b b + b c<br />

b<br />

c + c c<br />

a a ≥ a b<br />

b c + b c<br />

c a + c a<br />

a<br />

b<br />

• Bu seferde sıralı ikililerimizi aykırı düzenliler arasından seçelim. Yani,<br />

(a 2 , b 2 , c 2 ) ve ( 1 a , 1 b , 1 c<br />

) kullanacağımız üçlülerimiz olsun. Buna göre,<br />

elde edilecektir.<br />

a 2 1 a + b2 1 b + c2 1 c ≤ a2 1 b + b2 1 c + c2 1 a<br />

Örnek.<br />

göre,<br />

[1963, Moskova Mate<strong>mat</strong>ik Olimpiyatı] a, b, c > 0 olduğuna<br />

a<br />

b + c +<br />

b<br />

a + c +<br />

c<br />

a + b ≥ 3 2<br />

eşitsizliğini kanıtlayınız.<br />

Çözüm. Varsayalım a ≤ b ≤ c olsun ve benzer sıralılarımız da (a, b, c) ve<br />

olsun. Buna göre,<br />

1<br />

(<br />

a + b , 1<br />

a + c , 1<br />

a + b ) (2.1)<br />

a 1<br />

b + c + b 1<br />

c + a + c 1<br />

a + b<br />

a 1<br />

b + c + b 1<br />

a + c + c 1<br />

a + b<br />

≥<br />

≥<br />

a<br />

c + a + b<br />

a + b + c<br />

b + c<br />

a<br />

a + b + b<br />

b + c + c<br />

a + c<br />

eşitsizliklerini elde edebiliriz. Eğer [1] ve [2.3] eşitsizlerini altalta toplarsak,<br />

a<br />

2(<br />

b + c +<br />

b<br />

a + c +<br />

c<br />

a + b ) ≥<br />

a<br />

a + c +<br />

b<br />

a + b +<br />

c<br />

b + c +<br />

a<br />

a + b +<br />

eşitsizliğindende soruda istenen [2.1] eşitsizliğini elde ederiz.<br />

b<br />

b + c +<br />

(2.2)<br />

(2.3)<br />

c<br />

a + c = 3


70 BÖLÜM 2. KONULAR<br />

Örnek.<br />

düzenliler ise<br />

[Chebyshev Eşitsizliği] Eğer (a 1 , a 2 , a 3 ) ve (b 1 , b 2 , b 3 ) benzer<br />

a 1 b 1 + a 2 b 2 + a 3 b 3<br />

3<br />

≥ ( a 1 + a 2 + a 3<br />

3<br />

)( b 1 + b 2 + b 3<br />

) (2.4)<br />

3<br />

eşitsizliğini kanıtlayınız.<br />

Çözüm. Soruda zaten benzer düzenli üçlüler verildiğine göre, yeniden düzenleme<br />

metodunu kullanalım<br />

a 1 b 1 + a 2 b 2 + a 3 b 3 = a 1 b 1 + a 2 b 2 + a 3 b 3 (2.5)<br />

a 1 b 1 + a 2 b 2 + a 3 b 3 ≥ a 1 b 2 + a 2 b 3 + a 3 b 1 (2.6)<br />

a 1 b 1 + a 2 b 2 + a 3 b 3 ≥ a 1 b 3 + a 2 b 1 + a 3 b 2 (2.7)<br />

eşitsizliklerini elde ederiz. Eğer [2.5], [2.6] ve [2.7] eşitsizliklerini altalta toplarsak,<br />

3(a 1 b 1 + a 2 b 2 + a 3 b 3 ) ≥ a 1 (b 1 + b 2 + b 3 ) + a 2 (b 1 + b 2 + b 3 ) + a 3 (b 1 + b 2 + b 3 ))<br />

eşitsizliğini ve sonuç olarakta soruda verilen [2.4] eşitsizliğini elde ederiz.<br />

Not. Benzer biçimde, (a 1 , a 2 , a 3 ) ve (b 1 , b 2 , b 3 ) aykırı düzenlileri içinde,<br />

a 1 b 1 + a 2 b 2 + a 3 b 3<br />

3<br />

eşitsizliği elde edilebilir.<br />

≤ ( a 1 + a 2 + a 3<br />

3<br />

)( b 1 + b 2 + b 3<br />

) (2.8)<br />

3<br />

Örnek. [Aritmetik Orta - Karesel Orta Eşitsizliği] a 1 , a 2 , a 3 reel<br />

sayılar <strong>olm</strong>ak üzere verilen<br />

a 1 + a 2 + a 3<br />

3<br />

≤<br />

√<br />

a<br />

2<br />

1 + a 2 2 + a2 3<br />

3<br />

(2.9)<br />

eşitsizliğini kanıtlayınız.<br />

Çözüm. Sorunun çözümü için Örnek 0.7’de kanıtladığımız eşitsizliği kullanmamız<br />

kafidir. Buna göre, (a 1 , a 2 , a 3 ) ve (a 1 , a 2 , a 3 ) benzer düzenlileri için,<br />

a 2 1 + a2 2 + a2 3<br />

3<br />

≤ ( a 1 + a 2 + a 3<br />

3<br />

)( a 1 + a 2 + a 3<br />

3<br />

) = ( a 1 + a 2 + a 3<br />

) 2 (2.10)<br />

3


2.8. YENİDEN DÜZENLEME [REARRANGEMENT] EŞİTSİZLİĞİ 71<br />

olduğuna göre, soruda istenen [2.9] eşitsizliği kanıtlanmış olur.<br />

Örnek. [Aritmetik Orta-Geometrik Orta Eşitsizliği]a 1 , a 2 , a 3 pozitif sayılar<br />

<strong>olm</strong>ak üzere verilen<br />

a 1 + a 2 + a 3<br />

3<br />

≥ 3√ a 1 a 2 a 3 (2.11)<br />

eşitsizliğini kanıtlayınız.<br />

Çözüm. P = 3√ a 1 a 2 a 3 <strong>olm</strong>ak üzere, sorua kullanacağımız aykırı düzenlilerimizi,<br />

x 1 = a 1<br />

P , x 2 = a 1a 2<br />

P 2 , x 3 = a 1a 2 a 3<br />

P 3 = 1 ve (2.12)<br />

y 1 = 1 x 1<br />

, y 2 = 1 x 2<br />

, y 3 = 1 x 3<br />

= 1 (2.13)<br />

olarak seçelim. Burada (x 1 , x 2 , x 3 ) artan sıralı ise (y 1 , y 2 , y 3 ) azalan sıralı olacaktır.<br />

Buna göre,<br />

olacaktır. Buradan da,<br />

x 1 y 1 + x 2 y 2 + x 3 y 3 ≤ x 1 y 3 + x 2 y 1 + x 3 y 2<br />

1 + 1 + 1 ≤ a 1<br />

P + a 2<br />

P + a 3<br />

P = a 1 + a 2 + a 3<br />

P<br />

3P ≤ a 1 + a 2 + a 3<br />

P ≤ a 1 + a 2 + a 3<br />

3<br />

olacağından, istenen kanıt tamamlanmış olur.<br />

Örnek. Chebyshev eşitsizliğini iki reel sayı için gösteriniz.<br />

Çözüm. Varsayalım benzer düzenli ikililerimiz (a 1 , a 2 ) ve (b 1 , b 2 ) olsun.<br />

Buna göre,<br />

a 1 b 1 + a 2 b 2 = a 1 b 1 + a 2 b 2<br />

a 1 b 1 + a 2 b 2 ≥ a 1 b 2 + a 2 b 1


72 BÖLÜM 2. KONULAR<br />

eşitsizlikleri taraf tarafa toplanırsa,<br />

olacaktır.<br />

Örnek.<br />

2(a 1 b 1 + a 2 b 2 ) ≥ (a 1 + a 2 )(b 1 + b 2 )<br />

a 1 b 1 + a 2 b 2<br />

≥ ( a 1 + a 2<br />

)( b 1 + b 2<br />

)<br />

2<br />

2 2<br />

a n + b n<br />

a + b<br />

≥ 1 2 (an−1 + b n−1 )<br />

eşitsizliğini kanıtlayınız.<br />

Çözüm. Örnek 0.11’de kanıtladığımız eşitsizliği kullanalım. Buna göre<br />

ikililerimiz (a, b) ve (a n−1 , b n−1 ) ikilileri olsun. Buna göre,<br />

aa n−1 + bb n−1<br />

2<br />

a n + b n<br />

a + b<br />

≥ ( a + b + b n−1<br />

2 )(an−1 )<br />

2<br />

≥ 1 2 (an−1 + b n−1 )<br />

Örnek.<br />

a, b ≥ 0 <strong>olm</strong>ak üzere, aşağıdaki eşitsizlikleri kanıtlayınız.<br />

a. 2(a 5 + b 5 ) ≥ (a 3 + b 3 )(a 2 + b 2 )<br />

b. a 9 + b 9 ≥ a 2 b 2 (a 5 + b 5 )<br />

c. (a + b) n ≤ 2 n−1 (a n + b n )<br />

Çözüm.<br />

a. (a 2 , b 2 ), (a 3 , b 3 ) benzer düzenlileri olsun. Buna göre,<br />

a 2 a 3 + b 2 b 3<br />

2<br />

≥ ( a2 + b 2<br />

2<br />

)( a3 + b 3<br />

) ⇒ 2(a 5 + b 5 ) ≥ (a 2 + b 2 )(a 3 + b 3 )<br />

2<br />

olarak bulunur.


2.8. YENİDEN DÜZENLEME [REARRANGEMENT] EŞİTSİZLİĞİ 73<br />

b. Bu şıktada benzer düzenlilerimizi (a 4 , b 4 ) ve (a 5 , b 5 ) olarak seçelim. Buna<br />

göre,<br />

a 4 a 5 + b 4 b 5<br />

2<br />

≥ ( a4 + b 4<br />

2<br />

)( a5 + b 5<br />

2<br />

) ≥ √ a 4 b 4 ( a5 + b 5<br />

) = a 2 b 2 ( a5 + b 5<br />

)<br />

2<br />

2<br />

olacaktır. Bu eşitsizliktende,a 9 + b 9 ≥ a 2 b 2 (a 5 + b 5 ) eşitsizliği elde edilir.<br />

c. Soruyu Chebyshev Eşitsizliği’ni kullanarak çözmeye çalışalım. İkililerimiz<br />

(a n−1 , b n−1 ) ve (a, b) olsun. Buna göre,<br />

a n + b n ≥ 1 2 (an−1 + b n−1 )(a + b) = 1 2 (an−2 a + b n−2 b)(a + b)<br />

a n + b n ≥ 1 2 2 (an−2 + b n−2 )(a + b)(a + b)<br />

a n + b n<br />

≥<br />

Buna göre istenen eşitsizlik,<br />

elde edilmiş olur.<br />

1<br />

1<br />

(a + b)(a + b) · · · (a + b) =<br />

2n−1 2 n−1 (a + b)n olacaktır.<br />

2 n−1 (a n + b n ) ≥ (a + b) n<br />

Örnek.<br />

a, b, c > 0 ve n ∈ Z + ise<br />

a n<br />

b + c +<br />

bn<br />

a + c +<br />

cn<br />

a + b ≥ an−1 + b n−1 + c n−1<br />

2<br />

eşitsizliğini kanıtlayınız.<br />

Çözüm. a ≤ b ≤ c <strong>olm</strong>ak üzere benzer düzenlilerimiz (a n , b n , c n ) ve<br />

( 1<br />

b+c , 1<br />

a+c , 1<br />

a+b<br />

) olarak seçelim. Buna göre,<br />

a n<br />

b + c +<br />

a n<br />

b + c +<br />

bn<br />

a + c +<br />

bn<br />

a + c +<br />

cn<br />

a + b<br />

cn<br />

a + b<br />

≥<br />

≥<br />

a n<br />

a + c +<br />

a n<br />

b + a +<br />

bn<br />

a + b +<br />

bn<br />

b + c +<br />

cn<br />

b + c<br />

cn<br />

a + c<br />

eşitsizliklerini elde ederiz. Eğer bu iki eşitsizliği taraftarafa toplarsak,<br />

2( an<br />

b + c +<br />

bn<br />

a + c +<br />

cn<br />

a + b ) ≥ an + b n<br />

a + b<br />

+ bn + c n<br />

b + c<br />

+ an + c n<br />

a + c<br />

(2.14)<br />

(2.15)


74 BÖLÜM 2. KONULAR<br />

eşitsizliğini elde ederiz. Bundan sonraki basamaktada Örnek 0.12 ’de kanıtladığımız<br />

eşitsizliği kullanırsak,<br />

2( an<br />

b + c +<br />

bn<br />

a + c +<br />

cn<br />

a + b ) ≥ an + b n<br />

a + b<br />

+ bn + c n<br />

b + c<br />

+ an + c n<br />

a + c<br />

≥ 1 2 (an−1 + b n−1 ) + 1 2 (bn−1 + c n−1 ) + 1 2 (an−1 + c n−1 )<br />

= a n−1 + b n−1 + c n−1<br />

eşitsizliğinden, soruda bizden istenen eşitsizliğe ulaşılır.<br />

Örnek.<br />

a, b, c > 0 <strong>olm</strong>ak üzere,<br />

a a b b c c ≥ (abc) a+b+c<br />

3<br />

eşitsizliğini kanıtlayınız.<br />

Çözüm. a ≤ b ≤ c olarak alalım. Üçlülerimizide, biraz sıradışı bir seçim<br />

yaparak, (a, b, c) ve (log a, log b, log c) olarak alalım. Buna göre,Chebyshev<br />

eşitsizliğinden<br />

a log a + b log b + c log c<br />

3<br />

eşitsizliği elde edilir. Buna göre,<br />

log a a b b c c<br />

olacağından istenen eşitsizlik,<br />

3<br />

≥ ( a + b + c log a + log b + log c<br />

)( )<br />

3<br />

3<br />

≥ ( a + b + c log abc<br />

)( )<br />

3 3<br />

olarak bulunur.<br />

log a a b b c c ≥ log(abc) a+b+c<br />

3 ⇒ a a b b c c ≥ (abc) a+b+c<br />

3<br />

Örnek. A, B, C bir üçgenin açıları <strong>olm</strong>ak üzere (radyan cinsinden), a, b, c<br />

uzunukları da bu üçgenin kenar uzunluklarıdır. p = 1 2<br />

(a+b+c) olduğuna göre,<br />

A<br />

p − A +<br />

B<br />

p − b +<br />

C<br />

p − c ≥ 3π p<br />

(2.16)


2.9. TRİGONOMETRİK DEĞİŞKEN DEĞİŞTİRME 75<br />

eşitsizliğini kanıtlayınız.<br />

Çözüm. Varsayalım A ≤ B ≤ C olsun. Buna göre benzer düzenli üçlülerimizi<br />

(A, B, C) ve ( 1<br />

p−a , 1<br />

p−b , 1<br />

p−c<br />

) seçebiliriz. Chebyshev eşitsizliğinden,<br />

1<br />

3 ( A<br />

p − A + B<br />

p − b + C<br />

p − c ) ≥ (A + B + C 1<br />

)(<br />

3 p − a + 1<br />

p − b + 1<br />

p − c )1 3<br />

= π 3 ( 1<br />

p − a + 1<br />

p − b + 1<br />

p − c )1 3<br />

≥ π 9 · 9<br />

p<br />

eşitsizliğindende [2.16] eşitsizliği kolaylıkla çıkarılır.<br />

Örnek.<br />

[1995,IM0] a, b, c ∈ R + ve a · b · c = 1 olduğuna göre,<br />

1<br />

a 3 (b + c) + 1<br />

b 3 (a + c) + 1<br />

c 3 (a + b) ≥ 3 2<br />

(2.17)<br />

eşitsiliğini kanıtlayınız.<br />

Çözüm. Varsayalım x = 1 a , y = 1 b , z = 1 c<br />

olsun. a · b · c = 1 olduğundan,<br />

x · y · z = 1 olacağı açıktır. Buna göre, yeni değişkenlere göre eşitizliğimiz<br />

düzenlersek,<br />

x 2<br />

y + z +<br />

y2<br />

x + z +<br />

z2<br />

y + x ≥ 3 2<br />

eşitsizliğini elde ederiz. Çözümün bundan sonraki basamakları sadece Örnek<br />

0.14’deki eşitsizliğin kanıtının kullanılmasıyla olacaktır. Buna göre,<br />

olacaktır.<br />

x 2<br />

y + z +<br />

y2<br />

x + z +<br />

z2<br />

y + x ≥ x + y + z<br />

2<br />

≥ 3 2 3√ xyz = 3 2<br />

2.9 Tr˙ıgonometr˙ık Değ˙ışken Değ˙ışt˙ırme<br />

Trigonometrik özdeşliklerin çokluğu, bu denklemleri kullanarak cebirsel<br />

denklemlere oldukça kolay çözümler geliştirmemize yardımcı olur. Değişken değiştirmede,<br />

hangi özdeşliği kullanacağımız aslında çoğu zaman verilen sorunun


76 BÖLÜM 2. KONULAR<br />

içine gömülüdür. Bu sebebten sorulara çözmeye başlamadan önce denklemi<br />

nasıl yeniden düzenleyeceğimizin kararına varmamız yerinde olur.<br />

Örnek.<br />

Aşağıdaki denklem sisteminin tüm reel çözümlerini bulunuz.<br />

x 3 − 3x = y<br />

y 3 − 3y = z<br />

z 3 − 3z = x<br />

Çözüm. Soruda verilen x 3 − 3x formu kosinüs üç kata açı özdeşliğini<br />

anımsatıyor. Ancak tabiki x 3 değişkeninin önündeki katsayı yok. Bu yüzden<br />

değişken yerine çift kat açı kosinüsü yerine, sadece cos x değişkeni almamız<br />

yerinde olacaktır. Buna göre x = 2 cos u, y = 2 cos v, z = 2 cos w u, v, w ∈ [0, π]<br />

ise<br />

2 cos 3u = 2 cos v<br />

2 cos 3v = 2 cos w<br />

2 cos 3w = 2 cos u<br />

sistemini elde ederiz. Kosinüs üç kat açı formülünü cos 3u ve cos v için kullanırsak<br />

cos 9u = cos 3v olacaktır. Bu eşitliği ikinci denklem de kullanırsak<br />

cos 9u = cos w olur. Aynen daha önceki gibi cos 27u = cos 3w ise üçüncü denklem<br />

cos 27u = cos u olacaktır.Demek ki, u = kπ/14, k = 0, 1, 2, · · · , 14 ve<br />

u = kπ/13, k = 1, 2, · · · , 12 olacağından<br />

ve<br />

x = 2 cos kπ/14, y = 2 cos 3kπ/14, z = 2 cos 9kπ/14, k = 0, 1, · · · , 14<br />

x = 2 cos kπ/13, y = 2 cos 3kπ/13, z = 2 cos 9kπ/13, k = 1, 2, · · · , 12<br />

olacağından 3 × 3 × 3 = 27 tane çözüm elde edilir.<br />

Şimdide tanjant fonksiyonunu kullanarak sıradaki örneği çözmeye çalışalım.<br />

Örnek.<br />

{x n } ∞ n=1<br />

<strong>olm</strong>ak üzere,<br />

x n+1 =<br />

√<br />

3xn − 1<br />

x n + √ 3 , n 1


2.9. TRİGONOMETRİK DEĞİŞKEN DEĞİŞTİRME 77<br />

İse , bu dizinin periyodik olduğunu kanıtlayınız.<br />

Çözüm.<br />

Tanjant fark formülünü hatırlarsak<br />

tan (a − b) =<br />

tan a − tan b<br />

1 − tan a · tan b<br />

olacaktır. tan (π/6) = 1/ √ 3 olduğunuda biliyoruz. Buna göre,<br />

x n+1 = x n − 1/ √ 3<br />

1 + x n 1/ √ 3<br />

olarak yazılabilir. Eğer x 1 = tan t alırsak x 2 = tan (t − π/6) olacağından,<br />

x n = tan (t − (n − 1) π/6) , n 1<br />

olur. Tanjant fonksiyonuda periyodik olduğuna göre, demek ki, x n = x n+6<br />

olacağından soruda verilen dizimiz periyodiktir ve periyodu 6 dır.<br />

Örnek. a, b, c, x, y, z > 0 ise<br />

(<br />

a 2 + x 2) ( b 2 + y 2) ( c 2 + z 2) (ayz + bzx + cxy − xyz) 2<br />

eşitsizliğini kanıtlayınız.<br />

Çözüm. a = x tan α, b = y tan β, c = z tan γ α, β, γ ∈ [0, π/2] olarak<br />

alalım. Buna göre,<br />

1 ≥ cos 2 (x + y + z) = (sin x · sin (y + z) − cos x · cos (y + z)) 2<br />

eşitsizliğinin iki tarafınıda<br />

ile bölersek,<br />

= (sin x · sin y · sin z + sin x · cos y · sin z + cos x · sin y · sin z − cos x<br />

cos 2 α cos 2 β cos 2 γ<br />

sec 2 α sec 2 β sec 2 γ (tan α + tan β + tan γ − 1) 2<br />

olacağından,<br />

(<br />

x 2 tan 2 α + x 2) ( y 2 tan 2 β + z 2) ( z 2 tan 2 γ + z 2) x 2 y 2 z 2 (tan α + tan β + tan γ − 1) 2<br />

olacaktır. Buda zaten soruda istenen eşitsizliği doğrular.


78 BÖLÜM 2. KONULAR<br />

2.9.1 Çalışma Soruları<br />

1. a reel parametresinin hangi değerleri için<br />

√<br />

1 − x 2 a − x<br />

eşitsizliğini sağlayan bir x reel değeri vardır<br />

2. (0, 1) aralığından dört farklı sayı alınıyor. Buna göre, bu sayılardan iki<br />

tanesi olan xve y’nin<br />

eşitliğini sağlayacağını kanıtlayınız.<br />

3. Seçilen 4 farklı reel sayı arasından<br />

0 < x √ 1 − y 2 − y √ 1 − x 2 < 1 2<br />

1 + ab<br />

√<br />

1 + a 2 √ 1 + b 2 > 1 2<br />

eşitsizliğini sağlayan bir (a, b) ikilisinin bulunabileceğini kanıtlayınız.<br />

4. x ∈ R <strong>olm</strong>ak üzere verilen x 2 + ( 4x 3 − 3x ) 2 = 1 denkleminin tüm reel<br />

çözümlerini bulunuz.<br />

5.<br />

I =<br />

∫ √<br />

2 + √ 2 + · · · + 2 + xdx<br />

integralini hesaplayınız.(Not: İfade içinde n ≥ 1 tane kök vardır.)<br />

6. {x n } ∞ n1 dizisi √ x n+2 + 2 x n 2 eşitsizliğini sağlandığına göre, x 1986<br />

’nın alabileceği tüm değerleri bulunuz.<br />

7.<br />

2x + x 2 y = y<br />

2y + y 2 z = z<br />

2z + z 2 x = x<br />

denklem sisteminin tüm reel çözümlerini bulunuz.


2.9. TRİGONOMETRİK DEĞİŞKEN DEĞİŞTİRME 79<br />

8. Aşağıdaki denklem sisteminin tüm reel çözümlerini bulunuz.<br />

x 1 − 1 x 1<br />

= 2x 2<br />

x 2 − 1 x 2<br />

= 2x 3<br />

x 3 − 1 x 3<br />

= 2x 4<br />

x 4 − 1 x 4<br />

= 2x 1<br />

9. x, y ∈ R <strong>olm</strong>ak üzere, aşağıdaki eşitsizliği kanıtlayınız,<br />

− 1 2<br />

<br />

(x + y) (1 − xy)<br />

(1 + x 2 ) (1 + y 2 ) 1 2<br />

10. x ∈ R , {x n } n<br />

ve x 1 = x <strong>olm</strong>ak üzere<br />

x n+1 = 1<br />

x n<br />

− 1<br />

1 + x n<br />

eşitliği veriliyor. Eğer x n = ±1 ise dizimiz kilitleniyor, çünkü x n+1 tanımlanamıyor.<br />

Buna göre, 8. terimden sonra kilitlenen bu şekilde bir dizi<br />

var mıdır<br />

11. Reel tanımlı {a k } ∞ k=1 dizisi, k ∈ Z+ <strong>olm</strong>ak üzere<br />

a k+1 = k · a k + 1<br />

k − a k<br />

olarak veriliyor. Buna göre bu dizinin sonsuz sayıda pozitif ve sonsuz<br />

sayıda negatif terimi olduğunu kanıtlayınız.<br />

12. −1 a 1 a 2 · · · a n 1 ise<br />

n−1<br />

√<br />

∑<br />

√ (1 ) ( )<br />

1 − a i a i+1 − − a<br />

2<br />

i<br />

1 − a<br />

2<br />

π √ 2<br />

i+1<br />

<<br />

2<br />

i=1<br />

eşitsizliğini kanıtlayınız.


80 BÖLÜM 2. KONULAR<br />

∑<br />

13. x 0 = 0, x 1 , x 2 , · · · , x n > 0 ve n x k = 1 ise<br />

n∑<br />

k=1<br />

k=1<br />

x k<br />

√ 1 + x0 + x 1 + · · · + x k−1<br />

√<br />

xk + x k+1 + · · · + x n<br />

< π 2<br />

eşitsizliğini kanıtlayınız.<br />

14. x, y, z ∈ (0, 1) <strong>olm</strong>ak üzere<br />

x 2 + y 2 + z 2 + 2xyz = 1<br />

eşitliğini sağlayan tüm (x, y, z) tamsayı üçlülerini bulunuz.<br />

15. a, b, c pozitif tamsayılar <strong>olm</strong>ak üzere verilen<br />

x + y + z = a + b + c<br />

4xyz − (a 2 x + b 2 y + c 2 z) = abc<br />

eşitliklerini sağlayan x, y, z reel sayılarını bulunuz.<br />

16. cos 2π/5 ifadesinin değerini bulunuz.<br />

2.9.2 Çözümler<br />

1. t ∈ [0, π] <strong>olm</strong>ak üzere cos t = x olsun. Çünkü |x| ≤ 1 <strong>olm</strong>ak zorundadır.<br />

Buradan √ 1 − x 2 = √ 1 − cos 2 x = sin t olacaktır. Çünkü t ∈ [0, π] için<br />

sinüs fonksiyonu pozitiftir. Buna göre eşitsizliğimiz,<br />

sin t + cos t a<br />

olacaktır.<br />

f (t) = sin t + cos t = 2 sin π (<br />

4 cos t − π )<br />

4<br />

= √ (<br />

2 cos t − π )<br />

4<br />

olacağından, f (t)’nin [0, π] aralığındaki en büyük değeri √ 2 olacaktır.<br />

Buna göre, a’nın bulunduğu aralık √ 2’den büyük <strong>olm</strong>ayan reel sayılar<br />

kümesi olacaktır.


2.9. TRİGONOMETRİK DEĞİŞKEN DEĞİŞTİRME 81<br />

2. Varsayalım sayılarımız a 1 , a 2 , a 3 , a 4 olsun. Burada, a k = sin t k , t k ∈<br />

(0, π/2) alalım. Buna göre aslında problemde sorulan soru i ve j gibi<br />

iki indisin<br />

0 < sin t i cos t j − sin t j cos t i < 1 2<br />

eşitsizliğini sağlayacak biçimde varolup <strong>olm</strong>adıklarıdır. Eğer eşitsizliğimizi<br />

düzenlersek<br />

sin t i cos t j − sin t j cos t i = sin (t i − t j )<br />

olacaktır. Buna göre, ispatlamamız gereken durum artık t i > t j ve<br />

t i − t j < π/6 olacak şekilde i ve j’nin bulunduğunu göstermek olacaktır.<br />

Burada devreye güvercin yuvası ilkesi girer öyleki, seçeceğimiz dört<br />

sayıdan iki tanesi kesinlikle<br />

(0, π/6] , (0, π/3] , (π/3, π/2)<br />

aralığında olacaktır. Buna göre eşitsizliği sağlayan t i ve t j vardır, dolayısıyla<br />

x ve y’de vardır.<br />

3. x ∈ (0, π) <strong>olm</strong>ak üzere, tüm reel sayılar tan x cinsinden ifade edilebilir.<br />

Buna göre a = tan x ve b = tan y olarak alırsak<br />

1 + tan x · tan y<br />

√<br />

1 + tan 2 x · √1<br />

+ tan 2 y<br />

= cos x · cos y + sin x · sin y = cos (x − y)<br />

olacaktır. Buna göre şimdi problemimizi yeniden kuralım. Yeni durumumuz,<br />

cos(x − y) > 1/2<br />

olduğundan (x − y) < π/3 <strong>olm</strong>alıdır. Gerçektende (0, π) aralığında seçeceğimiz<br />

dört sayıdan ikisinin farkı π/3’ten küçük 11 olacaktır.<br />

4. Dikkat edilirse |x| ≤ 1 ve |4x 3 − 3x| ≤ 1 olarak verilmiştir. Buna göre,<br />

x = cos a, 0 ≤ a ≤ π alabiliriz. Eğer cos 3a ve cos a ifadelerini kullanırsak<br />

soruda verilen denklemimiz<br />

cos 2 a + cos 2 3a = 1<br />

11 Pigeon Hole Principle yada Güvercin Yuvası İlkesi bu küçüklüğü garantiler.


82 BÖLÜM 2. KONULAR<br />

olacaktır. Buradan da,<br />

1 + cos 2a<br />

2<br />

+<br />

1 + cos 6a<br />

2<br />

= 1<br />

olacağından 2a ve 4a değerleri<br />

2a = π 2 , 3π 2 ve 4a = π 2 , 3π 2 , 5π 2 , 7π 2<br />

olarak bulunur. Buna göre istenilen çözümler, ± π 2 ve ± √<br />

2± √ 2<br />

2<br />

değerlerini<br />

alacaktır.<br />

5. Varsayalım x = 2 cos t olsun, buna göre<br />

√<br />

2 +<br />

√<br />

2 + · · · + √ 2 + x =<br />

=<br />

√<br />

√<br />

2 + 2 + · · · + √ 2 + 2 cos t<br />

√<br />

√<br />

2 +<br />

= 2 cos t<br />

2 n<br />

2 + · · · + 2 cos t 2<br />

olacaktır. Buradan integralimiz<br />

∫<br />

I =<br />

2 cos t ∫<br />

2 n dx = −2 cos t 2 sin t dt<br />

2n ∫<br />

= −4 sin t · cos t<br />

2 n dt<br />

∫ (<br />

)<br />

= −2 sin 2n + 1<br />

2 n t − sin 2n − 1<br />

2 n t<br />

dt<br />

eşitliğinden istenilen cevap,<br />

2 n+1 ( 2<br />

n+1<br />

2 n + 1 cos 2 n + 1 arccos x ) (<br />

− 2n+1 2 n<br />

2 2 n − 1 cos − 1<br />

2 n arccos x )<br />

+ c<br />

2<br />

olarak bulunur.


2.9. TRİGONOMETRİK DEĞİŞKEN DEĞİŞTİRME 83<br />

6. 0 ≤ x n ≤ 2 olduğuna göre, x n = 2 cos y n , y n ∈ [0, π/2] değişken değiştirmesini<br />

kullanabiliriz. Buna göre<br />

√<br />

xn+2 + 2 ≤ x n<br />

eşitsizliği ve<br />

eşitliğinden<br />

cos 2α + 1 = 2 cos 2 α<br />

( )<br />

yn + 2<br />

cos ≤ cos y n<br />

2<br />

eşitsizliği bulunacaktır. Seçilen aralıkta kosinüs fonksiyonu azalan bir<br />

fonksiyon olduğuna göre,<br />

y n+2<br />

≥ y n<br />

2<br />

olacaktır. Buradan da<br />

y n ≤ y n+2k<br />

2 k , k ∈ Z +<br />

elde edilir. Burada k değeri sonsuza giderken y n ≤ 0 yani y n = 0 olacaktır.<br />

Dolayısıyla<br />

x n = 2 cos y n = 2 · 1 = 2<br />

olacağından x n = 2 ve x 1986 = 2 bulunur.<br />

7. Eğer değişkenlerden herhangi biri, varsayalım x = ±1 olursa<br />

2x + x 2 y = y ⇒ 2 · ±1 + y = y ⇒ 2 = 0<br />

olacaktır ki, bu durum imkansızdır. Buna göre denklem sistemimizi yeniden<br />

yazarsak,<br />

2x<br />

1 − x 2 = y<br />

2y<br />

1 − y 2 = z<br />

2z<br />

1 − z 2 = x


84 BÖLÜM 2. KONULAR<br />

eşitliklerini elde ederiz. Burada da x = tan a, a ∈ (−π/2, π/2) olarak<br />

alalım. Buna göre ilk iki denklemden y = tan 2a ve z = tan 4a olacaktır.<br />

Son denklemde ise tan 8a = tan a eşitliği elde edilir. Buna göre 8a −<br />

a = kπ ise a = kπ/7 olacaktır. a ∈ (−π/2, π/2) olduğuna göre k =<br />

−3, −2, −1, 0, 1, 2, 3 değerlerini alabiliriz. Buradan da istenilen değerler<br />

bulunabilir.<br />

8. Sorunun çözümüne yardımcı olacak trigonometrik özdeşlik aslında sorunun<br />

içine gömülmüştür. Eğer çift kat açı için kotanjant toplamını kontrol<br />

edersek<br />

2 cot 2α = cot α − 1<br />

cot α<br />

eşitliğini elde ederiz. Eğer x 1 = cot α, α ∈ (0, π) olarak alırsak x 2 =<br />

cot 2α olacağından, x 3 = cot 4α ve x 4 = cot 8α olacaktır. Benzer şekilde<br />

döngü başa geleceğinden x 1 = cot 16α olacaktır. Demek ki cot α =<br />

cot 16α yani 16α − α = kπ, k ∈ Z eşitliği bulunacaktır. Buradanda<br />

çözümlerimiz<br />

α = kπ , k = 1, 2, 3, · · · , 14<br />

15<br />

olacağından<br />

olacaktır.<br />

x 1 = cot kπ<br />

15 , x 2 = cot 2kπ<br />

15 , x 3 = cot 4kπ<br />

15 , x 4 = cot 8kπ<br />

15<br />

9. Varsayalım x = tan a ve y = tan b olsun. Buradan,<br />

eşitlikleri elde edilir. Ayrıca,<br />

sin(a + b)<br />

x + y = tan a + tan b =<br />

cos a cos b<br />

cos(a + b)<br />

1 − xy = 1 − tan a tan b =<br />

cos a cos b<br />

1<br />

1 + x 2 = cos 2 a<br />

1<br />

1 + y 2 = cos 2 b


2.9. TRİGONOMETRİK DEĞİŞKEN DEĞİŞTİRME 85<br />

eşitlikleri elde edilir. Eğer yerine koyarsak,<br />

cos a cos b · cos(a+b)<br />

cos a cos b<br />

1 1<br />

cos 2 a cos 2 b<br />

− 1 2 sin(a+b)<br />

1 2<br />

ise −1 ≤ 2 sin(a + b)cos(a + b) ≤ 1 eşitsizliğini kanıtlamamız gerekir.<br />

Zaten bu eşitsizlik −1 ≤ sin(2a + 2b) ≤ 1 olduğundan kanıt tamamlanır.<br />

10.<br />

1<br />

− 1 = 2x n<br />

1 − x n 1 + x n 1 − x 2 n<br />

eşitliğini yazabiliriz. Eğer x 1 = tan β, β ∈ (−π/2, π/2) alırsak<br />

x 2 =<br />

2 tan β<br />

1 − tan 2 β<br />

= tan 2β<br />

olacaktır. Bu şekilde devam edersek, x n = tan 2 n−1 β olacağından x 8 =<br />

tan 2 7 β = tan 128β olacaktır. 8 terim uzunluğundaki bir dizi için, tan 128β =<br />

±1 <strong>olm</strong>alıdır. Buna göre<br />

olacaktır. Dolayısıyla,<br />

olacaktır.<br />

x = tan<br />

128β =<br />

(2k + 1)π<br />

4<br />

(<br />

)<br />

(2k + 1)π<br />

± , k = −128, · · · , 128<br />

512<br />

11. Eğer b 1 = tan −1 a 1 ve b k+1 = b k + tan −1 (1/k), k = 1, 2, 3, · · · olacak<br />

biçimde bir dizi tanımlarsak tanjant toplam formülünden<br />

tan(x + y) =<br />

tan x + tan y<br />

1 − tan x tan y<br />

eşitliğinin ışığında a k = tan b k olduğunu görebiliriz.<br />

tan x<br />

lim = 1<br />

x→0 x


86 BÖLÜM 2. KONULAR<br />

olduğuna göre,<br />

olacaktır. Buradan da,<br />

tan −1 (1/k)<br />

lim<br />

= 1<br />

k→∞ 1/k<br />

b 0 +<br />

∞∑<br />

tan −1 1 k<br />

k=1<br />

ıraksak olacaktır. Öte yandan serinin elemanları k → ∞ iken, sıfıra yakınsayacaktır.<br />

(2πn, 2πn + π/2) aralığında sonsuz çoklukta parçalı toplam<br />

olacaktır. Benzer biçimde (2πn + π/2, (2n + 1)π) aralığında da sonsuz<br />

çoklukta toplam olacaktır. Fakat, parçalı toplamlardam birisi b m ve<br />

a m = tan b m olduğundan, demek ki sonsuz çoklukta pozitif ve negatif<br />

a m değeri bulunmaktadır.<br />

13. Her bir a i değerimiz [−1, 1] aralığında olduğuna göre, a i = cos x i , x i ∈<br />

[0, π] şeklinde seçim yapabiliriz. Buna göre sorudaki eşitsizliğin sol tarafı<br />

n−1<br />

∑ √<br />

1 − cos xi cos x i+1 − sin x i sin x i+1 =<br />

n−1<br />

∑ √<br />

1 − cos (xi+1 − x i )<br />

i=1<br />

i=1<br />

= √ n−1<br />

∑<br />

2 sin x i+1 − x i<br />

2<br />

i=1<br />

olacaktır.<br />

Sinüs fonksiyonu [0, π] aralığında aşağı konkav olduğuna göre Jensen<br />

Eşitsizliğinden<br />

n−1<br />

1 ∑<br />

( ) (<br />

)<br />

n−1<br />

xi+1 − x i<br />

1 ∑ x i+1 − x i<br />

sin<br />

sin<br />

n − 1<br />

2<br />

n − 1 2<br />

i=1<br />

eşitsizliği elde edilir. Buna göre,<br />

n−1<br />

√ ∑<br />

( )<br />

xi+1 − x i<br />

2 sin<br />

2<br />

i=1<br />

i=1<br />

(n − 1) √ 2 sin x n − x n−1<br />

2 (n − 1) √ 2 (n − 1) sin<br />

olacaktır. Burada x n − x i ∈ (0, π) dir. Aynı aralıkta, sin x < x, x > 0<br />

eşitsizliği kullanılırsa<br />

( )<br />

√ π<br />

2 (n − 1) sin √ 2 π 2 (n − 1) 2<br />

π<br />

2 (n − 1)


2.9. TRİGONOMETRİK DEĞİŞKEN DEĞİŞTİRME 87<br />

olacaktır.<br />

13. Tüm x i değerleri pozitif ve toplamlarıda 1 olduğuna göre,<br />

x 1 + x 2 + · · · + x k = sin a k<br />

ve<br />

a 0 = 0 < a 1 < a 2 < · · · < a n = π , k = 0, 1, · · · , n<br />

2<br />

seçimlerini yapabiliriz. Buna göre,<br />

n∑<br />

k=1<br />

k=1<br />

sin a k − sin a k−1<br />

√ 1 + sin ak−1<br />

√ 1 − sin ak−1<br />

< π 2<br />

olacaktır. Bu eşitsizliği yeniden düzenlersek<br />

( ) ( )<br />

ak −a<br />

n∑ 2 sin k−1 ak +a<br />

2<br />

cos k−1<br />

2<br />

cos (a k − 1)<br />

olacaktır. 0 < x < π/2 aralığında kosinüs fonksiyonu azalandır ve sin x <<br />

x eşitsizliği vardır. Buna göre eşitsizliğin sol tarafı<br />

ise<br />

n∑<br />

k=1<br />

n∑<br />

2 a k−a k−1<br />

2<br />

cos a k−1<br />

cos a k−1<br />

k=1<br />

(a k − a k−1 ) = π 2<br />

olacaktır ki bu da zaten ispatı tamamlar.<br />

14. Denklemi sağlayan üçlülerden herbiri dar açılı bir üçgenin açılarının kosinüs<br />

değerleridir. Önce, eğer A, B, C açıları bir üçgenin açı değerleri<br />

ise<br />

cos 2 A + cos 2 B + cos 2 C + 2 cos A cos B cos C = 1<br />

olduğunu gösterelim. Aslında,<br />

cos A = − cos (B + C) = sin B sin C − cos B cos C


88 BÖLÜM 2. KONULAR<br />

olacağına göre<br />

= cos 2 A + cos 2 B + cos 2 C + 2 cos A cos B cos C<br />

= (cos A + cos B + cos C) 2 + 1 − ( 1 − cos 2 A ) ( 1 − cos 2 C )<br />

= (sin B sin C) 2 + 1 − sin 2 B sin 2 C = 1<br />

olarak bulunur. Demek ki, A + B + C = π eşitliğini sağlayan A, B, C<br />

değerleri birer çözümdür.<br />

15.<br />

İkinci denklemi düzenlersek<br />

olacaktır. Buna göre,<br />

x 1 =<br />

a 2<br />

yz + b2<br />

zx + c2<br />

xy + abc<br />

xyz = 4<br />

a √ yz<br />

, y 1 =<br />

b √ zx<br />

, z 1 =<br />

c √ xy<br />

olarak alırsak, eşitliğimiz<br />

x 2 1 + y 2 1 + z 2 1 + x 1 y 1 z 1 = 4<br />

olacağından 0 < x 1 < 2, 0 < y 1 < 2, 0 < z 1 < 2 olacaktır. Buna göre,<br />

x 1 = 2 cos A, y 1 = 2 cos B, z 1 = 2 cos C<br />

olarak alabiliriz. Bu değerleri kullanırsak,<br />

2 √ yz cos A = a, 2 √ zx cos B = b, 2 √ xy cos C<br />

olacaktır. Eğer x + y + z = a + b + c ise<br />

x + y + z − 2 √ yx cos A − √ 2zx cos B − 2 √ xy cos C = 0<br />

olacaktır. Bu eşitliğin sol tarafını iki kare toplamı şeklinde yazmaya çalışırsak<br />

= x + y + z − 2 √ yz cos A − 2 √ zx cos B − 2 √ xy cos C<br />

= x + y + z − 2 √ yz cos A − 2 √ zx cos B + 2 √ xy (cos A cos B − sin A sin B)<br />

= x ( sin 2 B + cos B 2) + y ( sin 2 A + cos 2 A ) + z − 2 √ yz cos A<br />

− 2 √ zx cos B + 2 √ xy cos A cos B − 2 √ xy sin A sin B<br />

= (√ x sin B − √ y sin A ) 2 +<br />

(√ x cos B +<br />

√ y cos A −<br />

√ z<br />

)


2.9. TRİGONOMETRİK DEĞİŞKEN DEĞİŞTİRME 89<br />

olacaktır. Burada iki kare toplamı sıfır olduğuna göre<br />

√ x sin B −<br />

√ y sin A = 0<br />

ve<br />

√ x cos B +<br />

√ y cos A =<br />

√ z<br />

olacaktır. Buradan,<br />

√ √ b z = x<br />

2 √ zx + √ a<br />

y<br />

2 √ yz = b + a<br />

2 √ z<br />

olacaktır. Demek ki<br />

’dir. Simetriden dolayı<br />

z = a + b<br />

2<br />

olarak bulunur.<br />

y = a + c<br />

2<br />

ve x = b + c<br />

2<br />

16. Önce bildiğimiz bazı trigonometrik soruda kullanabileceğimiz bazı geometrik<br />

eşitlikleri yazalım. Buna göre,<br />

ve<br />

cos 2θ = 2 cos 2 θ − 1<br />

cos 3θ = 4 cos 3 θ − 3 cos θ<br />

eşitliklerini kullanmamız yerinde olacaktır. Eğer x = 2π/5 alırsak, cos 6π/5 =<br />

cos 4π/5 olacaktır. Şimdi denklemi x değişkenine bağımlı olarak yazalım.<br />

Burada<br />

4x 3 − 2x 2 − 3x + 1 = 0<br />

ise<br />

olacaktır. Buradan<br />

(x − 1) ( 4x 2 + 2x − 1 ) = 0<br />

x = 1 veya x =<br />

√<br />

5 − 1<br />

2


90 BÖLÜM 2. KONULAR<br />

olacağından cos(2π/5) ≠ 1 ise istenilen cevap<br />

olacaktır.<br />

( ) √<br />

2π 5 − 1<br />

cos =<br />

5 2<br />

2.10 Ceb˙ırde Teleskop˙ık Toplamlar ve Çarpımlar<br />

Bu konuda da cebirde sonsuz toplamlar ve farklarla alakalı soruları çözeceğiz.<br />

Aslında çözümlerde kullanacağımız genel yöntem oldukça basittir. Soruda<br />

verilen toplamı yada çarpımı, farkların toplamı biçiminde yazarak sonuca ulaşmaya<br />

çalışacağız. Buna göre elde edeceğiz farkların toplamı<br />

n∑<br />

[F (k) − F (k − 1)]<br />

k=2<br />

formunda olacağından ve 2 ≤ k ≤ n − 1 indislerine sahip değerler sadeleşeceği<br />

için, toplamımız<br />

F (n) − F (1)<br />

olacaktır.<br />

Örnek.<br />

toplamını hesaplayınız.<br />

n∑<br />

k!k<br />

k=1<br />

Çözüm.<br />

yaparsak,<br />

olacaktır.<br />

Eğer k! · k = k!(k + 1 − 1) = (k + 1)! − k! değişken değiştirmesini<br />

n∑<br />

(k + 1)! − k! = (n + 1)! − 1<br />

k=1


2.10. CEBİRDE TELESKOPİK TOPLAMLAR VE ÇARPIMLAR 91<br />

Örnek.<br />

toplamını hesaplayınız.<br />

n∑ 1<br />

k=1<br />

(k + 1) √ k + k √ k + 1<br />

Çözüm.<br />

Temel düşünce paydayı rasyonel yapmak <strong>olm</strong>alıdır. Yani,<br />

(<br />

(k + 1) √ k − k √ ) (<br />

k + 1 (k + 1) √ k + k √ )<br />

k + 1<br />

olduğuna göre,<br />

= k (k + 1) 2 − (k + 1) k 2<br />

= k(k + 1)<br />

n∑<br />

k=1<br />

(k + 1) √ k − k √ k + 1<br />

k (k + 1)<br />

=<br />

n∑<br />

k=1<br />

( 1 √k −<br />

1<br />

√<br />

k + 1<br />

)<br />

= 1 −<br />

1<br />

√ n + 1<br />

olacaktır.<br />

Örnek.<br />

n ∈ Z + <strong>olm</strong>ak üzere aşağıda verilen eşitsizliği kanıtlayınız.<br />

n − 1 <<br />

1<br />

√<br />

1 +<br />

√<br />

2<br />

+<br />

1<br />

√<br />

2 +<br />

√<br />

5<br />

+ · · · +<br />

2n − 1<br />

√<br />

(n − 1) 2 + 1 + √ n 2 + 1<br />

< n<br />

Çözüm. Soruda verilen eşitsizliği iki yönünüde ispatlamadan evvel, bazı<br />

genellemeler yapalım. Buna göre,<br />

ise<br />

2k − 1<br />

√<br />

(k − 1) 2 + 1 + √ k 2 + 1<br />

( √k<br />

√<br />

)<br />

(2k − 1) 2 + 1 − (k − 1) 2 + 1<br />

=<br />

k 2 + 1 − (k − 1) 2 − 1<br />

= √ √<br />

k 2 + 1 − (k − 1) 2 + 1


92 BÖLÜM 2. KONULAR<br />

1 3<br />

2n − 1<br />

= √ √ + √ √ + · · · + √<br />

1 + 2 2 + 5<br />

(n − 1) 2 + 1 + √ n 2 + 1<br />

= √ 2 − 1 + √ 5 − √ 2 + · · · + √ √<br />

n 2 + 1 − (n − 1) 2 + 1<br />

= √ n 2 + 1 − 1<br />

olacaktır.<br />

Buna göre, yeni eşitsizliğimiz<br />

n − 1 < √ n 2 + 1 < n<br />

olacaktır ki, bu eşitsizliği göstermek oldukça kolaydır.<br />

Örnek.<br />

x, Q(x) polinomunun bir kökü değildir. Buna göre,<br />

P (x) P (x + 1)<br />

−<br />

Q (x) Q (x + 1) = 1<br />

x (x + 2)<br />

eşitliğini sağlayan tüm P ve Q polinomlarını bulunuz.<br />

Çözüm.<br />

Önce soruda verilen eşitliği düzenleyelim, buna göre<br />

P (x) P (x + 1)<br />

−<br />

Q (x) Q (x + 1) = 2x + 1 2 (x + 1) + 1<br />

−<br />

2x (x + 1) 2 (x + 1) (x + 2)<br />

eşitliği elde edilir. Buna göre, n ∈ Z + ve yeterince büyük bir x değeri için,<br />

P (x + k) (x + k + 1)<br />

−P<br />

Q (x + k) Q (x + k + 1) = 2 (x + k) + 1<br />

2 (x + k + 1) (x + k + 2) − 2 (x + k + 1) + 1<br />

2 (x + k + 1) (x + k + 2)<br />

olacaktır. Eğer eşitlikleri taraf tarafa toplarsak,<br />

P (x) P (x + n)<br />

−<br />

Q (x) Q (x + n) = 2x + 1<br />

2x (x + 1) − 2 (x + n) + 1<br />

2 (x + n) (x + n + 1)<br />

eşitliğini elde ederiz. Burada n ⇒ ∞ iken, P (x+n)/Q(x+n) ifadeside sonsuza<br />

gidecektir. Buna göre,<br />

P (x)<br />

Q (x) = 2x + 1<br />

2x (x + 1) + c, c ∈ R


2.10. CEBİRDE TELESKOPİK TOPLAMLAR VE ÇARPIMLAR 93<br />

olacaktır. Eğer işlemi devam ettirirsek,<br />

2x (x + 1) P (x) = (2x + 1 + 2cx (x + 1)) Q (x)<br />

eşitliğini elde ederiz. Buradan istenen polinomlar<br />

P (x) = (2x + 1 + 2cx (x + 1)) R (x) ve Q (x) = 2x (x + 1) R (x)<br />

olarak bulunur.<br />

Örnek.<br />

eşitliğini kanıtlayınız.<br />

∞∏<br />

(1 − 1 )<br />

n 2<br />

n=2<br />

= 1 2<br />

Çözüm.<br />

N∏<br />

lim<br />

(1 − 1 )<br />

n 2 = lim<br />

N→∞<br />

n=2<br />

olduğuna göre,<br />

N∏<br />

N→∞<br />

n=2<br />

2.10.1 Çalışma Soruları<br />

(<br />

1 − 1 ) (<br />

1 + 1 )<br />

n n<br />

N + 1<br />

lim<br />

N→∞ 2N<br />

= 1 2 bulunur.<br />

=<br />

N∏<br />

n=2<br />

n − 1<br />

n<br />

= 1 N · N + 1<br />

2<br />

N∏<br />

n=2<br />

n + 1<br />

n<br />

= N + 1<br />

2N<br />

1.<br />

toplamını hesaplayınız.<br />

n∑<br />

k! ( k 2 + k + 1 )<br />

k=1<br />

2. a 1 , a 2 , · · · , a n bir aritmetik dizinin elemanları ve ortak farkları d ise<br />

n∑ 1<br />

a k · a k+1<br />

toplamını hesaplayınız.<br />

k=1


94 BÖLÜM 2. KONULAR<br />

3.<br />

toplamını hesaplayınız.<br />

∞∑<br />

k=1<br />

6 k<br />

(3 k − 2 k ) (3 k+1 − 2 k+1 )<br />

4. {x n } ∞ n=1 dizisinde x 1 = 1/2 , x k+1 = x 2 k + x k ise<br />

1<br />

x 1 + 1 + 1<br />

x 2 + 1 + · · · + 1<br />

x 100 + 1<br />

toplamından küçük en büyük tamsayı değerini bulunuz.<br />

5. F n bir Fibonacci dizisi <strong>olm</strong>ak üzere F 1 = 1, F 2 = 1, F n+1 = F n + F n−1<br />

olarak veriliyor. Buna göre aşağıdaki toplamları hesaplayınız.<br />

a.<br />

b.<br />

∞∑<br />

n=2<br />

F n<br />

F n−1 · F n+1<br />

∞∑ 1<br />

F n−1 · F n+1<br />

n=2<br />

6. √1 + 1 1 2 + 1 2 2 + √1 + 1 2 2 + 1 3 2 + · · · + √<br />

toplamını hesaplayınız.<br />

1 + 1<br />

1999 2 + 1<br />

2000 2<br />

7.<br />

1<br />

√<br />

1 +<br />

√<br />

3<br />

+<br />

1<br />

√<br />

5 +<br />

√<br />

7<br />

+ · · · +<br />

1<br />

√<br />

1997 +<br />

√<br />

9999<br />

> 24<br />

eşitsizliğini kanıtlayınız.<br />

8. 1 m < n m, n ∈ Z olduğuna göre, aşağıdaki eşitsizliği kanıtlayınız.<br />

2 (√ n + 1 − √ m ) < √ 1 1<br />

+ √ +· · ·+ √ 1 + √ 1 < 2 (√ n − √ m − 1 )<br />

m m + 1 n − 1 n


2.10. CEBİRDE TELESKOPİK TOPLAMLAR VE ÇARPIMLAR 95<br />

9.<br />

10.<br />

a k =<br />

ise<br />

eşitsizliğini kanıtlayınız.<br />

eşitsizliğini kanıtlayınız.<br />

k<br />

(k − 1) 3 + k 4/3 + (k + 1) 4/3<br />

∞∑<br />

n=1<br />

∑999<br />

a k < 50<br />

k=1<br />

1<br />

(n + 1) √ n < 2<br />

11. F n , fibonacci serisi <strong>olm</strong>ak üzere verilen,<br />

∞∑ 1<br />

toplamının eşitini bulunuz.<br />

F<br />

n=0 2 n<br />

12.<br />

13.<br />

eşitliğini kanıtlayınız.<br />

çarpımını hesaplayınız.<br />

∞∏<br />

n=2<br />

∞∑<br />

n=0<br />

n 3 − 1<br />

n 3 + 1 = 2 3<br />

(<br />

1 + 1<br />

2 2n )<br />

14. L 1 = 2, L 2 = 2 ve L n+2 = L n+1 + L n , n 1 olarak tanımlanan Lucas<br />

Dizisi veriliyor. Buna göre,<br />

m∏<br />

L 2 k +1 = F 2 m+1<br />

eşitliğini kanıtlayınız.<br />

k=1


96 BÖLÜM 2. KONULAR<br />

Édouard Lucas 12 (1842-1891)<br />

Fransız <strong>mat</strong>e<strong>mat</strong>ikçi ve bilimadamı olan Lucas<br />

daha çok kendi adı ile anılan Lucas Dizileri ve Fibonacci<br />

Dizisinin n. terimini veren formülü ile anılır.<br />

Üniversite eğitimini École Normale Supérieure’de<br />

tamamlayan Lucas uzun bir süre Paris Gözlemevinde’de<br />

çalışmalarını sürdürmüş ve Profesörlük ünvanınıda<br />

yine aynı şehirde Paris’te almıştır. Ayrıca<br />

N∑<br />

n 2 = M 2<br />

n=1<br />

Diphantine denkleminin yegane çözümünün N = 24<br />

ve M = 70 ikilisi olduğunu savunmuştur. Bu ikiliden<br />

başka ikililerin <strong>olm</strong>adığı ise ancak 1918 yılında<br />

hiperbolik fonksiyonlar kullanılarak kanıtlanabilmiştir.<br />

Ayrıca, sayıların asallığı üzerinede çalışmalar yapan<br />

Lucas 19 yıllık bir çalışma sonucunda, 2 127 − 1 sayısının asal olduğunu<br />

kanıtlamış ve bu kanıtı sonucunda ulaştığı sayı o zamana kadar ulaşılabilen<br />

en büyük Mersenne Asalı olarak kabul edilmiştir. Mate<strong>mat</strong>iğin eğlenceli tarafıylada<br />

alakadar olan Lucas, özellikle Hanoi Kuleleri’ni keşfetmesi ve çözüm<br />

yöntemleri geliştirmesi bulmaca meraklılarının büyük saygısını kazanmıştır.<br />

2.10.2 Çözümler<br />

1.<br />

n∑<br />

k! ( k 2 + k + 1 ) =<br />

k=1<br />

=<br />

n∑<br />

k=1<br />

[<br />

]<br />

(k + 1) 2 − k k!<br />

n∑<br />

[(k + 1)! (k + 1) − k!k]<br />

k=1<br />

= (n + 1)! (n + 1) − 1 olacaktır.<br />

12 François Édouard Anatole Lucas


2.10. CEBİRDE TELESKOPİK TOPLAMLAR VE ÇARPIMLAR 97<br />

2.<br />

n∑<br />

k=1<br />

1<br />

a k a k+1<br />

= 1 d<br />

n∑<br />

k=1<br />

a k+1 − a k<br />

a k a k+1<br />

= 1 d<br />

olacağından, toplam<br />

1 a n+1 − a 1<br />

d a n+1 a 1<br />

olacaktır. a n+1 − a 1 = nd olduğuna göre<br />

olacaktır.<br />

n∑<br />

k=1<br />

1 nd<br />

n<br />

=<br />

d (nd + a 1 ) a 1 (nd + a 1 ) a 1<br />

( 1<br />

a k<br />

− 1<br />

a k+1<br />

)<br />

3. Kesri parçalara bölerek çözüme gitmek yerinde olacaktır. Buna göre,<br />

6 k<br />

(3 k − 2 k ) (3 k+1 − 2 k+1 ) = A<br />

3 k − 2 k − B<br />

3 k+1 − 2 k+1<br />

eşitliğinde (<br />

3 k+1 − 2 k+1) (<br />

A − 3 k − 2 k) B = 6 k<br />

olacaktır. Buradan A = 2 k ve B = 2 k + 1 olarak kolaylıkla bulunabilir.<br />

Buna göre,<br />

∞∑<br />

k=1<br />

6 k<br />

(3 k − 2 k ) (3 k+1 − 2 k+1 ) = 2<br />

3 − 2 − lim 2 k+1<br />

k→∞ 3 k+1 − 2 k+1 = 2<br />

olarak bulunur.<br />

Not. Benzer biçimde A = 3 k ve B k + 1 bulunabilirdi, ancak bu değerler<br />

sonucu değiştirmezdi. Deneyip görebilirsiniz.<br />

4. İndirgemeli dizi çözüm yöntemini kullanalım. Buna göre, x k+1 = x 2 k +x k<br />

ise<br />

1 1<br />

=<br />

x k+1 x k (x k + 1) = 1 − 1<br />

x k x k + 1<br />

olacağından<br />

1<br />

x 1 + 1 + 1<br />

x 2 + 1 + · · · + 1<br />

x 100 + 1 = 1 x 1<br />

− 1<br />

x 101


98 BÖLÜM 2. KONULAR<br />

olacaktır. x 1 = 1/2 ve 0 < 1/x 101 < 1 olduğuna göre istenen değer 1<br />

olacaktır.<br />

5. a.<br />

∞∑<br />

n=2<br />

F n<br />

F n−1 F n+1<br />

=<br />

∞∑ F n+2 − F n−1<br />

=<br />

F n−1 F n+1<br />

n=2<br />

= lim<br />

n→∞<br />

∞∑<br />

n=2<br />

( 1<br />

F n−1<br />

− 1<br />

F n+1<br />

)<br />

( 1<br />

F 1<br />

+ 1 F 2<br />

− 1<br />

F n<br />

− 1<br />

F n+1<br />

)<br />

= 1 F 1<br />

+ 1 F 2<br />

= 2 olacaktır.<br />

b.<br />

∞∑<br />

n=2<br />

olacaktır.<br />

1<br />

F n−1 F n+1<br />

=<br />

6. n ∈ Z + <strong>olm</strong>ak üzere,<br />

ise<br />

=<br />

∞∑<br />

= lim<br />

n→∞<br />

F n<br />

=<br />

F<br />

n=2 n−1 F n F n+1<br />

∞∑<br />

(<br />

1<br />

−<br />

F<br />

n=2 n−1 F n<br />

( 1<br />

−<br />

F 1 F 2<br />

∞∑<br />

n=2<br />

)<br />

1<br />

F n F n+1<br />

1 + 1 n 2 + 1<br />

(n + 1) 2 = n2 (n + 1) 2 + (n + 1) 2 + n 2<br />

n 2 (n + 1) 2 =<br />

F n+1 − F n−1<br />

F n−1 F n F n+1<br />

)<br />

1<br />

= 1 = 1<br />

F n F n+1 F 1 F 2<br />

(<br />

n 2 + n + 1 )<br />

n 2 (n + 1) 2<br />

√<br />

1 + 1 n 2 + 1<br />

(n + 1) 2 = n2 + n + 1 1<br />

n 2 = 1 +<br />

+ n n (n + 1)<br />

olacaktır. Buna göre,<br />

olur.<br />

1997<br />

∑<br />

n=1<br />

(<br />

1 +<br />

)<br />

1<br />

=<br />

n (n + 1)<br />

1999<br />

∑<br />

n=1<br />

(<br />

1 + 1 n − 1 )<br />

= 2000 − 1<br />

n + 1<br />

2000


2.10. CEBİRDE TELESKOPİK TOPLAMLAR VE ÇARPIMLAR 99<br />

7. Teleskopik toplam yapmak mantıklı görünsede, bazı elemanlar eksik olduğundan<br />

çözüme gidemeyiz. Buna göre eşitsizliğin sol kısmı,<br />

1<br />

√<br />

3 +<br />

√<br />

5<br />

+<br />

1<br />

√<br />

7 +<br />

√<br />

9<br />

+ · · · +<br />

1<br />

√<br />

9999 +<br />

√<br />

10001<br />

toplamından küçüktür. Eğer sorudaki eşitsizlikle son yazdığımız eşitsizliği<br />

toplarsak<br />

1<br />

√<br />

1 +<br />

√<br />

3<br />

+<br />

1<br />

√<br />

3 +<br />

√<br />

5<br />

+<br />

1<br />

√<br />

5 +<br />

√<br />

7<br />

+ · · · +<br />

1<br />

√<br />

9999 +<br />

√<br />

10001<br />

> 48<br />

olacaktır. Şimdi artık teleskopik toplamı kullanabilriz. Buna göre,<br />

1<br />

(√ √ √ √ √ √ √ √ )<br />

3 − 1 + 5 − 3 + 7 − 5 + · · · + 10001 − 9999 = 1 (√ )<br />

10001 − 1 ><br />

2<br />

2<br />

olacaktır.<br />

8. k ∈ R için (√ k + 1 − k ) (√ k + 1 + k ) = 1 olduğuna göre,<br />

( √k √ )<br />

2 + 1 − k =<br />

2<br />

√<br />

k + 1 +<br />

√<br />

k<br />

< 1 k<br />

ve<br />

1<br />

√<br />

k<br />

<<br />

2<br />

(√ √ )<br />

√ √ = 2 k − k − 1<br />

k + k − 1<br />

olacaktır. Buna göre,<br />

( √k √ )<br />

2 + 1 − k < √ 1 (√ √ )<br />

< 2 k − k − 1<br />

k<br />

olacaktır. Eğer bu eşitsizlikleri toplarsak,<br />

2 (√ n + 1 − √ m ) < √ 1 1<br />

+ √ +· · ·+ √ 1 + √ 1 < 2 (√ n − √ m − 1 )<br />

m m + 1 n − 1 n<br />

olacağından soruda verilen eşitsizlik kanıtlanmış olur.


100 BÖLÜM 2. KONULAR<br />

9. ilk önce a n ifadesinin paydasını küçültelim ve rasyonel hale getirelim.<br />

Buna göre, k 3 yerine (k − 1) 2/3 (k + 1) 2/3 yazılırsa,<br />

a k <<br />

=<br />

k<br />

(k − 1) 4/3 + (k − 1) 2/3 (k + 1) 2/3 + (k + 1) 4/3<br />

k<br />

((k + 1) 2/3 − (k − 1) 2/3)<br />

(k + 1) 2 − (k − 1) 2 = 1 (<br />

(k + 1) 2/3 − (k − 1) 2/3) olacağından<br />

4<br />

∑999<br />

a n = 1 ∑999<br />

((k + 1) 2/3 − (k − 1) 2/3)<br />

4<br />

k=1<br />

k=1<br />

= 1 (<br />

1000 2/3 + 999 2/3 − 1 2/3 − 0 2/3) < 1 (100 + 100 − 1) < 50 bulunur.<br />

4<br />

4<br />

10. Verilen toplamı düzenlersek<br />

1<br />

√ n (n + 1)<br />

=<br />

olacağından yeni toplamımız<br />

1 +<br />

√ n<br />

n (n + 1) = √ n<br />

n −<br />

∞∑<br />

√ √ n − n − 1<br />

n=2<br />

n<br />

√ n<br />

n + 1<br />

olacaktır. Bu toplam teleskopik değildir, ancak üstten sınırlıdır. Öyleki<br />

1 +<br />

∞∑<br />

n=2<br />

√ n −<br />

√ n − 1<br />

√ n<br />

√ n − 1<br />

= 1 +<br />

∞∑<br />

k=2<br />

( 1<br />

√ − 1 )<br />

√ n − 1 n<br />

olacakır. Ve bu toplam da 2’ye yakınsar. İspat tamamlanır.<br />

11. Sorunun çözümüne tümevarım ile başlayalım. Buna göre, tümevarım<br />

yöntemi kullanılarak<br />

F 2m F m−1 − F 2m−1 F m = (−1) m F m , m 1


2.10. CEBİRDE TELESKOPİK TOPLAMLAR VE ÇARPIMLAR 101<br />

olduğu bulunabilir. Eğer m = 2 n−1 alınırsa<br />

veya<br />

olacağından<br />

∞∑ 1<br />

F<br />

n=0 2 n<br />

olacaktır.<br />

F 2 nF 2 n−1 −1 − F 2 n −1F 2 n−1 = F 2 n−1, n 2<br />

1<br />

F 2 n<br />

= F 2 n−1 −1<br />

F 2 n−1<br />

= 1 F 1<br />

+ 1 F 2<br />

+ lim<br />

N→∞<br />

− F 2 n −1<br />

, n 2<br />

F 2 n<br />

(<br />

F1<br />

− F )<br />

2 N −1<br />

F 2 F 2 N<br />

= 3 − 1<br />

√<br />

5+1<br />

2<br />

= 7 − √ 5<br />

2<br />

12.<br />

lim<br />

N∏<br />

N→∞<br />

n=2<br />

olarak bulunur.<br />

n 3 − 1<br />

n 3 + 1<br />

= lim<br />

N∏<br />

N→∞<br />

n=2<br />

= lim<br />

N→∞<br />

n=2<br />

(n − 1) ( n 2 + n + 1 )<br />

(n + 1) (n 2 − n + 1)<br />

N∏ n − 1<br />

n + 1<br />

1 · 2 ·<br />

= lim<br />

N→∞<br />

N∏ (n + 1) 2 − (n + 1) + 1<br />

n 2 − n + 1<br />

n=2<br />

(<br />

)<br />

(N + 1) 2 − (N + 1) 4 + 1<br />

3N · (N + 1)<br />

= 2 3<br />

13. Eğer soruda verilen ifadeyi açarsak<br />

∞∏<br />

(<br />

1 + 1 )<br />

= 2 lim<br />

2 2n N→∞<br />

n=0<br />

(<br />

1 − 1 2 0 ) ∞<br />

∏<br />

n=0<br />

olacaktır. Buradan,<br />

(<br />

1 − 1 ) (<br />

1 + 1 ) (<br />

= 1 − 1 )<br />

2 2n 2 2n 2 2n+1<br />

(<br />

1 + 1<br />

2 2n )<br />

olacağından, çarpımımızı teleskopik hale getirebiliriz. Buradan çarpım<br />

1 − 1<br />

2 2N+1<br />

olacaktır. N → ∞ ve 1− 1<br />

2 2N+1 → 1 ise çarpımın sonucu 2 olarak bulunur.


102 BÖLÜM 2. KONULAR<br />

14. Eğer kontrol edilirse Fibonacci ve Lucas dizilerinin<br />

F n+1 + F n−1 = L n+1 ve F 2n = F 2 n+1 − F 2 n−1, n 1<br />

için sağlandığı görülebilir. Öyleyse,<br />

F 2n = (F n+1 + F n−1 ) (F n+1 − F n−1 ) = L n+1 F n<br />

olacaktır. Buna göre,<br />

olacaktır. Öyleyse,<br />

L n+1 = F 2n<br />

F n<br />

, n 1<br />

m∏<br />

L 2 k +1 =<br />

k=1<br />

m∏<br />

k=1<br />

F 2 k+1<br />

F 2 k<br />

= F 2 m+1<br />

F 2<br />

= F 2 m+1<br />

olacaktır. İspat tamamlanır.<br />

2.11 Tamdeğer Fonks˙ıyon Problemler˙ı<br />

x ∈ R <strong>olm</strong>ak üzere, x değerini geçmeyen en büyük tamsayı değeri [x] ile<br />

gösterilir. Bu foksiyona tam değer fonsiyon (floor function) denir. Buna göre<br />

x − 1 < [x] ≤ x olduğu açıktır. Bu ifade ayrıca [x] ≤ x < [x] + 1 olarakta<br />

yazılabilir. Ayrıca sayının ondalıklı kısmıda {x} ile temsil edilir. Buna göre;<br />

{x} = x − [x] dir. Buna göre x = [x] + {x}, 0 ≤ {x} < 1 dir. Tam değer<br />

fonksiyonun en temel teoremleri aşağıda verilmiştir.<br />

Teorem. α, β ∈ R, a ∈ Z, n ∈ N ise;<br />

1. [α + a] = [α] + a<br />

2. [ α n ] = [ [α]<br />

n ]<br />

3. [α] + [β] ≤ [α + β] ≤ [α] + [β] + 1<br />

Kanıt.


2.11.<br />

TAMDEĞER FONKSİYON PROBLEMLERİ 103<br />

1. Varsayalım m = [α + a] olsun. Buna göre m ≤ α + a < m + 1 dolayısıyla<br />

da m − a ≤ α < m − a + 1 olacaktır. m − a = [α] olduğuna göre ispat<br />

tamamlanmış olur.<br />

α<br />

2.<br />

n ifadesini α n = [ α n +θ], 0 ≤ θ < 1 olarak yazalım. n·[α n<br />

] ifadesi olduğnan<br />

artık elimizde 1 durumu oluştu. Yani;<br />

[α] = [n[α/n] + nθ] = n[α/n] + [nθ] olur.<br />

Şimdi, 0 ≤ [nθ] < nθ < n ve bundan dolayı 0 ≤ [nθ/n] < 1 olur.Eğer<br />

Θ = [nθ]/n olduğunu varsayarsak [α]<br />

n<br />

= [ α n<br />

] + Θ, 0 ≤ Θ < 1 olur ki, buda<br />

zaten istenen sonuçtur.<br />

3. Varsayalım α = n + r, β = m + t ve 0 ≤ r < 1, 0 ≤ t < 1 olsun.<br />

[α] + [β] = n + m ≤ [n + r + m + t] = [α + β] olur.<br />

= m + n + [r + t] ≤ n + m + 1<br />

= [α] + [β] + 1 olur.<br />

Örnek.<br />

bulunuz.<br />

[ x 3 ] = x 2<br />

+ 1 denklemini sağlayan x tamsayılarının toplamını<br />

Çözüm. Varsayalım [ x 3 ] = t olsun. Buna göre, t ≤ x 3<br />

< t + 1 ise 3t ≤<br />

x < 3t + 3 <strong>olm</strong>alıdır. [ x 3 ] = t = x 2<br />

+ 1 olduğuna göre x = 2t − 2 olur. Bu<br />

son eşitliği eşitsizliğimizde yerine koyarsak 3t ≤ 2t − 2 < 3t + 3 ise −5 < t ≤<br />

−2 bulunur. t = −2, −3, −4 olabileceğinden x değerleride −8, −10, −6 olarak<br />

bulunur. istenen cevap −24 olur.<br />

Teorem.(Hermite Özdeşliği) [nx] = [x]+[x+ 1 n ]+[x+ 2 n<br />

dir.<br />

Kanıt. Eğer teoremde verilen eşitliği düzenlersek;<br />

n∑<br />

[x + k n ] = [nx]<br />

k=0<br />

]+· . . .·+[x+<br />

n−1<br />

n ]


104 BÖLÜM 2. KONULAR<br />

ifadesini elde ederiz. Öyleyse bu eşitliği ispatlamamız yeterlidir. x = [x] + {x}<br />

olduğunu zaten biliyoruz. Buna göre; {1, 2, 3, . . . , n} kümesinin elemanı olan<br />

kesin bir k ′ değeri vardır ki;<br />

ise<br />

[x] = [x + k′ − 1<br />

k′<br />

] ≤ x < [x +<br />

n n ] = [x] + 1<br />

0 = [{x} + k′ − 1<br />

k′<br />

] ≤ {x} < [{x} +<br />

n n ] = 1<br />

olacaktır. Buna göre;<br />

1 − k′<br />

n ≤ {x} < 1 − k′ − 1<br />

⇒ n − k ′ ≤ n{x} < n − k ′ + 1 olur.<br />

n<br />

En başta yazdığımız eşitliği kullanırsak<br />

n−1<br />

∑<br />

[x + k k ′ −1<br />

n ] = ∑<br />

[x] +<br />

k=0<br />

k=0<br />

n−1<br />

∑<br />

k=k ′ ([x] + 1) = n · [x] + n − k ′<br />

= n[x] + [n{x}]<br />

= [n[x] + n{x}] = [nx] olur.<br />

Örnek. [x]+[2x]+[4x]+[8x]+[16x]+[32x] = 12345 denklemini sağlayan<br />

x değerlerini bulunuz.<br />

Çözüm. Bu soruda amacımız önce, denkliği sağlayan x değerlerinin varlığını<br />

araştırmak ve bu değerleri bulmak olacaktır. Konunun en başında verdiğimiz<br />

teoremlerden biliyoruz ki, x − 1 < [x] ≤ x olacaktır. Buna göre;<br />

x−1+2x−1+4x−1+. . .+32x−1 < [x]+[2x]+. . .+[32x] ≤ x+2x+4x+. . .+32x<br />

ise 63x − 6 < 12345 ≤ 63x ve x değeride 195 < x < 196 arasında olacaktır.<br />

Eğer x değerini 2 lik tabanda yazarsak x = 195 + a 1<br />

2<br />

+ a 2<br />

+ . . ., a<br />

2 2 k = 0 veya 1.<br />

Buna göre;<br />

[2x] = 2 · 195 + a 1<br />

[4x] = 4 · 195 + 2a 1 + 2a 2<br />

[8x] = 8 · 195 + 4a 1 + 2a 2 + a 3


2.11.<br />

TAMDEĞER FONKSİYON PROBLEMLERİ 105<br />

[16x] = 16 · 195 + 8a 1 + 4a 2 + 2a 3 + a 4<br />

[32x] = 32 · 195 + 16a 1 + 8a 2 + 4a 3 + 2a 4 + a 5<br />

Eğer bu değerleri alt alta toplarsak 63·195+31·a 1 +15·a 2 +7a 3 +3·a 4 +a 5<br />

olacaktır yani; 31 · a 1 + 15 · a 2 + 7 · a 3 + 3 · a 4 + a 5 = 60 olur. Ancak 0 ≤ a i ≤ 1<br />

olduğundan ifade en fazla 31 + 15 + 7 + 3 + 1 = 57 değerini alabilir. Çelişki<br />

vardır. Çözümü sağlayan bir x değeri yoktur.<br />

Örnek. [x], x in tamdeğer foksiyonu <strong>olm</strong>ak üzere, {x} = x − [x] olarak<br />

tanımlansın. Her x reel sayısı için x = f(x) − 2 · f({x}) eşitliğini sağlayan f<br />

fonksiyonunun x = − 13 5<br />

noktasındaki değerini bulunuz.<br />

Çözüm. {− 13 5 } = − 13<br />

5 − (−3) = 2 5 oluğundan; x = 2 5 için<br />

2<br />

5 = f(2 5 ) − 2 · f({2 5 }) ⇒ f(2 5 ) = −2 5<br />

olacaktır. x = − 13 5<br />

için<br />

− 13 5 = f(−13 5 ) − 2 · f({−13 5 }) = f(−13 5 ) − 2 · f(2 5 )<br />

olduğuna göre<br />

olur.<br />

f(− 13<br />

5 ) = −17 5<br />

Teorem. a ve b aralarında asal doğal sayılar <strong>olm</strong>ak üzere;<br />

[ a b ] + [2a b ] + [3a b<br />

− 1) · b (a − 1) · (b − 1)<br />

] + . . . + [(a ] =<br />

b<br />

2<br />

eşitliği vardır.<br />

Kanıt. Aslında teoremde verilen ifadeyi düzenlersek;<br />

∑a−1<br />

[ kb b−1<br />

a ] = ∑<br />

[ ka (a − 1)(b − 1)<br />

] =<br />

b 2<br />

k=1<br />

k=1


106 BÖLÜM 2. KONULAR<br />

eşitliğini elde ederiz. Öyleyse bu eşitlikleri ispatlamamız yeterli olacaktır.Varsayalım<br />

analitik düzlem üzerinde, köşe koordinatları (0, 0), (0, b), (a, 0), (a, b) olan bir<br />

dikdörtgenimiz olsun. Bu dikdörtgen üzerinde (a − 1)(b − 1) tane latis (lattice)<br />

noktası, yani koordinat bileşnleri tamsayıar olan noktalar vardır. Bu dikdörtgeni<br />

y = xb<br />

a<br />

doğrusu ile iki parçaya ayıralım. Bu doğru üzerindeki başlangıç ve<br />

bitiş noktaları hariç hiçbir noktanın koordinatlarının tamsayı <strong>olm</strong>adığı açıktır.<br />

n<br />

Eğer böyle bir nokta varsa, mesela (m, n), 0 < m < a, 0 < n < b,<br />

m = b a<br />

n<br />

<strong>olm</strong>alıdır. Demek ki<br />

m<br />

kesri, b<br />

a<br />

sadeleşebilen kesrinin bir sadeleşmi halidir.<br />

Ancak bu durum açık bir çelişkidir. Çünkü (a, b) = 1 kabul etmiştik.<br />

L k = (k, kb<br />

a<br />

), 1 ≤ k ≤ a − 1 oktalarının her biri doğrunu üzerindedir. Buna<br />

göre [ kb<br />

kb<br />

a<br />

] ifadesi (k, 0) dan (k,<br />

a<br />

) noktasına giden doğrunun üzerindeki latis<br />

noktalarıdır. Yani<br />

∑a−1<br />

[ kb<br />

a ]<br />

k=1<br />

ifadesi dikdörtgenin alt yarısındaki latis noktalarının sayısıdır. Benzer biçimde,<br />

∑b−1<br />

[ ka<br />

b ]<br />

k=1<br />

ifadeside üst yarıdaki noktalar olduğuna göre ve bu noktalar alt ve üst yarıda<br />

eşit sayıda bulunabildiğine göre toplam nokta sayısının yarısı<br />

sitenen cevap olur. ispat tamamlanır.<br />

(a − 1)(b − 1)<br />

2<br />

Örnek.<br />

kanıtlayınız.<br />

n ∈ Z + <strong>olm</strong>ak üzere, [ √ n + √ n + 1] = [ √ 4n + 2] eşitliğini<br />

Çözüm. İfadenin karesini alırsak √ n + √ n + 1 toplamının √ √<br />

4n + 1 ile<br />

4n + 3 ifadelerinin arasında olduğunu görmek zor değildir. Tam kare ifadeler<br />

mod4 altında 0 veya 1 olduklarından 4n + 2 ve 4n + 3 ifadelerinin birer tam<br />

kare <strong>olm</strong>adıkları açıktır. Buna göre [ √ 4n + 2] = [ √ 4n + 3] olduğu açıktır.<br />

Teorem.(De - Polignac ve Legendre Formülü) n! ifadesini bölen p asal


2.11.<br />

TAMDEĞER FONKSİYON PROBLEMLERİ 107<br />

sayısının en büyük kuvveti α ise<br />

eşitliği vardır.<br />

∞∑<br />

α = [ n p k ]<br />

k=1<br />

Örnek.<br />

999! ifadesinin ondalık yazılımının sonunda kaç tane 0 vardır<br />

Çözüm. Sıfırların sayısı çarpımdaki 2 ve 5 sayılarına bağlıdır buna göre<br />

5 α |999! <strong>olm</strong>asını sağlayan en büyük α değeri istenen cevap olacaktır. Buna<br />

göre;<br />

[ 999<br />

5 ] + [999 5 2 ] + [999 5 3 ][999 ] = 179 + 39 + 7 + 1 = 246 olur.<br />

54 Örnek.<br />

ifadesi doğruluğunu araştırınız.<br />

( ) 1000<br />

7|<br />

500<br />

Çözüm.<br />

7 α |1000! <strong>olm</strong>asını sağlayan en büyük α değeri‘;<br />

[ 1000<br />

7 ] + [1000 7 2 ] + [1000 7 3 ] = 164<br />

olacaktır. Benzer biçimde 7 α |500! <strong>olm</strong>asını sağlayan en büyük α değeride 82<br />

bulunur. ( ) 1000<br />

= 1000!<br />

500 (500!) 2<br />

olduğundan, bu ifadeyi bölen 7 nin en büyük kuvveti 164 − 2 · 82 = 0 olduğundan,<br />

soruda verilen ifade doğru değildir.<br />

2.11.1 Çalışma Soruları<br />

1.<br />

[ 3√ 1] + [ 3√ 2] + . . . + [ 3√ x 3 − 1] = 400<br />

denkleminin doğal sayılardaki çözüm kümesinin elemanlarını bulunuz.


108 BÖLÜM 2. KONULAR<br />

2. (6 + √ 35) 1980 açılımındaki virgülden sonraki ilk 1000 basamağın 9 olduğunu<br />

gösteriniz.<br />

3. [x 2 ] = [x] denkleminin çözüm kümesini bulunuz.<br />

4.<br />

8<br />

15 < n<br />

n + k < 7<br />

13<br />

eşitsizliğini sağlayan en büyük n değeri kaçtır (k değeri tektir).<br />

5. r ∈ R <strong>olm</strong>ak üzere;<br />

91∑<br />

k=19<br />

ise [100r] ifadesinin eşitini bulunuz.<br />

[r + k<br />

100 ] = 546<br />

6. f(n) fonksiyonu n 1/4 sayısına en yakın tamsayıyı temsil etmek üzere,<br />

toplamının tamdeğerini bulunuz.<br />

1995<br />

∑<br />

n=1<br />

1<br />

f(n)<br />

7. [x] ifadesi x sayısının tamdeğerini temsil etmek üzere<br />

1 2<br />

[<br />

1998 ], [ 2 2<br />

], . . . , [19972<br />

1998 1998 ]<br />

dizisindeki farklı tamsayıların sayısını bulunuz.<br />

2.11.2 Çözümler<br />

1. x ∈ N olduğuna göre;<br />

3√<br />

1, . . . ,<br />

3 √ 7 → 1 · 7 = 7<br />

3√<br />

8, . . . ,<br />

3 √ 26 → 2 · 19 = 38<br />

3√<br />

27, . . . ,<br />

3 √ 63 → 3 · 37 = 111<br />

3√<br />

64, . . . ,<br />

3 √ 124 → 4 · 61 = 244<br />

ise 7 + 38 + 111 + 244 = 400 ise tek çözüm x = 5 <strong>olm</strong>alıdır.


2.11.<br />

TAMDEĞER FONKSİYON PROBLEMLERİ 109<br />

2. (6 + √ 35) 1980 + (6 − √ 35) 1980 = 2k, k ∈ Z olur. Burada 6 − √ 35 <<br />

1<br />

1<br />

10<br />

<strong>olm</strong>alıdır aksi durumda yani eğer<br />

10 < 6 − √ 35 olursa, iki tarafın<br />

karesinden 3500 < 3481 olur.<br />

0 < (6 − √ 35) 1980 < 10 −1980 ⇒ 2k − 1<br />

10 1980 < (6 + √ 35) 1980 < 2k<br />

⇒ 2k − 1 + 0, 999 . . . 9<br />

} {{ }<br />

1979 tane 9<br />

< (6 + √ 35) 1980 < 2k<br />

3. x 2 ≥ 0 ise [x 2 ≥] olur. x ∈ [0, 1] aralığındaki tüm sayıların sağladığı<br />

açıktır. Biz (1, 2) aralığına bakalım;<br />

x = 1 + {x}, [(1 + {x} 2 )] = [1 + {x}] ise [1 + 2{x} + {x} 2 ] = [1 + {x}]<br />

[1 + 2{x} + {x} 2 ] = 1 0 ≤ 2{x} + {x} 2 < 1 ⇒ {x} > 0 veya {x} 2 +<br />

2{x} − 1 < 0 durumları oluşur. Buradan eğer y = {x} olarak alırsak<br />

y ∈ (0, √ 2 − 1) <strong>olm</strong>alıdır. Buradan da, x ∈ (0, √ 2) arasında olur. 0 zaten<br />

ahil olduğundan istenilen aralık x ∈ [0, √ 2] bulunur.<br />

4. Verilen ifadeyi düzenlersek<br />

6n<br />

7 < k < 7n 8<br />

olacaktır. Fakat buradan 7n 8 − 6n 7 = n 56<br />

bulunur. Buna göre eğer n > 112<br />

ise, bu aralıkta kesinlikle, yani ( 6n 7 , 7n 8<br />

) aralığında, iki tamsayı olacaktır.<br />

n = 112 için ise, 96 < k < 98 olacağından cevap 112 <strong>olm</strong>alıdır.<br />

5. 546 = 7 · 73 + 35 = 38 · 7 + 35 · 8 olur. Demek ki<br />

7 = [r + 0, 19] = [r + 0, 20] = [r + 0, 21] = . . . [r + 0, 56]<br />

8 = [r + 0, 57] = [r + 0, 58] = . . . = [r + 0, 91]<br />

olacağından 7, 43 ≤ x < 7, 44 ise 74, 3 ≤ 100r < 74, 4 ve [100r] = 74<br />

olur.<br />

6. f(k) ifadesi 1, 2, 3, . . . , 7 değerlerinden birini alır. Sınır değerler<br />

(n + 1 2 )4 = n 4 + 2n 3 + 1 2 · (3n2 + n) + 1<br />

16


110 BÖLÜM 2. KONULAR<br />

olduğuna göre<br />

5 tanesi : 1<br />

34 tanesi : 1/2<br />

111 tanesi : 1/3<br />

260 tanesi : 1/4<br />

505 tanesi : 1/5<br />

870 tanesi : 1/6<br />

210 tanesi : 1/7<br />

olacağına göre, toplam 400 olur.<br />

7. [ 9982<br />

9992<br />

1998<br />

] = 498 < 499 = [<br />

1998<br />

] olduğuna göre önce k = 1, 2, 3, . . . , 998 ve<br />

k = 999, 1000, . . . , 1997 durumlarını ayrı ayrı inceleyelim. Buna göre,<br />

k = 1, 2, 3, . . . , 997 için<br />

olacaktır. Buna göre<br />

(k + 1) 2<br />

1998<br />

1 2<br />

−<br />

k2<br />

1998 = 2k + 1<br />

1998 < 1<br />

9982<br />

[ ] = 0, 1, 2, . . . , 498 = [<br />

1998 1998 ]<br />

dizisinde her bir sayı en az bir defa tekrarlanacaktır. Buna göre dizinin<br />

bu bölümünde 499 terim vardır. k = 999, 1000, . . . , 1996 ise<br />

(k + 1) 2<br />

1998<br />

−<br />

k2<br />

1998 = 2k + 1<br />

1998 > 1<br />

ise burada da, 1997−999+1 = 999 farklı terim vardır. Buna göre dizideki<br />

farklı tamsayıların sayısı 1498 olacaktır.<br />

2.12 Bölüneb˙ılme ve Asal Sayılar<br />

Tanım. a ≠ 0, b ∈ Z <strong>olm</strong>ak üzere, eğer a · c = b eşitliğini sağlayan bir<br />

c ∈ Z bulunuyorsa bu duruma a böler b denir ve a|b olarak yazılır.


2.12. BÖLÜNEBİLME VE ASAL SAYILAR 111<br />

Teorem.<br />

1. a, b, c, m, n ∈ Z <strong>olm</strong>ak üzere c|a, c|b ise c|(a · m + b · n) dir.<br />

2. x, y, z ∈ Z <strong>olm</strong>sk üzere; x|y, y|z ise x|z dir.<br />

Kanıt.<br />

1. s, t ∈ Z <strong>olm</strong>ak üzere varsayalım s · c = a, t · c = b olsun. Buna göre<br />

am + bn = c(sm + tn) ise c|(am + bn) olur.<br />

2. Ve varsayalım u, v ∈ Z <strong>olm</strong>ak üzere xu = y, yv = z olsun. Buna göre<br />

xuv = z ise x|z olduğu açıktır. Ayrıca eğer a|b ve b ≠ 0 ise ≤ |a| < |b|<br />

durumu vardır.<br />

Örnek.<br />

bulunuz.<br />

(n + 1)|(n 2 + 1) durumunu doğrulayan tüm n ∈ Z + değerlerini<br />

Çözüm. n 2 + 1 = n 2 − 1 + 2 = (n − 1) · (n + 1) + 1 ise (n − 1) · (n + 1)<br />

sayısı (n + 1) ise bölünür. Bu durumda (n + 1)|2 <strong>olm</strong>alıdır. |n + 1| < |2| ise<br />

n = 1 olur.<br />

Örnek. n herhangi bir doğal sayı <strong>olm</strong>ak üzere; n 3 + 2n sayısının 3 ile<br />

bölünebildiğini gösteriniz.<br />

Çözüm. n değeri 3k + 1, 3k + 2 ve 3k formunda olabilir.<br />

n = 3k ise (3k) 3 + 2 · (3k) = 3 · (3 2 · k 3 ) + 3 · (2k) ise 3|n 3 + 2n<br />

n = 3k+1 ise (3k+1) 3 +2·(3k+1) = (3k+1) 3 +2·3k+2 ≡ 1+0+2 ≡ 0(mod3)<br />

ise 3|n 3 + 2n<br />

n = 3k+2 ise (3k+2) 3 +2·(3k+2) = (3k+2) 3 +2·3k+4 ≡ 2+0+1 ≡ 0(mod3)<br />

ise 3|n 3 + 2n.<br />

Örnek.<br />

7|3x + 2 ise 7|(15x 2 − 11x − 14) olduğunu kanıtlayınız.<br />

Çözüm. 15x 2 − 11x − 14 = (3x + 2) · (5x − 7) = 7s · (5x − 7) ise kanıt<br />

tamamlanmış olur.


112 BÖLÜM 2. KONULAR<br />

Örnek. a, b, c doğal sayılar <strong>olm</strong>ak üzere, a 2 + b 2 = c 2 eşitliği sağlanmaktadır.<br />

Buna göre, a, b ve c den en az birinin 3 ile bölünebildiğini gösterelim.<br />

Çözüm. Eğer a, b ve c değerleri için bir genellme yaparsak, varsayalım<br />

hiçbiri 3 ile tam bölünmesin. Öyleyse, a = 3k + 1, b = 3t + 1, c = 3r + 1 olsun.<br />

Buna göre,<br />

(3k + 1) 2 + (3t + 1) 2 ≡ 1 + 1(mod3)<br />

ise c 2 ≡ (mod3) olur, ancak tamsayının karesi bu denkliği sağlamaz.<br />

Teorem. Ardışık n tamsayının çarpımı n! ile tam bölünür.<br />

Kanıt. Varsayalım ardışık n tamsayımız m + 1, m + 2, m + 3, . . . , m + n formunda<br />

olsun ve bu dizinin her bir elemanı pozitif tamsayı olsun. Buna göre,<br />

göstermemiz gereken durum;<br />

n!|m + 1, m + 2, m + 3, . . . , m + n<br />

dir. Burada binom katsayıların tamsayı <strong>olm</strong>a özelliğini kullanırsak<br />

( ) m + n<br />

(m + n)!<br />

(m + n)! (m + 1) · (m + 2) · (m + 3) · . . . · (m + n)<br />

=<br />

= = ∈<br />

n (m + n − m)!(m + n − n)! n!m!<br />

n!<br />

ise ispat tamamlanır.<br />

Örnek.<br />

6|n 3 − n olduğunu gösteriniz.<br />

Çözüm. n 3 − n = n · (n 2 − 1) = n · (n − 1) · (n + 1) ise daha önce verilen<br />

teoremden dolayı doğrudur.<br />

Teorem. n ∈ Z + <strong>olm</strong>ak üzere, n bir bileşik sayı 13 ise, n sayısının p ≤ √ n<br />

olacak şekilde bir asal çarpanı vardır.<br />

Kanıt. Varsayalım n = a · b, 1 < a ≤ b olsun. Eğer bu iki değerde √ n de<br />

büyükse, n = a · b = √ n · √n = n olur ki, bu bir çelişkidir. Demek ki, n<br />

sayısının 1 den farklı ve √ n den küçük eşit olan bir çarpanı vardır. Buna göre,<br />

p bir asal çarpan <strong>olm</strong>ak üzere, bu değerde √ n den küçük olur.<br />

13 2 den büyük olan ve asal <strong>olm</strong>ayan sayılar


2.12. BÖLÜNEBİLME VE ASAL SAYILAR 113<br />

Örnek. 101 sayısının asal olup <strong>olm</strong>adığını anlamak için bu sayıyı en az<br />

kaç sayıya bölmeliyiz.<br />

Çözüm.<br />

√<br />

101 ≈ 10 ise p = 2, 3, 5, 7 <strong>olm</strong>alıdır. Toplamda 4 sayı yeterlidir.<br />

Örnek. Elemanlarından hiçbirisi asal sayı <strong>olm</strong>ayan gelişigüzel uzunlukta<br />

bir ardışık sayı dizisi bulunabilir mi<br />

Çözüm.<br />

k ∈ Z, k ≥ 2 <strong>olm</strong>ak üzere<br />

k! + 2, k! + 3, . . . , k! + k<br />

dizisinin elemanları ardışıktır ve asal değildir. Yani<br />

A = (n + 1)! + 2, (n + 1)! + 3, . . . , (n + 1)! + (n + 1)<br />

dizisini ele alalım. 2 ≤ k < n + 1 bağıntısını sağlayan herbir k tamsayısı için<br />

k|(n + 1)! dir. k|(n + 1)! + k ise ispat tamamlanır. Mesela<br />

23! + 1, 23! + 2, . . . , 23! + 28<br />

dizisindeki sayılardan hiçbirisi asal değildir.<br />

Not. p > 3 <strong>olm</strong>ak üzere bütün asal sayılar, n ∈ Z <strong>olm</strong>ak üzere p = 6n ± 1<br />

formundadır.<br />

Örnek.<br />

p ve p 2 + 2 asal sayılar ise p 3 + 2 ninde asal olduğunu gösteriniz.<br />

Çözüm. p değeri tek <strong>olm</strong>alıdır. p = 3 için p 2 + 2 = 11 ve p 3 + 2 = 29<br />

dur. Burada p > 3 ise p = 6n ± 1 olarak alınırsa, p 2 + 2 sayısı 3 ile bölünebilir.<br />

Demek ki, p = 3 dışında soruda verilen durumu doğrulayan bir sayı yoktur.<br />

Tek çözüm, p = 3 olur.<br />

Örnek.<br />

19 dan 92 ye kadar olan sayılar yanyana yazılarak elde edilen,<br />

19202122 . . . 9192<br />

sayısını bölen 3 ün en büyük kuvvetini bulunuz.


114 BÖLÜM 2. KONULAR<br />

Çözüm. 9 ile bölübeilme kuralına bakalım 19+20+21+. . .+92 = 37 2 ·3<br />

ise bu sayı 9 ile bölünemez, sadece 3 ile bölünebilir.<br />

Örnek.<br />

p 3 + p 2 + 11p + 2<br />

ifadesini asal yapan en büyük p asal sayısını bulunuz.<br />

Çözüm. p = 3 için, ifademiz 71 değerini alır. 3 ten büyük değerler için<br />

yani p = 3k + 1 ve p = 3k + 2 değerlerinde ifademiz sırasıyla 3 · (9k 3 + 12k 2 +<br />

16k + 5) ve 9 · (3k 3 + 7k 2 + 9k + 4) değerlerini alır. Ancak bu değerlerin ikiside<br />

asal değildir. Demek ki tek değer p = 3 tür.<br />

Örnek.<br />

gösteriniz.<br />

Çözüm.<br />

2a + 1 ve 3a + 1 birer tamkare ise 5a + 3 sayısının asal <strong>olm</strong>adığını<br />

2a + 1 = x 2 ve 3a + 1 = y 2 ise<br />

5a + 3 = 4 · (2a + 1) − (3a + 1) = 4x 2 − y 2 = (2x − y)(2x + y)<br />

durumu vardır. 5a+3 asal ise, 2x−y = 1 ve 2x+y = 5a+3 <strong>olm</strong>alı. 4x = 5a+3<br />

ise 5x 2 −8x+3 = 0 olur. Burada bulunan her bir kök tamsayı <strong>olm</strong>ak zorundadır.<br />

Buradan istenen tamsayı x değeri −1 bulunur. Buna göre, sadece (1, 1, 3) için<br />

durum sağlanır diğer durumlarda sağlanmaz.<br />

2.12.1 Çalışma Soruları<br />

1. n ∈ Z + <strong>olm</strong>ak üzere,<br />

(n + 10)|(n 3 + 100)<br />

<strong>olm</strong>asını sağlayan en büyük n tamsayı değerini bulunuz.<br />

2. n 5 −5n 3 +4n iadesinin daima 120 ile kalansız bölünebileceğini gösteriniz.<br />

3. p > 3 ve p asal bir sayı <strong>olm</strong>ak üzere, 24|(p 2 − 1) olduğunu gösteriniz.<br />

4. Herhangi bir tamsayının karesinin 4k veya 4k + 1 formunda olduğunu<br />

gösteriniz.


2.12. BÖLÜNEBİLME VE ASAL SAYILAR 115<br />

5. 11, 111, 1111, 11111, . . . dizisindeki sayılardan hiçbirinin tamkare <strong>olm</strong>adığını<br />

kanıtlayınız.<br />

6. Bir tamsayının 5. kuvvetine eşit olan 6p + 1 formundaki tüm p asal<br />

sayılarını bulunuz.<br />

7. 21 · p + 1 sayısını tamkare yapan kaç p asal sayısı vardır<br />

8. 105 sayısı bir kaç ardışık doğal sayının toplamı olarak kaç farklı biçimde<br />

yazılabilir<br />

9. Kaç tane p asal sayısı için p 2 + 17 sayısının 4 tane farklı pozitif böleni<br />

vardır<br />

10. n > 3 <strong>olm</strong>ak üzere, (n − 3)|n 2 − n durumunu doğrulayan bütün n tamsayılarını<br />

bulunuz.<br />

11. {1, 2, 3, 4, . . . , 23, 24, 25} kümesinden en az kaç eleman atılmalı ki geriye<br />

kalan elemenların çarpımı bir tam kareye eşit olsun.<br />

12. √ 13p + 289 sayısının bir tamsayı <strong>olm</strong>asını sağlayan en büyük p asal sayısını<br />

bulunuz.<br />

13. a. N sayısı b tabanında 777 dir. N nin bir tamsayının dördüncü kuvvetine<br />

eşit <strong>olm</strong>asını sağlayan en küçük b tamsayısını bulunuz.<br />

b. n bir tamsayı olsun. n 2 nin onlar basamağındaki rakam 7 ise, n 2<br />

nin birler basamağındaki rakamı bulunuz.<br />

14. n+1 ve 16n+1 ifadelerinin ikisinide tamkare yapan n ≥ 1 tamsayılarının<br />

sayısını bulunuz.<br />

15. (n + 1) · (n 4 + 2n) + 3 · (n 3 + 57) ifadesinin (n 2 + 2) ile bölünmesini<br />

sağlayan en büyük n tamsayısının değerini bulunuz. (ipucu: polinom<br />

bölmesini kullanınız)<br />

16.<br />

49, 4489, 444889, 44448889, . . . , 44 } .{{ . . 44}<br />

88 } .{{ . . 88}<br />

9, . . .<br />

n tane 4 n-1 tane 8<br />

dizisindeki her bir terimin tamkare olduğunu kanıtlayınız.


116 BÖLÜM 2. KONULAR<br />

17. } 111 {{ . . . 11}<br />

sayısının bileşik sayı olduğunu gösteriniz.(ipucu: 221 = 17·13)<br />

221 tane 1<br />

18. a = } 1111 .{{ . . 1111}<br />

, b = 1 } 000 .{{ . . 000}<br />

5 ise a · b + 1 sayısının bir tamkare<br />

m tane 1<br />

m-1 tane 0<br />

olduğunu gösteriniz.<br />

19.<br />

111 } .{{ . . 111}<br />

− 222 } .{{ . . 222}<br />

2n tane 1 n tane 2<br />

farkının bir tamkare olduğunu gösteriniz.<br />

20. 2x 2 − x − 36 ifadesini bir asal sayının karesi yapan bütün x tamsayılarını<br />

bulunuz.<br />

21. En az iki ardışık pozitif tamsayının toplamı şeklinde yazılabilen pozitif<br />

bir tamsayının 2 nin kuvvetine eşit olamayacağını gösteriniz.<br />

22. 1 den 100 e kadar (100 dahil) sayılardan kaç tanesi, iki veya daha fazla<br />

ardışık pozitif tamsayının toplamı olarak yazılabilir<br />

23. İlk terimi n olan 98 ardışık sayının toplamı 192 ile bölünmektedir. Buna<br />

göre en küçük n ∈ Z değerini bulunuz.<br />

24. a · b = c · d eşitliğini sağlayan a, b, c, d > 0 tamsayıları verildiğinde a 2 +<br />

b 2 + c 2 + d 2 toplamının hiç bir zaman asal olamayacağını gösteriniz.<br />

25. 2903 n −803 n −464 n +261 n ifadesinin n in alabileceği herhani bir tamsayı<br />

değeri için 1897 ile bölünebileceğini gösteriniz.<br />

26. 1492 n − 1770 n − 1863 n + 2141 n ifadesinin n nin alabileceği herhangi bir<br />

tamsayı değeri için 1946 ile bölünebileceğini gösteriniz.<br />

2.12.2 Çözümler<br />

1. Eğer polinom bölmesi özeliğini kullanırsak elimizde<br />

n 3 + 100 = (n + 10)(n 2 − 10n + 100) − 900


2.12. BÖLÜNEBİLME VE ASAL SAYILAR 117<br />

eşitliği oluşur. Ancak bölme işleminin kalansız olabilmesi için bölme işleminde<br />

kalanın sıfır <strong>olm</strong>ası gerekir. Demek ki,<br />

n + 10|900<br />

<strong>olm</strong>alıdır. n + 10 ≤ 900 ise n değerinin en büyük değeri 890 olur.<br />

2. n(n 4 −5n 2 +4) = (n 2 −1)·(n 2 −1)·n = (n−2)·(n+2)·(n−1)·(n+1)·n<br />

ise ifade ardışık 5 sayının çarpımıdır ve 5! ile kalansız bölünür.<br />

3. p > 3 ise p = 6k ± 1 formundadır. Buna göre,<br />

4.<br />

(6k+1) 2 −1 2 = (6k+1−1)·(6k+1+1) = 6k·(6k+2) = 6k·2(3k+1) = 12k·(3k+1)<br />

olur. k sayısı tek ise 3k + 1 = 2A, A ∈ Z ⇒ 12k · (2A) = 24 · A · K olur.<br />

2a, 2a + 1 ⇒ (2a) 2 = 4a 2<br />

(2a + 1) 2 = 4a 2 + 4a + 1 = 4(a 2 + a) + 1 olur.<br />

5. Bir önceki örnekten dolayı tamkare <strong>olm</strong>adığı açıktır. Çünkü tamkare bir<br />

ifade 4k veya 4k + 1 formundadır.<br />

6. (x) 5 = (6p + 1) ise x sayısı da kesinlikle asaldır. x = 3 ise x 5 = 243<br />

olur. 240 = 6 · 40 + 3 ise x = 3 olamaz. 6p = x 5 − 1 = (x − 1) ·<br />

( x<br />

} 4 + x 3 +<br />

{{<br />

x 2 + x + 1<br />

}<br />

). Demek ki (x − 1) de 6 nın çarpanı ise<br />

Bu kısım x=2 için 6 dan büyük olur.<br />

x = 2, 3, 4 veya 7 olur. Bunlardan sadece 7 sağlar. 7 5 = 6 · 2801 + 1 olur.<br />

7.<br />

21 · p + 1 = x 2<br />

21 · p = x 2 − 1<br />

21 · p = (x − 1) · (x + 1) ⇒ (x + 1) − (x − 1) = 2 <strong>olm</strong>alıdır.<br />

Yani, p = 23 veya p = 19 olabilir.3 · 7 · p = (x − 1) · (x + 1) ⇒ p = 3<br />

olabilir.


118 BÖLÜM 2. KONULAR<br />

8. n = k + (k + 1) + (k + 2) + . . . + (k + m) olarak seçelim. Buna göre<br />

n =<br />

(m + 1) 1<br />

m + 2k 2<br />

olur. Burada, (2k + m) − (m + 1) = 2k − 1 ifadesi tek sayı olduğuna göre<br />

sayısını veren<br />

(m + 1) · (m + 2k) = 210<br />

(105, 2); (70, 3); (42, 5); (35, 6); (30, 7); (21, 10); (15, 14)<br />

biçiminde 7 sayı vardır.<br />

9. p 2 + 17 sayısında p asal ise p = 2 için 4 + 17 = 21 sayısının 4 farklı böleni<br />

ve p = 3 için p 2 +17 = 26 = 2·13 ise 4 tane bölende burada vardır. p ≥ 3<br />

ise p 2 +17 = x ve p 2 −1 = (x−18) olduğuna göre, (p−1)(p+1) = x−18<br />

olur. Burada (p − 1)(p + 1), 3 ile tam bölünür. Buna göre;<br />

(p − 1)(p + 1) ≡ 0 ≡ x − 18 ≡ x(mod3) ise 3|x durumu vardır.<br />

(p − 1)(p + 1) ≡ 0 ≡ x − 18 ≡ x(mod2) ise 2|x durumu vardır.<br />

x = a·b veya x = c 3 olabilir. x = 2·3 = 6 <strong>olm</strong>alıdır. x = 6 ise p 2 +17 = 6<br />

durumu <strong>olm</strong>az. Demek ki sadece 2 ve 3 için 4 asal böleni vardır.<br />

10. Eğer doğrulayan n değerlerini bulmak istiyorsak, ifademizi<br />

n 2 − n = (n − 3) · (n + 2) + 6<br />

olarak yazabiliriz. Tam bölünebilme olabilmesi için (n − 3)|6 <strong>olm</strong>alıdır.<br />

Buna göre n değerleri, {4, 5, 6, 9} <strong>olm</strong>alıdır.<br />

11. Varsayalım<br />

A = 25! = 2 a1 · 3 a2 · 5 a3 · 7 a4 · 11 a5 · 13 a6 · 17 a7 · 19 a8 · 23 a 9<br />

olsun. Burada, a 1 = 22, a 2 = 10, a 3 = 6, a 4 = 3, a 5 = 2, a 6 = 1, a 7 =<br />

1, a 8 = 1, a 9 = 1 olacağından bu çarpımdan çıkarılması gereken sayılar<br />

{23, 19, 17, 13, 7} olacaktır. Bu sayıların atılması yeterlidir.


2.12. BÖLÜNEBİLME VE ASAL SAYILAR 119<br />

12. √ 13p + 289 = x, x ∈ Z + olduğuna göre, 13p + 289 = x 2 ve 13p =<br />

(x − 17) · (x + 17) bulunur. p asal olduğundan böleni yoktur. x − 17 = 13<br />

ve x = 30 bulunur. Demek ki, 13 · p = 13 · 4 ise p = 47 bulunur.<br />

13. a. N = (777) b = 7b 0 + 7b 1 + 7b 2 = x 4 ⇒ 7(b 2 + b + 1) = x 4 eşitliğinden<br />

b 2 + b + 1 = 7 3 · A <strong>olm</strong>alıdır. A = 1 olursa, b · (b + 1) = 7 3 − 1 ise<br />

buradan b = 18 bulunur.<br />

b. Varsayalım n = 10x+y olsun. Buradan n 2 = } 100 {{· x}<br />

2 + 20 · xy + y 2 .<br />

} {{ } }{{}<br />

çift çift tek <strong>olm</strong>alı<br />

Buna göre y 2 = 36 yada 16 olabilir. Buna göre istenen cevap 6 <strong>olm</strong>alıdır.<br />

14. Varsayalım<br />

x 2 = n + 1<br />

y 2 = 16n + 1<br />

olsun. Buna göre, y 2 − x 2 = (y − x) · (y + x) = 15 · n ifadesinde n yerine<br />

x 2 −1 yazarsak yeni ifademiz (4x−y)·(4x+y) = 15 elde edilir. Buradan<br />

istenen tek değer n = 3 olur.<br />

15. Bu sorunun çözümü okuyucuya bırakılmıştır.<br />

16.<br />

44 } .{{ . . 44}<br />

88 } .{{ . . 88}<br />

9 = 44 . . . 4 · 10 n + 88 . . . 8 · 10 + 9<br />

n tane 4 n-1 tane 8<br />

= 4 9 (10n − 1) + 8 9 (10n−1 − 1) + 9<br />

17. Bu sorunun çözümü okuyucuya bırakılmıştır.<br />

18. Bu sorunun çözümü okuyucuya bırakılmıştır.<br />

= 4 9 · 102n + 4 9 · 10n + 1 9<br />

= 1 9 · (2 · 10n + 1) 2 = ( 2 · 10n + 1<br />

) 2 olur.<br />

3


120 BÖLÜM 2. KONULAR<br />

19. Bu sorunun çözümü okuyucuya bırakılmıştır.<br />

20. 2x 2 − x − 36 = p 2 ise (x + 4) · (2x − 9) = p 2 olacaktır. Buna göre;<br />

i.<br />

x + 4 = p 2<br />

2x − 9 = 1 ⇒ x = 5, p = 3<br />

ii.<br />

x + 4 = p<br />

2x − 9 = p ⇒ x = 13, p = 17<br />

iii.<br />

x + 4 = 1<br />

2x − 9 = p 2 ⇒ x = −3, p 2 = −15 olamaz.<br />

iv.<br />

x + 4 = −p 2<br />

2x − 9 = −1 ⇒ x = 4, p 2 = −8 olamaz.<br />

v.<br />

x + 4 = −p<br />

2x − 9 = −p ⇒ x = 13, p = −17 olamaz.<br />

vi.<br />

x + 4 = −1<br />

2x − 9 = −p 2 ⇒ x = −5, p = 19 olamaz.<br />

21. Varsayalım ardışık sayılarımız,<br />

m, (m + 1), (m + 2), . . . , (m + k)


2.12. BÖLÜNEBİLME VE ASAL SAYILAR 121<br />

ise Bu sayıların toplamı,<br />

(k + 1)(2m + k) 1 2<br />

olacaktır. Eğer bu toplam 2 nin bir kuvvetine eşit ise, u ∈ Z için,<br />

2 u = (k + 1)(2m + k) 1 2 ⇒ 2u+1 = (k + 1)(2m + k)<br />

eşitliği ele edilir. Buradan da,<br />

k tek ise, m çift olacaktır. Buna göre, (T + 1)(C + T ) = C · T →<br />

Tek sayı ≠ 2 u+1<br />

k çift ise, m tek olacaktır. Buna göre, (C + 1)(C + C) = T · C →<br />

Tek sayı ≠ 2 u+1 olacaktır.<br />

22. Önceki çözülen örneklerin sonucundan faydalanırsak, 2 nini kuvvetlerinin<br />

yazılamayacağını görürüz. Eğer ilk yüz sayıdan ikinin kuvvetlerini<br />

çıkarırsak, istenen sonuca ulaşabiliriz.<br />

23.<br />

n + (n + 1) + (n + 2) + · · · + (n + 97) = 19 2 · K<br />

97 · 98<br />

98 · n +<br />

2<br />

= 19 2 · K<br />

98 · n + 97 · 49 = 19 2 · K<br />

49 · (2n + 97) = 19 2 · K<br />

(2n + 97) = 19 2 = 361 ise n = 32 olur.<br />

24. Varsayalım, a = p · q, b = r · s, c = p · r ve d = q · s olsun. Buna göre<br />

soruda verilen toplam<br />

p 2 q 2 + r 2 s 2 + p 2 r 2 + q 2 s 2 = (p 2 + s 2 )(q 2 + r 2 )<br />

elde edilirki buda çözüm için yeterlidir.<br />

25. (x − y)|(x n − y n ) olduğunu zaten biliyoruz. Buna göre, 2903 n − 803 n<br />

ifadesi 2903 − 803 = 2100 = 7 · 300 ve 261 n − 464 n ifadeside 261 − 464 =<br />

−203 = −29 · 7 ile bölünebilir. Demek ki, verilen ifade 7 ile bölünebilir.


122 BÖLÜM 2. KONULAR<br />

Benzer biçimde, 2903 n − 464 n ifadeside 2903 − 464 = 2439 = 9 · 271 ve<br />

−803 n + 261 n ifadeside −803 + 261 = −542 = −2 · 271 ise deekki, ifade<br />

271 ilede tam bölünür. Buna göre, ifade 7·271 = 1897 ilede tam bölünür.<br />

26. Bu sorunun çözümü okuyucuya bırakılmıştır.<br />

2.13 Tr˙ıgonometr˙ıde Sonsuz Toplam ve Farklar<br />

Aslında başlık olarak hernekadar cebirdeki sonsuz toplam ve farklarıanımsatsa<br />

da aynısı değildir. Ancak genel mantıkta benzeşmeler ve hatta örtüşmeler<br />

olduğu soruların çözümleri yapılırken göze çarpar.İlerleyen örneklerde ve<br />

çalışma sorularında sizde farkına varacaksınız ki, genel amacımız verilen bir<br />

sonsuz toplamı fark formunda, yani<br />

n∑<br />

[F (k) − F (k − 1)]<br />

k=2<br />

formunda yazarak F (n) − F (1) sonucuna ulaşmak olacaktır. Önce nispeten<br />

daha kolay bir örnekle başlayalım.<br />

Size tavsiyemiz bu bölümdeki sorulara çalışmaya başlamadan evvel trigonometrik<br />

özdeşliklere bir göz atmanız yönünde olacaktır. Böylelikle konuya<br />

daha hakim olabilirsiniz.<br />

Örnek.<br />

n∑<br />

cos kx<br />

k=1<br />

toplamını hesaplayınız.<br />

Çözüm. Eğer<br />

n∑<br />

cos kx = A eşitliğinde eşitliğin iki tarafınıda 2 sin x 2<br />

k=1<br />

ile çarparsak, çarpımdan toplama geçebilmek için özdeşlikleri kullanabiliriz.


2.13. TRİGONOMETRİDE SONSUZ TOPLAM VE FARKLAR 123<br />

Öyleki<br />

2 sin x n∑<br />

2 · A = sin x cos kx<br />

2<br />

=<br />

k=1<br />

n∑<br />

k=1<br />

= sin<br />

( (<br />

sin k + 1 ) (<br />

x − sin k − 1 ) )<br />

x<br />

2<br />

2<br />

(<br />

n + 1 )<br />

x − sin x 2 2<br />

olacaktır.<br />

Buna göre, bu eşitlikten<br />

A = sin ( n + 1 )<br />

2 x<br />

2 sin ( )<br />

x<br />

− 1 2<br />

2<br />

bulunur.<br />

İkinci örneğimiz de tanjant toplamını kullanarak rahatlıkla çözüme ulaşabileceğimiz<br />

bir örnek.<br />

Örnek.<br />

n∑<br />

( )<br />

tan −1 1<br />

k 2 + k + 1<br />

k=0<br />

toplamını hesaplayınız.(tan −1 x = arctan x)<br />

Örnek.<br />

Tanjant fark formülü<br />

tan (a − b) =<br />

tan a − tan b<br />

1 + tan a tan b<br />

olduğuna göre<br />

tan −1 u − tan −1 v = tan −1 u − v<br />

1 + uv<br />

olacaktır. Daha da basite indirgemek için a k = tan −1 k olarak alalım. Buna<br />

göre,<br />

tan (a k+1 − a k ) = tan a k+1 − tan a k<br />

1 + tan a k+1 tan a k<br />

k + 1 − k<br />

=<br />

1 + k (k + 1) = 1<br />

k 2 + k + 1<br />

olacaktır.


124 BÖLÜM 2. KONULAR<br />

Öyleyse toplamımız,<br />

n∑<br />

tan −1 (tan (a k+1 − a k )) =<br />

k=0<br />

n∑<br />

(a k+1 − a k ) = a n+1 − a 0<br />

k=0<br />

= tan −1 (n + 1) olacaktır.<br />

Benzer çözüm biçimleri verilen çalışma soruları üzerinde uygulanabilir. Buna<br />

göre sonsuz bir toplamı çözerken, ifadeyi farkların sonsuz toplamı biçiminde<br />

yazarak sadeleştirmeler yapmak yerinde olacaktır.<br />

2.13.1 Çalışma Soruları<br />

1.<br />

2.<br />

sin x sin 2x sin nx cos (n + 1) x<br />

+<br />

cos x cos 2 + · · · +<br />

x cos n = cot x −<br />

x sin x · cos n x<br />

eşitliğini kanıtlayınız.<br />

1<br />

cos 0 · cos 1 + 1<br />

cos 1 · cos 2 + · · · + 1<br />

cos 88 · cos 89 = cos 1<br />

sin 2 1<br />

eşitliğini kanıtlayınız.<br />

3. n ∈ Z + ve a ∈ R <strong>olm</strong>ak üzere a/π bir rasyonel sayı olarak veriliyor. Buna<br />

göre,<br />

4.<br />

1<br />

cos a − cos 3a + 1<br />

cos a − cos 5a + · · · + 1<br />

cos a − cos (2n + 1) a<br />

toplamının eşitini bulunuz.<br />

n∑<br />

( )<br />

tan −1 1<br />

n<br />

2k 2 = tan−1 n + 1<br />

k=1<br />

eşitliğini kanıtlayınız.


2.13. TRİGONOMETRİDE SONSUZ TOPLAM VE FARKLAR 125<br />

5. a ≠ kπ, k ∈ Z <strong>olm</strong>ak üzere,<br />

toplamının eşitini bulunuz.<br />

∞∑<br />

n=1<br />

1<br />

2 n tan a 2 n<br />

6.<br />

eşitliğini kanıtlayınız.<br />

∞∑<br />

3 n−1 sin 3 a 3 n = 1 (a − sin a)<br />

4<br />

n=1<br />

7. n = 2, 4, 6, · · · , 180 <strong>olm</strong>ak üzere verilen n · sin n ◦ sayılarının aritmetik<br />

ortalamasının cot 1 ◦ olduğunu kanıtlayınız.<br />

8. m, n ∈ Z + ve x ≠ kπ m<br />

olduğuna göre,<br />

eşitliğini kanıtlayınız.<br />

1<br />

sin 2x + 1<br />

sin 4x + · · · + 1<br />

sin 2 n x = cot x − cot 2n x<br />

9.<br />

tan 1<br />

cos 2 + tan 2<br />

cos 4 + · · · + tan 2n<br />

cos 2 n+1<br />

toplamını hesaplayınız.<br />

10. Sıfırdan farklı her x reel sayısı için,<br />

eşitliğini kanıtlayınız.<br />

∞∏<br />

n=1<br />

cos x 2 n = sin x<br />

x<br />

11. n > 1 ve n ∈ Z için<br />

cos<br />

eşitliğini kanıtlayınız.<br />

2π<br />

2 n − 1 · cos 4π<br />

2 n − 1 · · · · · cos 2 n π<br />

2 n − 1 = 1<br />

2 n


126 BÖLÜM 2. KONULAR<br />

12.<br />

n∏<br />

k=1<br />

çarpımının eşitini bulunuz.<br />

(<br />

1 − tan 2 2 k )<br />

π<br />

2 n + 1<br />

13. (1 − cot 1 ◦ ) (1 − cot 2 ◦ ) (1 − cot 3 ◦ ) · · · (1 − cot 44 ◦ ) çarpımını hesaplayınız.<br />

14.<br />

( 1<br />

2 + cos π 20<br />

) ( 1<br />

2<br />

) ( ) ( )<br />

3π 1 9π 1 27π<br />

+ cos + cos + cos = 1 20 2 20 2 20 16<br />

eşitliğini kanıtlayınız.<br />

15. n ∈ Z + , x ≠ 2 k+1 ( π<br />

3 + lπ) ve k = 1, 2, · · · , n, l ise<br />

n∏ (1 − 2 cos x )<br />

2 k<br />

k=1<br />

çarpımının sonucunu bulunuz.<br />

16.<br />

eşitliğini kanıtlayınız.<br />

n∑<br />

)<br />

2π · 3k<br />

(1 + 2 cos<br />

3 n = 1<br />

+ 1<br />

k=1<br />

2.13.2 Çözümler<br />

1. sin kx·sin x = cos kx·cos x−cos (k + 1) x ise bu eşitsizliğin iki tarafınıda<br />

sin x · cos k x ile bölelim. Buradan<br />

sin kx<br />

cos k x =<br />

cos kx cos (k + 1) x<br />

sin x · cos k−1 −<br />

x sin x · cos k x


2.13. TRİGONOMETRİDE SONSUZ TOPLAM VE FARKLAR 127<br />

olacaktır. Buna göre,<br />

= sin x sin 2x<br />

+<br />

cos x cos 2 x<br />

= cos x<br />

sin x − cos 2x<br />

= cot x −<br />

olarak bulunacaktır.<br />

sin 3x sin nx<br />

+<br />

cos 3 + · · · +<br />

x cos n x<br />

cos 2x<br />

sin x · cos x −<br />

sin x · cos x +<br />

cos (n + 1) x<br />

sin x · cos n x<br />

2. Eşitliğin iki tarafınıda sin 1 ile çarparsak,<br />

cos 3x<br />

sin x · cos 2 x + · · · +<br />

sin 1<br />

cos 0 cos 1 + sin 1<br />

cos 1 cos 2 + · · · + sin 1<br />

cos 88 cos 89 = cos 1<br />

sin 1<br />

olacaktır. Buradan<br />

sin (1 − 0) sin (2 − 1)<br />

+<br />

cos 1 cos 0 cos 2 cos 1<br />

eşitliği elde edilir. Burada<br />

+ · · · +<br />

sin (89 − 88)<br />

cos 89 cos 88 = cot 1<br />

sin (a − b)<br />

= tan a − tan b<br />

cos a cos b<br />

ise son yazdığımız eşitliğin sol tarafı<br />

olacaktır.<br />

k=1<br />

89∑<br />

k=1<br />

[tan k − tan (k − 1)] = tan 89 − tan 0 = cot 1<br />

3. Soruda verilen eşitliği düzenlersek,<br />

n∑ 1<br />

cos a − cos (2k + 1) a = 1 n∑ 1<br />

2 sin ka · sin (k + 1) a<br />

eşitliğini elde ederiz. Bir önceki problemin çözümünde olduğu gibi, eğer<br />

sin a ile çarparsak, toplam<br />

1<br />

n∑ sin ((k + 1) a − ka)<br />

2 sin ka · sin (k + 1) a = 1 n∑<br />

(cot ka − cot (k + 1) a)<br />

2<br />

k=1<br />

k=1<br />

k=1<br />

= 1 (cot a − cot (n + 1) a)<br />

2<br />

cos nx<br />

sin x · cos n−1 x − cos<br />

sin


128 BÖLÜM 2. KONULAR<br />

olacağından sorudaki toplamın eşiti,<br />

olarak bulunur.<br />

(cot a − cot (n + 1) a)<br />

2 sin a<br />

4. Eğer soruda verilen eşitliğin sol tarafını düzenlesek<br />

5.<br />

n∑<br />

k=1<br />

tan −1 1<br />

2k 2 =<br />

n∑<br />

k=1<br />

( )<br />

(2k + 1) − (2k − 1)<br />

tan −1 1 − (2k − 1) (2k + 1)<br />

eşitliğini elde ederiz. Eğer arktanjant için fark formülünü kullanırsak<br />

n∑ (<br />

tan −1 (2k + 1) − tan −1 (2k − 1) )<br />

k=1<br />

olacaktır. Bu toplam da<br />

( )<br />

2n + 1 − 1<br />

tan −1 (2n + 1) − tan −1 (1) = tan −1 1 + 2n + 1<br />

olacaktır.<br />

( ) n<br />

= tan −1 n + 1<br />

tan x = 1<br />

tan x − 1 − tan2 x<br />

= 1<br />

tan x tan x − 2<br />

tan 2 = cot x − 2 cot 2x<br />

x<br />

olduğuna göre,<br />

olacaktır. Bu toplam da<br />

olacağından istenilen değer<br />

olarak bulunur.<br />

1<br />

2 n tan a 2 n = 1<br />

2 n cot a 2 n − 1<br />

2 n−1 cot a<br />

2 n−1<br />

lim x cot ax = lim cos ax<br />

x<br />

x→0 x→0 sin ax = 1 a<br />

1<br />

a − cot a


2.13. TRİGONOMETRİDE SONSUZ TOPLAM VE FARKLAR 129<br />

6. sin 3x = 3 sin x − 4 sin 3 x olduğuna göre,<br />

3 n−1 sin 3 a 3 n = 1 (<br />

3 n sin a 4 3 n − 3n−1 sin a )<br />

3 n−1<br />

olacaktır. Teleskopik toplamdan<br />

olacaktır.<br />

7. Öncelikle,<br />

1<br />

4 lim<br />

n→∞<br />

sin a<br />

3 n<br />

1<br />

− 1 4 sin a = 1 (a − sin a)<br />

4<br />

3 n<br />

90∑<br />

k=1<br />

2k sin (2k) sin 1<br />

ifadesini hesaplayalım. Çarpımdan, farkların toplamına gidersek<br />

90∑<br />

k=1<br />

k cos (2k − 1) − k cos (2k + 1) =<br />

=<br />

∑90<br />

k=1<br />

90<br />

(k − (k − 1)) cos (2k − 1) − 90 cos 181<br />

∑<br />

cos (2k − 1) + 90 cos 1<br />

olacaktır. cos(180 − x) = − cos x olduğuna göre, son toplamdaki elemanlar<br />

ikili ikili sadleşecektir. Sonuç olarak geriye sadece 90 cos 1 kalacaktır.<br />

Burayıda sin 1 ile bölersek<br />

olacaktır.<br />

90 cos 1<br />

sin 1<br />

k=1<br />

= cot 1<br />

8.<br />

1<br />

sin 2x = 2 cos2 x − ( 2 cos 2 x − 1 )<br />

sin 2x<br />

=<br />

2 cos 2 x<br />

2 sin x cos x − 2 cos2 x − 1<br />

sin 2x<br />

= cos x cos 2x<br />

− = cot x − cot 2x<br />

sin x sin 2x


130 BÖLÜM 2. KONULAR<br />

olduğuna göre, soruda verilen eşitliğin sol tarafındaki teleskopik toplam,<br />

yine bir teleskopik farklar toplamına gideceğinden, eşiti<br />

olacaktır.<br />

cot x − cot 2 n x<br />

9.<br />

tan a<br />

cos 2a = tan a ( 1 + tan 2 a )<br />

1 − tan 2 a<br />

=<br />

= 2 tan a − tan a ( 1 − tan 2 a )<br />

1 − tan 2 a<br />

2 tan a<br />

1 − tan 2 − tan a = tan 2a − tan a<br />

a<br />

olduğuna göre soruda verilen toplam,<br />

tan 2 − tan 1 + tan 4 − tan 2 + · · · + tan 2 n+1 − tan 2 n = tan 2 n+1 − tan 1<br />

olacaktır.<br />

10. Sinüs fonksiyonunda yarımaçı formülünden,<br />

olacaktır. Bundan dolayı,<br />

cos u =<br />

sin 2u<br />

2 sin u<br />

∞∏<br />

cos x 2 n = lim<br />

n=1<br />

k→∞<br />

n=1<br />

eşitliğini elde ederiz. Bu eşitliğe göre,<br />

k∏<br />

cos x 2 n = lim<br />

k∏<br />

k→∞<br />

n=1<br />

x<br />

2 n−1<br />

sin x<br />

2 n<br />

1 sin<br />

2<br />

lim<br />

k→∞<br />

1<br />

2 k sin x<br />

sin x 2 k<br />

= sin x<br />

x<br />

lim<br />

k→∞<br />

x<br />

2 k<br />

sin x 2 k<br />

= sin x<br />

x<br />

olacaktır.<br />

11. Eğer eşitliğin sol kısmını<br />

2 n sin 2π<br />

2 n − 1


2.13. TRİGONOMETRİDE SONSUZ TOPLAM VE FARKLAR 131<br />

ile çarpıp sinüs formülünü uygularsak<br />

= 2 n sin 2π<br />

2 n − 1 · cos 2π<br />

2 n − 1 · cos 4π<br />

2 n − 1 · · · · · cos 2 n π<br />

2 n − 1<br />

= 2 n−1 sin 4π<br />

2 n − 1 · cos 4π<br />

2 n − 1 · · · · · cos 2 n π<br />

2 n − 1<br />

)<br />

= sin 2n+1 π<br />

2 n − 1 = sin (<br />

2π +<br />

olacaktır. Eğer sonucu<br />

2π<br />

2 n − 1<br />

( ) 2π<br />

2 n sin<br />

2 n − 1<br />

ile bölersek, istenen eşitliği elde etmiş oluruz.<br />

= sin<br />

( 2π<br />

2 n − 1<br />

)<br />

12.<br />

ise<br />

olacaktır. Buradanda<br />

1 − tan 2 x = 2 tan x<br />

tan 2x<br />

1 − tan 2 2 k π<br />

2 n + 1 = 2 tan 2k π<br />

2 n +1<br />

tan 2k+1 π<br />

2π<br />

tan<br />

n 2<br />

2 n +1<br />

tan 2n+1 π<br />

2 n +1<br />

2 n +1<br />

= 2 n tan 2π<br />

(<br />

tan 2π −<br />

2 n +1<br />

2π<br />

2 n +1<br />

) = −2 n<br />

13.<br />

olacaktır.<br />

1 − cot x = √ 2<br />

(cos π 4 − cos π )<br />

4 cot x = √ sin (x − π/4)<br />

2<br />

sin x<br />

olacağından,<br />

22 sin (−44)<br />

(1 − cot 1) (1 − cot 2) · · · (1 − cot 44) = 2 ·<br />

sin 1<br />

= 2 22<br />

olacaktır.<br />

sin (−43)<br />

sin 2<br />

· · · · ·<br />

sin (−1)<br />

sin 44


132 BÖLÜM 2. KONULAR<br />

14.<br />

cos 3x<br />

cos x = 4 cos2 x − 3 = 2 (1 + cos 2x) − 3 = 2 cos 2x − 1<br />

olacaktır. Dolayısıyla,<br />

1<br />

2 + cos x = −1 2<br />

( (<br />

cos<br />

3<br />

2 x + 3π 2<br />

cos ( x<br />

2 + π )<br />

2<br />

olacaktır. Buna göre soruda verilen çarpım<br />

))<br />

= 1 sin 3 2 x<br />

2 sin x 2<br />

= 1 16<br />

= 1 16<br />

3π sin<br />

40 ·<br />

sin π 40<br />

81π sin<br />

40 ·<br />

sin π 40<br />

·<br />

9π sin<br />

40<br />

sin 3π<br />

40<br />

= 1<br />

16<br />

·<br />

sin<br />

27π<br />

40<br />

sin 9π<br />

40<br />

·<br />

sin<br />

81π<br />

40<br />

sin 27π<br />

40<br />

olacaktır.<br />

15.<br />

1 − 2 cos a = 1 − 4 cos2 a<br />

1 + 2 cos a<br />

ise sorumuzda ki sonsuz çarpım<br />

olacaktır.<br />

(−1) n ·<br />

1 − 2 (1 + cos 2a)<br />

=<br />

1 + cos a<br />

1 + 2 cos 2a<br />

= −<br />

1 + cos a<br />

1 + cos x<br />

1 + 2 cos x<br />

2 n<br />

16.<br />

1 + 2 cos a k = 1 + 2 ( 1 − sin 2 a k<br />

)<br />

= 3 − 4 sin 2 a k<br />

= sin 3a k<br />

sin a k<br />

ve buradanda,<br />

a k = 3k · 2π<br />

3 n + 1


2.14. KUŞ, GÜVERCİN, YUVA... 133<br />

olacaktır. Buradan sonsuz çarpımımız,<br />

sin 3a n<br />

sin a 1<br />

olacaktır. Son yazılan kesrin payı<br />

(<br />

sin 3π −<br />

3π )<br />

3 n + 1<br />

= sin 3n+1 π<br />

3 n +1<br />

sin<br />

ise eşitliğin sonucunun 1 olacağı açıktır.<br />

3π<br />

3 n +1<br />

= sin 3π<br />

3 n + 1<br />

EAT SLEEP DO MATH<br />

2.14 Kuş, Güverc˙ın, Yuva...<br />

Güvercin Yuvası İlkesi her nekadar isminde güvercinlere yer versede aslında<br />

bu güzel canlılarla uzaktan yakından bir alakası yoktur. Ancak temel prensibin<br />

kavranması için verilen örneklerde hep bu canlıyla alakalı örneklemeler yapılmıştır.<br />

Peki nedir bu ilke Açıklayalım. Eğer (n + 1) tane nesneyi (Güvercini),<br />

n tane kutu (Güvercin yuvası) arasından seçersek, bu nesnelerden en az iki<br />

tanesi aynı kutudan gelmiş olacaktır. Veya İstanbul ili sınırları içerisinde yaşayan<br />

ve saç tellerinin sayısı aynı olan en az iki kişi bulabiriz. Yazdığımız bu<br />

iki önermede güvercin yuvası ilkesinin kavranması için yeterlidir. Yada benzer<br />

biçimde, herhangi bir ormandan seçilecek en az iki ağacın yaprak sayılarının<br />

aynı <strong>olm</strong>asını sağlayan yeterli ve gerekli şartlar nelerdir Sorusunun cevabıda<br />

yine aynı ilkeyle bulunacaktır. Şimdi bu ilkenin sorular üzerindeki uygulamasını<br />

görmek için örnekleri inceleyelim.<br />

Örnek. 1, 2, 3, · · · , 99, 100 sayıları arasında 51 tane sayı seçiliyor. Buna<br />

göre bu 51 sayıdan iki tanesinin aralarında asal olduğunu kanıtlayınız.


134 BÖLÜM 2. KONULAR<br />

Çözüm.<br />

Varsayalım bu 100 sayı arasından<br />

(1, 2), (3, 4), · · · , (99, 100)<br />

şeklinde ikililer oluşturalım. 51 tane sayı seçileceğine göre, güvercin yuvası<br />

ilkesi uyarınca bu sayılar arasında (k, k + 1) ikilisi bulunacaktır. Buna göre<br />

k + 1 ve k sayılarını bölen p asalı varsa, bu asal (k + 1) − k = 1 sayısını<br />

da bölecektir ki, bu durum çelişki olur. Demek ki, (k, k + 1) sayılarının ortak<br />

böleni yoktur.<br />

Örnek. 1, 2, 3, · · · , 99, 100 sayıları arasından 51 tane sayı seçiliyor. Buna<br />

göre, bu 51 sayı arasında ki sayılardan birinin, bir diğer sayıyı böleceğini gösteriniz.<br />

Çözüm. 50 tek sayıyı, 1, 3, 5, · · · , 99 sayılarını alalım. Her biri için<br />

bir kutu düşünelim. Bu kutular hem sayının kendisini hem de sayı ile 2’nin<br />

kuvvetinin çarpımını içersin. Buna göre ilk kutu 1, 2, 4, 8, 16, · · · , ikinci kutu<br />

3, 6, 12, 24, 48, · · · olarak gidecektir. Eğer 51 sayı seçersek güvercin yuvası ilkesine<br />

göre bu iki sayı mecburen aynı kutudan gelecektir. Yani sayılardan biri<br />

2 n k iken diğeri 2 m k olacak ve aynı k çarpanına sahip olacaklardır. Dolayısıyla<br />

sayılardan biri diğerini bölecektir.<br />

Çözülen iki örnektede farkedeceğiniz üzere, her nekadar soru içerisinde<br />

bahsi geçmesede kutu formuyla çözüme gidilmiştir. Demek ki çözümdeki anahtar<br />

basamak kutu prensibini kavramak olacaktır. Şimdi aşağıda verilen örnekleri<br />

inceleyerek bu formu daha iyi kavramaya çalışınız.<br />

Örnek.<br />

Seçilen herhangi dokuz faklı reel sayı arasında<br />

0 <<br />

(a − b)<br />

1 + ab < √ 2 − 1<br />

eşitsizliğini sağlayan bir (a, b) ikilisinin bulunabileceğini kanıtlayınız.<br />

Çözüm. Verilen eşitsizliğin ortasındaki ifadeyi incelersek bu ifadenin<br />

tan(x − y) formülüne benzediğini görebiliriz. Yani ifade aslında<br />

tan (x − y) =<br />

tan x − tan y<br />

1 + tan x tan y


2.14. KUŞ, GÜVERCİN, YUVA... 135<br />

biçimindedir. Şimdi artık sorumuza biraz daha trigonometrik yaklaşalım. Buna<br />

göre,(−π/2, π/2] aralığını 8 eşit uzunlukta parçaya ayıralım. Bu parçaların<br />

(<br />

− π ] (<br />

2 , −3π , − 3π ] ( π<br />

8 8 , −π , · · · ,<br />

4 4 , 3π ] ( 3π<br />

,<br />

8 8 , π ]<br />

2<br />

olacağı açıktır. Burada x i = arctan a i , i = 1, 2, · · · , 9 olacak biçimde sayılar<br />

seçelim. Güvercin yuvası prensibine göre, x i ’lerden iki tanesi, biz bunlara x j ve<br />

x k diyelim, x j > x k olacak biçimde ayırdığımız 8 aralıktan birinde olacaktır.<br />

Buna göre<br />

0 < x j − x k < π 8<br />

ve dolayısıyla<br />

olacaktır.<br />

0 < tan (x j − x k ) = a j − a<br />

(<br />

k π<br />

)<br />

< tan<br />

1 + a j a k 8<br />

= √ 2 − 1<br />

Örnek. Bir kenarı 1 birim olan bir karenin içine yerleştirilen köşeleri kare<br />

üzerinde bulunan bir üçgenin karenin merkezini içermemesi isteniyor. Buna<br />

göre üçgenin bir kenarının 1 birimden küçük <strong>olm</strong>ası gerektiğini kanıtlayınız.<br />

Çözüm. Karenin merkezini C noktası olarak alıp, kareyi 4 eşit parçaya<br />

bölelim. Eğer çizilen üçgenin C noktasını içermemesi isteniyorsa, üçgenin köşelerinin<br />

komşu iki parçanın içerisinde <strong>olm</strong>ası gerekir. Eğer üçgen oluşturmak<br />

istiyorsak alacağımız 3 noktanında komşu kareler içerisinde <strong>olm</strong>ası gerekecektir.<br />

Ancak 3 noktadan 2 tanesini aynı kareden alacağımıza göre bu noktalar<br />

arası uzaklık kesinlikle 1 birimden küçük olacaktır.<br />

2.14.1 Çalışma Soruları<br />

1. 1, 2, 3, · · · , 99, 100 sayıları arasından 11 tane sayı seçiliyor. Buna göre,<br />

bu 11 sayının altkümelerinden en az iki tanesinin elemanları toplamının<br />

aynı olacağını kanıtlayınız.<br />

2. Varsayalım koordinatları birer tamsayı olan 9 nokta üç boyutlu koordinat<br />

düzleminde seçilsin. Buna göre, bu noktaların birleştirilmesiyle oluşturulan<br />

doğru parçalarından seçilen bir tanesinin üçüncü bir tamsayı<br />

koordinatlı noktayı içerdiğini kanıtlayınız.


136 BÖLÜM 2. KONULAR<br />

3. Altı kişilik bir grup içinde ya herhangi üç kişinin birbirini tanıdığını yada<br />

en az üç kişinin birbirini tanımadığını kanıtlayınız.<br />

4. Seçilecek herhangi bir 16 basamaklı doğal sayının ardışık basamaklarının<br />

çarpımının birer tamkare <strong>olm</strong>asını sağlayan en bir zincirin bulunduğunu<br />

kanıtlayınız. Örneğin<br />

2.14.2 Çözümler<br />

12 }{{} 343 21524716 }{{} 91 8 gibi.<br />

36<br />

9<br />

1. Onbir elemanlı bir kümenin 2 11 − 2 = 2046 tane eleman sayısı 11’den az<br />

olan boş kümeden farklı altkümesi vardır. Bu alt kümelerden herhangi<br />

birinin elemanları toplamı en fazla<br />

91 + 92 + 93 + · · · + 99 + 100 = 955<br />

olacaktır. Buna göre, güvercin yuvası ilkesi uyarınca, elemanları toplamı<br />

birbirine eşit olan iki, tane alt küme bulunacaktır. Eğer bu kümelerin ortak<br />

elemanları varsa, bu elemanı iki kümedende çıkararak kesişimleri boş<br />

üme olan ve elemanları toplamları aynı olan iki alt küme elde edebiliriz.<br />

2. Dokuz nokta için her bir (x, y, z) koordinatları ya tek yada çift olacaktır.<br />

Bu durumda 2 3 = 8 farklı teklik çiftlik durumu ortaya çıkar. Buna göre<br />

bu dokuz noktadan iki tanesi mecburen koordinat pariteleri eşit noktalar<br />

olacaklardır. Ve dolayısıyla orta noktalarının koordinatlarıda birer<br />

tamsayı olacaktır.<br />

3. Önce elimizdeki 6 kişiyi düzgün bir altıgenin köşelerine yerleştirdiğimiz<br />

düşünelim. Eğer iki kişi biribirini tanıyorsa bu iki köşeyi birleştiren köşegen<br />

kırmızıya boyansın, tanımıyorsa da maviye boyansın. Aslında sorunun<br />

çözümü ile bizim altıgenin köşeleriyle tüm kenarları aynı renk olan<br />

bir üçgen elde edip edemeyeceğimiz sorusunun cevabı özdeşitir. Herhangi<br />

bir köşeyi alalım, güvercin yuvası ilkesi uyarınca, bu köşeden çıkan üç<br />

köşegen aynı renk olsun. Bundan sonraki herbir köşegende rengi, üçgen<br />

oluşturmayacak biçimde seçmeye çalışsak bile mutlaka ortaya her defasında<br />

kenarları aynı renk olan bir üçgen ortaya çıkacaktır.


2.15. ÜSTEL DİYOFANT DENKLEMLERİ 137<br />

4. Varsayalım d 1 , d 2 , d 3 , · · · , d 11 sayımızın basamakları olsun. Eğer bu sayının<br />

basamaklarından her hangi biri 0, 1, 4 veya 9 ise problemin çözümü<br />

açıktır. Varsayalım sayımızın basamakları 2, 3, 5, 6, 7 ve 8 sayılarından<br />

oluşsun. Eğer x 0 = 1 ve x i değeride d 1 , d 2 , · · · , d i i = 1, · · · , 16 sayılarının<br />

çarpımı olsun. Her bir x i = 2 pi · 3 qi · 5 ri · 7 s i<br />

olacaktır. Burada üst<br />

değerler zaten ya tektik yada çift sayıladır. Dolayısıyla 2 4 = 16 farklı durum<br />

elde edilebilir. Güvercin yuvası ilkesi uyarınca p i , q i , r i , s i değerleri<br />

onyedi x i ’lerin ikisi için, varsayalım bu ikililerde x j ve x k olsun, teklik<br />

çiftlik bakımından aynı pariteye sahip olacaklardır. Buna göre<br />

tamkare olacaktır.<br />

d j+1 · d j+2 · · · · · d k = x k<br />

x j<br />

2.15 Üstel D˙ıyofant Denklemler˙ı<br />

Üstel diyofant denklemlerinin diğer diyofant denklemlerinden farkları değişkenlerin<br />

genelde üstel durumda bulunmasıdır. Genel bir çözüm yöntemleri<br />

<strong>olm</strong>amasına rağmen bazı cebirsel oyunlarla çözüm yolları geliştirilebilir. Aşağıda<br />

verilen örnekleri incelediğiniz zaman tam olarak ne demek istediğimizi<br />

daha iyi anlayacaksınız. İlk olarak kolay bir örnekle başlayalım.<br />

Örnek. (x, y, z) ∈ Z + <strong>olm</strong>ak üzere verilen 3 x + 4 y = 5 z denklemini<br />

sağlayan tüm üçlüleri bulunuz.<br />

Çözüm I. Eğer soruda verilen eşitliğe (mod4) altında bakarsak 3 x ≡<br />

1 (mod4) dekliğini elde ederiz. Buna göre x = 2x 1 <strong>olm</strong>alıdır. Benzer biçimde<br />

5 z ≡ 1 (mod3) olduğundan z = 2z 1 alabiliriz. Bunları denklemde yerine koyarsak<br />

4 y = (5 z 1<br />

+ 3 x 1<br />

) (5 z 1<br />

− 3 x 1<br />

)<br />

olacaktır. Buradan 5 z 1<br />

+ 3 x 1<br />

= 2 s ve 5 z 1<br />

− 3 x 1<br />

= 2 t <strong>olm</strong>alıdır. s + t = 2y ve<br />

s > t olacağı açıktır. Eğer bu iki denklemi çözersek 5 z 1<br />

= 2 t−1 ( 2 s−t + 1 ) ve<br />

3 x 1<br />

= 2 t−1 ( 2 s−t − 1 ) bulunur. Bu iki eşitlikte de sol kısımlar birer tek sayıdır.<br />

Öyleyse t = 1 <strong>olm</strong>alıdır. Eğer s − t = u dersek 3 x 1<br />

= 2 u − 1 ve 5 z 1<br />

= 2 u + 1<br />

olacaktır. Eğer birinci denkleme (mod3) altında bakarsak u = 2u 1 bulunur.


138 BÖLÜM 2. KONULAR<br />

Sağ tarafı çarpanlara ayırırsak 2 u 1<br />

+ 1 = 3 α ve 2 u 1<br />

− 1 = 3β eşitlikleri elde<br />

edilir. Buradan 3 α − 3 β = 2 olduğuna göre α = 1 ve β = 0 olacaktır. Buradan<br />

u 1 = 1 ve u 2 = 2 olduğundan çözümde x = y = z = 2 olarak bulunur.<br />

Çözüm II. Bu sefer farklı bir yönden probleme yaklaşacağız. Birinci<br />

çözümde olduğu üzere x = 2x 1 ve z = 2z 1 eşitliklerini kullanırsak,<br />

(3 x 1<br />

) 2 + (2 y ) 2 = (5 z 1<br />

) 2<br />

eşitliğini elde ederiz. Eğer x = 3 x 1<br />

, y = 2 y , z = 5 z 1<br />

denkleminden<br />

x 2 + y 2 = z 2<br />

olarak alırsak Pisagor<br />

olur. Burada x, y, z değerleri aralarında asal ve y çift bir sayıdır. Pisagor üçlülerini<br />

kullanırsak değerlerimiz<br />

x = u 2 − v 2 , y = 2uv, z = u 2 + v 2<br />

olur. Bu eşitliklerin ikincisinden u ve v ’nin 2 ’nin kuvvetleri oldukları açıktır.<br />

Buna göre v = 1 <strong>olm</strong>alıdır. Çünkü birinci denklemden, 3’ün kuvvetleri 2 ’nin<br />

kuvvetleri ile bölünemezler. Demek ki,3 x 1<br />

= 2 y − 1 ve 5 z 1<br />

= 2 y + 1 olacaktır.<br />

İkinci denkleme (mod4) altında bakarsak y = 2y 1 olur. Buradan<br />

3 x 1<br />

= (2 y 1<br />

− 1) (2 y 1<br />

+ 1)<br />

olacağından çarpanlardan her ikisi de 3 ’ün kuvveti <strong>olm</strong>alıdır. Buna göre y 1 = 1<br />

ve x 1 = z 1 = 1 <strong>olm</strong>alıdır. Sonuç olarak, x = y = z = 2 olarak bulunur. Sıradaki<br />

sorumuz 2005 Romanya Takım Seçme Sınavı’ndan alınmıştır.<br />

Örnek.<br />

bulunuz.<br />

(x, y) ∈ Z + <strong>olm</strong>ak üzere 3 x = 2 x·y+1 eşitliğini sağlayan değerleri<br />

Çözüm. Denklemi yeniden yazarsak 3 x − 1 = 2 x · y olacaktır. Buna<br />

göre, (3 x − 1) ifadesinin asal çarpanlarından birisi ikinin kuvvetidir. Ve x değeri<br />

bu kuvvetten küçüktür. Şimdi bu kuvvetin ne kadar büyük olabileceğini<br />

araştıralım. Ayrıca, eğer n değeri tekse 3 n − 1 ≡ 2 (mod4) ve eğer n çift ise


2.15. ÜSTEL DİYOFANT DENKLEMLERİ 139<br />

3 n − 1 ≡ 0 (mod4) olacaktır. Buna göre x değişkenini 2 m (2n + 1) formunda<br />

yazabiliriz. Buna göre,<br />

3 x − 1 = 3 2m (2n+1) − 1 = ( 3 2n+1) 2 m − 1<br />

= ( 3 2n+1 − 1 ) ( 3 2n+1 + 1 ) m−1 ∏ ( (3<br />

2n+1 ) 2 k + 1)<br />

olur. Eğer bu eşitliğe (mod8) altında bakarsak ilk iki çarpan 2 ve 4 olacaktır.<br />

Buna göre (3 x − 1) içindeki 2’nin kuvvetleri (m + 2) tane olacaktır. Burada<br />

x ≤ m + 2 ise<br />

2 m (2n + 1) ≤ (m + 2) ve 2 m ≤ (m + 2)<br />

olacaktır. Burada m = 0, 1, 2 olabilir. Buna göre istenilen çözümler<br />

olacaktır.<br />

(1, 1), (2, 2), (4, 5)<br />

k=1<br />

2.15.1 Çalışma Soruları<br />

1. |3 x − 2 y | = 1 denklemini pozitif tamsayılarda çözünüz.<br />

2. (x, y) pozitif tamsayılar <strong>olm</strong>ak üzere verilen 3 x − 2 y = 7 denklemini<br />

çözünüz.<br />

3. n x + n y + n z = n t eşitliğini sağlayan tüm (n, x, y, z, t) pozitif tamsayı<br />

beşlilerini bulunuz.<br />

4. a. 3 x − y 3 = 1 denkleminin tüm negatif <strong>olm</strong>ayan tamsayı çözümlerini<br />

bulunuz.<br />

b. p bir tek asal sayı <strong>olm</strong>ak üzere verilen p x − x p = 1 denklemini<br />

sağlayan tüm negatif <strong>olm</strong>ayan (x, y) tamsayı ikililerini bulunuz.<br />

5. 1 n +9 n +10 n = 5 n +6 n +11 n eşitliğini sağlayan tüm n pozitif tamsayılarını<br />

bulunuz.


140 BÖLÜM 2. KONULAR<br />

2.15.2 Çözümler<br />

1. 3 x − 2 y = 1 için x = 1, y = 1 ve x = 2, y = 3 birer çözümdür. Eğer y ≥ 2<br />

ise x çift <strong>olm</strong>alıdır. x = 2z, z > 1 ve 3 z = 2m + 1, m > 1 alırsak<br />

2 y = (2m + 1) 2 − 1 = 4m(4m + 1)<br />

olacaktır ki, bu durum açık çelişkidir.<br />

3 x − 2 y = −1 için (mod8) altında bakarsak 3 x + 1 ifadesi 2 veya 4’e denk<br />

olacaktır. Buna göre y = 1 veya 2 olacaktır. Eğer kontrol edilirse x = 1,<br />

y = 2 tek çözüm olarak bulunacaktır.<br />

2. Varsayalım y 3 olsun. Buna göre<br />

3 x ≡ 7 (mod8)<br />

olacaktır. Ancak 3 x sayısı sadece 3 veya 1 sayılarına denk olabilir. Buna<br />

göre x = 1, 2 olabilir. x = 1 ise y /∈ Z ve x = 2, y = 1 olduğundan tek<br />

çözüm ikilisi (2, 1) olarak bulunur.<br />

3. Bu sorunun çözümünü iki farklı yoldan yapmayı deneyelim. Buna göre,<br />

Çözüm I. n = 1 durumun da denklemin sağlanamayacağı açıktır. n > 1<br />

için bakalım. Genelliği kaybetmeden x y z t olarak alabiliriz.<br />

Eşitliğin iki tarafını n x ile bölersek<br />

1 + n y−x + n z−x = n t−x<br />

denklemini elde ederiz. Eğer denkleme (modn) altında bakarsak y = x<br />

buluruz. Buna göre a = z − x ve b = t − x alırsak 2 + n a = n b olur. Eğer<br />

a = 0 ise n = 3, b = 1 ve çözümlerimiz de<br />

(n, x, y, z, t) = (3, x, x, x, x + 1)<br />

olur. Eğer a > 0 ise (modn) altında yine baktığımızda<br />

(n, x, y, z, t) = (2, x, x, x + 1, x + 2) , x ∈ N olur.


2.15. ÜSTEL DİYOFANT DENKLEMLERİ 141<br />

Çözüm II.<br />

Denklemimizi<br />

n x−t + n y−t + n z−t = 1<br />

olarak yazalım. Eşitliğin sol kısmındaki tüm terimler pozitif olduğuna<br />

göre x, y, z < t <strong>olm</strong>alıdır. Böylece sol kısım en fazla 3/n olabilir. n = 2<br />

ve 3 için bakarsak, n = 3 için x = y = z = t − 1 olacaktır. n = 2 için ise<br />

sadece bir terim 1/n olabilir. Buna göre, x = y = t − 2 ve z = t − 1 ve<br />

bu değerlerin permütasyonları bulunur.<br />

4. a. Eğer denklemde (0, 0) ikilisi alınırsa bunun bir çözüm olduğunu<br />

görmek zor <strong>olm</strong>ayacaktır. Denklemi düzenlersek<br />

olacağından eşitliğin sol tarafı<br />

3 x = y 3 + 1<br />

(y + 1) ( y 2 − y + 1 )<br />

olacaktır. Burada iki çarpanların ikisi de 3 ’ün kuvveti olacaktır.<br />

Yani çarpanlarımız<br />

y + 1 = 3 α ve y 2 − y + 1 = 3 β<br />

olacaktır. Burada ilk denklemin karesini alıp ikinci denklemden çıkarırsak<br />

elde edeceğimiz eşitlik<br />

( )<br />

3y = 3 β 3 2α−β − 1<br />

olacaktır. Soruda verilen denklemden y ’nin 3 ile bölünemediğini<br />

biliyoruz. Demek ki β = 1 olacaktır. Buna göre y 2 − y + 1 = 3 olacağından<br />

y = 2 olarak bulunur. Buradan da x = 2 olarak bulunur.<br />

Demek ki ikililerimiz<br />

olarak bulunacaktır.<br />

(x, y) = (2, 2) , (0, 0)


142 BÖLÜM 2. KONULAR<br />

b. Eğer (x, y) ikilisi soruda verilen denklemin bir çözümü ise<br />

p x = y p + 1 = (y + 1) ( y p−1 − y p−2 + · · · + y 2 − y + 1 )<br />

olacağından y + 1 = p n olacaktır. Eğer n = 0 ise y = x = 0 ve p<br />

herhangi bir asal sayı olacaktır. Eğer n 1 ise<br />

p x = (p n − 1) p + 1<br />

= p np − pp n(p−1) +<br />

( ( )<br />

p p<br />

p<br />

2)<br />

n(p−2) + · · · + p 2n + pp n<br />

p − 2<br />

olacaktır. p bir asal sayı olduğuna göre tüm katsayılar p ile bölünebilir.<br />

Dolayısıyla her bir terim p n+1 ile bölünebilecektir. Buna göre<br />

eşitliğin sağ tarafını bölen, p’nin en büyük kuvveti p n+1 olacaktır.<br />

Ve dolayısıyla x = n + 1 olacaktır. Buradan<br />

0 = p np − pp n(p−1) +<br />

bulunur. p = 3 için<br />

( p<br />

2)<br />

p n(p−2) + · · · +<br />

0 = 3 3n − 3 · 3 2n<br />

( ) p<br />

p 2n<br />

p − 2<br />

ise n = 1 ve x = y = 2 bulunur. p ≥ 5 için ( p<br />

p−2)<br />

ifadesi p 2 ile<br />

bölünemez. Dolayısıyla eşitliğin sağ kısmındaki son terim haricindekiler<br />

p 2n+2 ile bölünebilir. Terimlerin toplamı sıfır olduğundan,<br />

bu durum imkansızdır. Dolayısıyla çözümler sadece x = y = 0, tüm<br />

p asalları için ve x = y = 2, p = 3 olarak bulunur.<br />

5. Eğer her bir terimin birler basamağına bakarsak<br />

1 n , 5 n , 6 n , 10 n ve 11 n<br />

ifadelerinin son basamakları sırasıyla 1, 5, 6, 0, 1 olacaktır. Eşitliğin sağındaki<br />

terimlerin son basamakları toplamı 12 olduğuna göre, 9 n sayısının<br />

birler basamağı 1 <strong>olm</strong>alıdır. Dolayısıyla n ifadesi çift bir sayı olacaktır.<br />

Buradan n = 2 ve n = 4 birer çözüm olacaktır. Eğer n ≥ 6 ise<br />

11 n + 6 n + 5 n > 11 n = (10 + 1) n<br />

= 10 n + n10 n−1 + · · · + 1 ≥ 10 n + 9 n + 1 n


2.16. KALAN SINIFLARI [RESİDUES] 143<br />

olacaktır. Çünkü<br />

n · 10 n−1 ≥ 6 · 10 4 · 10 n−5 ≥ 9 5 · 10 n−5 ≥ 9 n<br />

eşitsizliği vardır. Buna göre çözümlerimiz n = 2 ve n = 4 olacaktır.<br />

2.16 Kalan Sınıfları [Res˙ıdues]<br />

Olimpiyat sınavlarında sorulan soruları eğer bir konu sınıflandırmasına tabi<br />

tutmaya kalkışsak herhalde bu sınavların <strong>olm</strong>azsa <strong>olm</strong>az soru gruplarından<br />

birisi de modüler aritmetik konusu olurdu. İleriki satırlarda anlatılan örnekleri<br />

ve problemleri daha iyi kavrayabilmeniz için bazı teoremleri bilmeniz gerekiyor.<br />

Bunun için ilk olarak, bilmeniz gereken bu teoremlerden başlayacağız.<br />

Teorem. [Çinli Kalanlar Teoremi] a 1 , a 2 , · · · , a r ikişerli olarak aralarında<br />

aralarında asal sayılar pozitif tamsayılar olarak veriliyor. Buna göre<br />

(a i , a j ) = 1 ve i ≠ j için<br />

x ≡ k 1 (moda 1 )<br />

x ≡ k 2 (moda 2 )<br />

· · · · · · · · ·<br />

x ≡ r 1 (moda r )<br />

· · · · · · · · ·<br />

denklik sisteminin mod(a 1 · a 2 · · · · · a r )’ye göre tek çözümü vardır. Bu çözümde<br />

( ) ( )<br />

( )<br />

a a a<br />

x = · b 1 · k 1 + · b 2 · k 2 + · · · + · b r · k r<br />

a 1 a 2 a r<br />

olacaktır. Bu eşitlikte<br />

a = a 1 · a 2 · · · · · a r<br />

ve ( a<br />

a i<br />

)<br />

· b i ≡ (moda i )


144 BÖLÜM 2. KONULAR<br />

olacaktır.<br />

Çinli kalanlar teoremini herhangi bir problem içerisinde nerede kullanacağınızı<br />

anlamanız oldukça kolaydır. Genelde sorulan tamsayının bir kaç tamsayı<br />

değeri ile bölümünden kalanlar verilir. Aşağıdaki örneğimiz 2009 Tübitak Mate<strong>mat</strong>ik<br />

Olimpiyatlarında sorulmuştu.<br />

Örnek. 1 ≤ n ≤ 455 ve n 3 ≡ 1 (mod455) koşullarını sağlayan kaç n<br />

tamsayı değeri vardır<br />

Çözüm.<br />

455 = 13 · 7 · 5 olduğuna göre,<br />

n 3 ≡ 1 (mod5)<br />

n 3 ≡ 1 (mod7)<br />

n 3 ≡ 1 (mod13)<br />

olduğuna göre,<br />

n ≡ 1 (mod5)<br />

n ≡ 1, 2, 4 (mod7)<br />

n ≡ 1, 3, 9 (mod13)<br />

olduğuna göre, 1 · 3 · 3 = 9 tane n tamsayı bulunur.<br />

Teorem. [Küçük Fer<strong>mat</strong> Teoremi] p asal sayı ve p ∤ a ise a p−1 ≡<br />

1 (modp) denkliği vardır.<br />

Şimdi bu teoremin ışığında aşağıda verilen örneği ve çözümünü inceleyelim.<br />

Örnek. a 1 = 4, a n = 4 a n−1<br />

, n > 1 olduğuna göre, a 100 tamsayısının 7 ile<br />

bölümünden kalanı bulunuz.<br />

Çözüm. Küçük Fer<strong>mat</strong> Teoremine göre, 4 6 ≡ 1 (mod7) olacaktır. Bunun<br />

yanında 4 n ≡ 4 (mod6) ve 4 n ≡ 4 + 6t olduğunu görmek zor değildir. Buna<br />

göre, soruda istenilen cevap<br />

a 100 ≡ 4 a 99<br />

≡ 4 4+6t ≡ 4 4 · (4 6) t<br />

≡ 4 (mod7)


2.16. KALAN SINIFLARI [RESİDUES] 145<br />

olacaktır.<br />

Teorem. [Wilson Teoremi] p bir asal sayı <strong>olm</strong>ak üzere, (p − 1)! ≡<br />

−1 (modp) denkliği vardır.<br />

Teorem. [Euler-φ Teoremi] a ve n birer pozitif tamsayı <strong>olm</strong>ak üzere,<br />

(a, n) = 1 ise a φ(n) ≡ 1 (modn) denkliği vardır.<br />

Euler-φ Teoreminde adı geçen φ fonksiyonu herhangi bir n pozitif tamsayı<br />

değeri için, n sayısından küçük eşit ve n ile aralarında asal olan sayıların<br />

sayısını vermektedir. Buna göre n = p a 1<br />

1 · pa 2<br />

2 · · · · · par r olarak asal çarpanlarına<br />

ayrılırsa, φ(n) ifadesinin eşiti<br />

olacaktır.<br />

φ (n) = n ·<br />

(1 − 1 p 1<br />

)<br />

·<br />

(1 − 1 )<br />

· · · · ·<br />

(1 − 1 )<br />

p 2 p r<br />

Örnek. 7 71000 sayısının son iki basamağını bulunuz.<br />

Çözüm. Önce φ(100) = φ(2 2 ) · φ(5 2 ) = (2 2 − 2)(5 2 − 5) = 40 değerini<br />

bululalım, çünkü ileri basamaklarda kullanacağız. Buna göre, Euler Teoreminden<br />

7 40 ≡ 1 (mod100) olacaktır. Benzer biçimde φ(40) = 16 olacağından<br />

7 16 ≡ 1 (mod40) olarak bulunur. 1000 = 16 · 62 + 8 olduğuna göre,<br />

olacaktır. Buna göre,<br />

7 1000 ≡ ( 7 16) 62<br />

7 8 ≡ 1 62 7 8 ≡ ( 7 4) 2<br />

≡ 1 (mod40)<br />

7 1000 = 1 + 40t<br />

olacaktır. Şimdi yaptığımız bu işlemleri ve sonuçları kullanarak çözüme gitmeye<br />

çalışırsak,<br />

7 71000 ≡ 7 1+40t ≡ 7 · (7 40) t<br />

≡ 7 (mod100)<br />

cevabına ulaşırız. Demek ki, sayının son iki basamağı 07 olacaktır.<br />

Bu konu ile alakalı bilmeniz gereken teoremler örnekleriyle birlikte yukarıda<br />

verildi. Eğer teoremlerden ispatlarını merak ettikleriniz varsa, herhangi bir<br />

sayılar teorisi kitabından kanıtlarını öğrenebilirsiniz. Şimdi, çalışma sorularını<br />

çözmeye geçebilirsiniz.


146 BÖLÜM 2. KONULAR<br />

2.16.1 Çalışma Soruları<br />

1. p bir asal sayı, w, n tamsayılar <strong>olm</strong>ak üzere verilen 2 p + 3 p = w n denklemini<br />

sağlayan tek n değerinin 1 olduğunu kanıtlayınız.<br />

2. m, n pozitif tamsayılar <strong>olm</strong>ak üzere √ 7 − m n > 0 ise √ 7 − m n > 1<br />

mn<br />

olduğunu kanıtlayınız.<br />

3. n ∈ Z <strong>olm</strong>ak üzere, 2 √ 28n 2 + 1 + 2 bir tamsayı ise, bu tamsayının tamkare<br />

olduğunu kanıtlayınız.<br />

4. {<br />

x 2 + 6y 2 = z 2<br />

6x 2 + y 2 = t 2<br />

denklem sisteminin (0, 0, 0, 0) dışında bir tamsayı dörtlüsü çözümü <strong>olm</strong>adığını<br />

kanıtlayınız.<br />

5. Bir tamkare tamsayının basamakları toplamının alabileceği olası tüm<br />

değerleri bulunuz.<br />

6. x 2 − y! = 2001 denklemini sağlayan tüm (x, y) pozitif tamsayı ikililerini<br />

bulunuz.<br />

7. Rakamlarının yerleri değiştirildiğinde 2 nin kuvvetine eşit olan kaç tane<br />

2 nin kuvveti olan tamsayı vardır<br />

8. 4444 4444 sayısının basamakları toplamı A ve A sayısının basamakları<br />

toplamı B ise B nin basamakları toplamı kaçtır<br />

9. y 2 = x 5 − 4 denkleminin tamsayılar kümesinde çözümünün <strong>olm</strong>adığını<br />

kanıtlayınız.<br />

10. 19 19 sayısının bir tamsayı küp ve bir tamsayı dördüncü kuvvetin toplamına<br />

eşit olamayacağını kanıtlayınız.<br />

11. x 2 + 3xy − 2y 2 = 122 denklemini sağlayan bir (x, y) tamsayı ikilisinin<br />

<strong>olm</strong>adığını kanıtlayınız.<br />

12. n > 1 <strong>olm</strong>ak üzere,


2.16. KALAN SINIFLARI [RESİDUES] 147<br />

a. 1! + 2! + 3! + · · · + n! toplamının ancak ve ancak n = 3 olduğunda<br />

bir tamsayının tam kuvvetine eşit olabileceğini kanıtlayınız.<br />

b. (1!) 3 + (2!) 3 + (3!) 3 + · · · + (n!) 3 toplamının ancak ve ancak n = 3<br />

olduğunda bir tamsayının tam kuvvetine eşit olabileceğini gösteriniz.<br />

13<br />

4 444<br />

sayısının ondalık yazılımda, sondan üçüncü yani yüzler basamağı kaçtır<br />

2.16.2 Çözümler<br />

1. Eğer p = 2 ise 2 2 + 3 2 = 13 ve n = 1 olacaktır. Eğer p ≥ 2 ise p tek sayı<br />

olacağından (2 + 3) ( 2 p−1 − 2 p−2 3 + · · · + 3 p−1) ifadesinden 5| (2 p + 3 p )<br />

diyebiliriz. Buna göre 5| w olacaktır. Eğer n > 1 ise 25| w n diyebiliriz.<br />

Dolayısıyla<br />

2 p + 3 p<br />

2 + 3 = 2p−1 − 2 p−2 · 3 + · · · + 3 p−1<br />

ifadesi de 5 ile bölünecektir. (−1) k · (2) p−k−1 · 3 k çarpımı (mod5) altında<br />

2 p−1 olduğuna göre üstte verilen toplam (mod5) altında p·2 p−1 olacaktır.<br />

Ancak, p · 2 p−1 ifadesi sadece p = 5 olduğunda 5 ile bölünebilir. Buna<br />

göre, 2 5 + 3 5 = 275 1 olacağından n = 1 olacaktır.<br />

2. Soruda verilen eşitsizliği düzenlersek, √ 7 − m n > 0 ise √ 7 > m n ve 7n2 −<br />

m 2 > 0 olacaktır. Pozitif bir tam kare ifadenin 7 ile bölümünden kalanlar<br />

sadece 0, 1, 2 veya 4 olabilir. 7n 2 −m 2 > 0 olduğuna göre 7n 2 −m 2 ≥ 7−<br />

4 = 3 olacaktır. Buradan √ 7n ≥ √ m 2 + 3 elde edilir. Bizim ispatlamaya<br />

çalıştığımız eşitsizlikte artık m + 1 m ≤ √ 7n olacaktır. Buradan,<br />

m + 1 m ≤ √ m 2 + 3<br />

olacaktır. Şimdi bu eşitsizliği kanıtlayalım. Aslında<br />

m 2 + 3 ≥ m 2 + 2 + 1 (<br />

m 2 = m + 1 ) 2<br />

m<br />

olduğu açıktır. Kanıt tamamlanır.


148 BÖLÜM 2. KONULAR<br />

3. Eğer 2 √ 28n 2 + 1+2 ifadesi bir tamsayı ise 28n 2 +1 = (2m + 1) 2 , m ≥ 0<br />

olacaktır. Buna göre<br />

olduğuna göre,<br />

28n 2 + 1 = 4m 2 + 4m + 1<br />

7n 2 = m (m + 1)<br />

olacaktır. m ve m + 1 aralarında asal olduklarına göre m = 7s 2 ve<br />

m + 1 = t 2 veya m = u 2 ve m + 1 = 7v 2 olacaktır. Burada ikinci durumun<br />

<strong>olm</strong>ası imkânsızdır. Çünkü (mod7)altında incelenirse kolaylıkla<br />

görülebilir. Demek ki m + 1 = t 2 olacağından,<br />

2 √ 28n 2 + 1 + 2 = 2 (2m + 1) + 2 = 4m + 4 = 4 (m + 1) = 4t 2 = (2t) 2<br />

olacaktır.<br />

4. Varsayalım soruda verilen dörtlü dışında da bir çözümümüz olsun. Ayrıca<br />

x, y, z, t sayılarının da ortak böleni <strong>olm</strong>asın. Buna göre soruda verilen<br />

sistemdeki iki denklemi toplarsak<br />

7 ( x 2 + y 2) = z 2 + t 2<br />

olacaktır. Tam kare bir sayının 7 ile bölümünden kalanlar yalnızca 0, 1, 2<br />

ve 4 olabilir. z 2 + t 2 ≡ 0 (mod7) ise z ve t sayılarının sırasıyla 7z 0 ve 7t 0<br />

formunda olacağı açıktır. Buna göre,<br />

7 ( x 2 + y 2) = (7z 0 ) 2 + (7t 0 ) 2<br />

olacağından x 2 +y 2 = 7 ( z 2 0 + t2 0)<br />

olacağından bu seferde x ve y sayıları 7<br />

ile kalansız bölünecektir. Buna göre, (x, y, z, t) = 7 olur. Ancak bu durum<br />

çelişkilidir. Çünkü biz çözümün en başında ortak bölenlerinin <strong>olm</strong>adığını<br />

kabul etmiştik. Demek ki sistemin yegane çözümü olacaktır.<br />

5. Bir tamsayının basamakları toplamı (mod9) altında sayının kendisine<br />

denktir. Dolayısıyla eğer kontrol edilirse tam kare bir sayı (mod9) altında<br />

0, 1, 4 veya 7 olacaktır. Buna göre elimizdeki sayımızın basamakları<br />

toplamı 9m, 9m + 1, 9m + 4, 9m + 7 formlarından biri olacaktır.<br />

Şimdi bu formları tek tek inceleyelim. Eğer n = 9m ise (10 m − 1) 2 =


2.16. KALAN SINIFLARI [RESİDUES] 149<br />

10 2m − 2 · 10 m + 1 sayısının basamakları toplamı n olacaktır ve sayımız<br />

9 · · · 980 · · · 01 formunda olacaktır. Eğer n = 9m + 1 ise (10 m − 2) 2 =<br />

10 2m − 4 · 10 m + 4 ve 9 · · · 960 · · · 04 olacaktır. Eğer m = 9m + 4 ise<br />

(10 m − 3) 2 = 10 2m − 6 · 10 m + 9 ve 9 · · · 940 · · · 09 olacaktır. Eğer n =<br />

9m−2 ise (10 m − 5) 2 = 10 2m −10·10 m +25 ve 9 · · · 900 · · · 025 olacaktır.<br />

6. y > 5 için y! sayısı 9 ile tam bölünür. Dolayısıyla y! + 2001 toplamı<br />

(mod9) altında 3 sayısına denktir. Ancak tamkare bir sayının 9 ile bölümünden<br />

kalanlar sadece 0, 1, 4, 7 olabilir. Demek ki, y = 1, 2, 3, 4 değerlerinden<br />

birini alabilir. Eğer kontrol edilirse (x, y) = (45, 4) ikilisi çözüm<br />

olarak bulunur.<br />

7. m < n için 2 m sayısı 2 n sayısının rakamlarının permüte edilmesiyle oluşsun.<br />

Bu iki sayının da aynı sayıda basamaklarının olduğu açıktır. Dolayısıyla<br />

2 n < 10 · 2 m olacaktır. Buna göre, n − m ≤ 3 olacaktır. Diğer<br />

taraftan 2 n ve 2 m sayıları (mod9) altında denktir. Dolayısıyla<br />

2 n − 2 m = 2 m ( 2 n−m − 1 )<br />

sayısı 9 ile bölünür. Fakat 2 m ve 9 aralarında asaldır ve n − m ≤ 3 için<br />

2 n−m − 1 ≤ 7 olduğundan, bu durum imkânsızdır. Demek ki, bu şekilde<br />

sayılar yoktur.<br />

8. Önce B sayısının basamakları toplamının küçük bir sayı olduğunu gösterelim.<br />

Buna göre,<br />

4444 4444 < 10000 5000<br />

eşitsizliğinden, 4444 4444 sayısı 20000 basamaktan azdır. Dolayısıyla, A <<br />

9 · 20000 = 180000 olacaktır. 180000 sayısından küçük sayılar arasında<br />

basamakları toplamı en büyük olan sayı 99999 dur ve basamakları toplamı<br />

45 tir. Dolayısıyla B ≤ 45 olacaktır. Buna göre B sayısının basamakları<br />

toplamı 39 sayısından 12 olur. Demek ki, bizim aradığımız sayı<br />

12 den küçük olacaktır. Diğer taraftan, tüm sayılar (mod9) altında basamakları<br />

toplamına denktir. Demek ki, bizim aradığımız sayı 4444 4444<br />

sayısına (mod9)altında denktir. Buna göre,<br />

4444 4444 ≡ 7 4444 ≡ ( 7 8) 555<br />

· 7 4 ≡ 49 · 49 ≡ 13 · 13 ≡ 7 (mod9)


150 BÖLÜM 2. KONULAR<br />

olacaktır. Demek ki B sayısının basamakları toplamı da (mod9) altında<br />

7 olacaktır. 12 sayısından küçük ve (mod9) altında 7 sayısına denk olan<br />

tek sayı 7 olduğuna göre, istenilen cevap 7 olur.<br />

9. Denklemimize (mod11) altında bakalım. Eğer (x, 11) = 1 ise<br />

(<br />

x<br />

5 ) 2<br />

= x 10 ≡ 0 veya 1 (mod11)<br />

olacaktır. Buna göre, x 5 ≡ 0, −1 veya 1 (mod11) olacaktır. Demek ki,<br />

eşitliğin sağ kısmı (mod11) altında 6, 7 veya 8 olacaktır. Ancak y 2 ≡<br />

0, 1, 3, 4, 5, 9 (mod11) olabilir. Demek ki, bu eşitliği sağlayan (x, y) tamsayı<br />

değerleri yoktur.<br />

10. Varsayalım soruda verilen eşitlik sağlansın ve x, y tamsayılar <strong>olm</strong>ak üzere,<br />

x 3 + y 4 = 19 19<br />

olsun. Şimdi toplamı oluşturan bileşenleri ayrı ayrı inceleyelim.<br />

x 3 ≡ 0, 1, 5, 8, 12 (mod13) ve y 4 ≡ 0, 1, 3, 9 (mod13)<br />

olduğuna göre 19 19 ≡ 6 19 ≡ 6 12 · 6 7 ≡ 7 (mod13) olarak bulunur. Buna<br />

göre,<br />

x 3 + y 4 ≡ 7 (mod13)<br />

denkliğini sağlayan (x, y) ikilisi <strong>olm</strong>adığından denklemin çözümü yoktur.<br />

11. Denklemin her iki tarafını 4 ile çarpıp düzenlersek,<br />

(2x + 3y) 2 − 17y 2 = 488<br />

eşitliğini elde ederiz. Bu denklemin her iki tarafını (mod17) altında incelersek<br />

(2x + 3y) 2 ≡ 12 (mod17)<br />

olacaktır. Ancak bir tam kare (mod17) altında sadece 0, 1, 4, 9, 16, 8, 2, 15, 13<br />

olabilir ancak 12 olamaz. Demek ki denklemi sağlayan (x, y) tam sayıları<br />

yoktur.<br />

12. Soruda olduğu gibi çözümüde şıklar halinde inceleyelim.


2.17. VİETE TEOREMI 151<br />

a. n = 3 için 1! + 2! + 3! = 3 2 olduğunu görmek zor değildir. Şimdi bu<br />

durumun tek olduğunu ve başka değeri bulunmadığını kanıtlayalım.<br />

Varsayalım 1! + 2! + 3! + · · · + n! = k m , m > 1 olsun. Burada<br />

n değerinin 9 dan büyük yada eşit olabileceğini kolaylıkla kontrol<br />

edebiliriz. Eğer n ≥ 5 ise 1!+2!+· · ·+n! sayısının son basamağı 3 ile<br />

biter dolayısıyla m = 2 olamaz. Çünkü tam kare sayılar 0, 1, 4, 5, 6<br />

veya 9 ile biter. Demek ki, m ≥ 3 <strong>olm</strong>alıdır. Bunun yanında n ≥ 9<br />

ise 1! + 2! + · · · + n! toplamı 3 ile bölünebilir. Dolayısıyla k sayısı da<br />

3 ile bölünebilir. Buna göre, k m sayısı da 27 ile bölünebilir. Demek<br />

ki, faktöriyellerin toplamı da 27 ile bölünebilir. Fakat, 27| a!, a ≥ 9<br />

olduğundan, toplam 1! + 2! + · · · + 8! toplamı (mod27) altında 9<br />

sayısına denk olacaktır. Bu durum açık çelişkidir. Demek ki n sadece<br />

3 <strong>olm</strong>alıdır.<br />

b. Eğer kontrol edilirse (1!) 3 +(2!) 3 +(3!) 3 = 15 2 olduğunu görmek zor<br />

değildir. Ayrıca n = 2, 4, 5 ve 6 için (1!) 3 +(2!) 3 +· · ·+(n!) 3 toplamı<br />

bir tamsayının tam kuvvetine eşit olamaz. n ≥ 7 için bu toplam<br />

(1!) 3 + (2!) 3 + (3!) 3 + (4!) 3 + (5!) 3 + (6!) 3 ≡ 7 (mod49)<br />

olacaktır. Ancak bu kalan bir tam kuvvetin kalanı <strong>olm</strong>az.<br />

13. Bu sorunun çözümü okuyucuya bırakılmıştır. Euler-φ foksiyonu ve Küçük<br />

Fer<strong>mat</strong> Teoremlerini uygulayarak kolaylıkla çözüme ulaşabilirsiniz.<br />

2.17 V˙ıete Teoremi<br />

Viete teoremi (Viyet diye okunur) genel olarak bir polinomun kökleri ile<br />

katsayıları arasındaki ilişkiyi açıklar. Bu teoremi reel katsayılı polinomlarda<br />

uygulayabildiğimiz gibi benzer biçimde complex katsayılı polinomlarda da kullanabiliriz.<br />

Buna göre, varsayalım x 1 ve x 2 değerleri P (x) = ax 2 +bx+c polinomunun<br />

kökleri olsun. Buna göre,<br />

ax 2 + bx + c = a (x − x 1 ) (x − x 2 ) = ax 2 − a (x 1 + x 2 ) x + ax 1 x 2


152 BÖLÜM 2. KONULAR<br />

olacağından, x 1 + x 2 = − b a ve x 1 · x 2 = c a olacaktır. Benzer biçimde eğer x 1, x 2<br />

ve x 3 değerleri P (x) = ax 3 + bx 2 + cx + d polinomunun kökleri ise, kökler<br />

arasında ki ilişki aşağıdaki gibi olacaktır.<br />

⎧<br />

x 1 + x 2 + x 3 = − b ⎪⎨<br />

a<br />

x 1 x 2 + x 2 x 3 + x 3 x 1 = c a<br />

⎪⎩ x 1 x 2 x 3 = − d a<br />

Eğer yukarıda verdiğimiz durumu genelleştirirsek, kökleri x 1 , x 2 , · · · , x n olan<br />

bir p (x) = a n x n + a n−1 x n−1 + a n−2 x n−2 + · · · + a 2 x 2 + a 1 x + a 0 polinomunun<br />

kökleri ve katsayıları arasındaki ilişki aşağıdaki gibi olacaktır.<br />

⎧<br />

x 1 + x 2 + · · · + x n = − a n−1<br />

a n<br />

⎪⎨ x 1 x 2 + x 1 x 3 + · · · + x n−1 x n = a n−2<br />

a n<br />

⎪⎩<br />

.<br />

x 1 x 2 x 3 · · · x n = (−1) n a 0<br />

a n<br />

Eğer yukarıda yazdığımız eşitlikleri yeniden düzenlersek en genel form olan,<br />

x 1 x 2 · · · x i + x 1 x 2 x 4 · · · x i+1 + · · · + x n−i+1 x n−i+2 · · · x n = (−1) i a n−i<br />

a n<br />

genel forma ulaşılır.<br />

Viete teoeremi her ne kadar bu ders notunun genel başlığı olsada bazı denklem<br />

çözümlerinde oldukça işimize yarayacak olan Newton-Girard Formülü’nü<br />

de vereceğiz. Bu formülün kullanımı her nekadar elzem <strong>olm</strong>asa da, çözümlerin<br />

daha pratik <strong>olm</strong>ası açısından önemlidir.<br />

2.17.1 Newton-Girard Formulæ<br />

Newton-Girard Formulæ. Eğer x n + a 1 x n−1 + · · · + a n = 0 denkleminin<br />

kökleri α 1 , α 2 , · · · , α n ve bu köklerin kuvvetleri toplamı<br />

S k = α k 1 + α k 2 + α k 3 + · · · + α k n, k ∈ N +


2.17. VİETE TEOREMI 153<br />

olarak verilirse, denklemin katsayıları ve k. kuvvetten köklerinin toplamları<br />

arasındaki ilişki aşağıdaki gibidir.<br />

Buna göre, genel denklem<br />

S 1 + a 1 = 0,<br />

S 2 + a 1 S 1 + 2a 2 = 0,<br />

S 3 + a 1 S 2 + a 2 S 1 + 3a 3 = 0,<br />

· · · · · · · · · · · · · · · · · · · · · · · · · · · = 0,<br />

S n + a 1 S n−1 + · · · + a n−1 S 1 + na n = 0<br />

S k + a 1 S k−1 + · · · + a n S k−n = 0, k > n<br />

biçiminde olacaktır. Şimdi bu denklemi kullanarak problemleri çözmeye çalışalım.<br />

Örnek.<br />

a, b, c sayıları x 3 − x 2 + 2 = 0 denkleminin kökleri ise<br />

a 2 + b 2 + c 2<br />

a 3 + b 3 + c 3<br />

a 4 + b 4 + c 4<br />

toplamlarının eşitlerini bulunuz.<br />

Çözüm. Her ne kadar bu sorunun çözümü üstte verdiğimiz formülle<br />

çözülebiliyor olsa da bu yöntemi sizin kullanacağınızı düşünerek çözüme daha<br />

da elementer bir yöntemle gidelim. Buna göre,<br />

a 2 + b 2 + c 2 = (a + b + c) 2 − 2 (ab + ac + bc) = 1 2 − 2 (0) = 1<br />

olacaktır. İkinci denklemin eşiti ise, x3 = x 2 − 2 olduğuna göre,<br />

a 3 + b 3 + c 3 = a 2 − 2 + b 2 − 2 + c 2 − 2 = a 2 + b 2 + c 2 − 6 = 1 − 6 = −5<br />

olacaktır. Üçüncü toplamın eşitini bulmak içinde x 3 = x 2 − 2 eşitliğin de<br />

eşitliğin her iki tarafını da x ile çarparsak, x 4 = x 3 − 2x ise<br />

a 4 +b 4 +c 4 = a 3 −2a+b 3 −2b+c 3 −2c = a 3 +b 3 +c 3 −2 (a + b + c) = −5−2 (1) = −7


154 BÖLÜM 2. KONULAR<br />

olarak sonuç bulunur.<br />

Örnek. Aşağıda verilen denklem sisteminin reel veya karmaşık tüm çözüm<br />

üçlülerini bulunuz.<br />

x + y + z = 3,<br />

x 2 + y 2 + z 2 = 3,<br />

x 3 + y 3 + z 3 = 3.<br />

Çözüm.<br />

Buna göre,<br />

Varsayalım x, y, z değerleri bir P (t) polinomunun kökü olsun.<br />

p (t) = (t − x) (t − y) (t − z) = t 3 − (x + y + z) t 2 + (xy + yz + zx) t − xyz<br />

olacaktır. Burada<br />

xy + yz + zx =<br />

(x + y + z)2<br />

2<br />

−<br />

(<br />

x 2 + y 2 + z 2)<br />

2<br />

= 9 2 − 3 2 = 3<br />

eşitliğini elde ederiz. Buradan<br />

x 3 + y 3 + z 3 − 3xyz = (x + y + z) ( x 2 + y 2 + z 2 − xy − yz − zx )<br />

eşitliğini kullanarak xyz = 1 olacaktır. Dolayısıyla<br />

p (t) = t 3 − 3t 2 + 3t − 1 = (t − 1) 3<br />

olacaktır. Buna göre soruda verilen sistemin tek çözümü x = y = z = 1<br />

olacaktır.<br />

Bu konuda öğrencilerin belkide bilmesi en önemli başlıklardan birisi de<br />

Lagrange Interpolasyon Tekniği’dir. Şimdi bu tekniği ve bu teknik ile çözülebilecek<br />

soruları inceleyelim.


2.17. VİETE TEOREMI 155<br />

2.17.2 Lagrange İnterpolasyon Tekn˙ığ˙ı<br />

Örnek. Kökleri 1, 2, 3 olan ve p(4) = 666 eşitliğini sağlayan kübik polinomu<br />

bulunuz.<br />

Soruda istenen polinom üçüncü dereceden olduğuna göre kesin-<br />

Çözüm.<br />

likle<br />

formunda <strong>olm</strong>alıdır. Buna göre,<br />

p (x) = a (x − 1) (x − 2) (x − 3) , a ∈ R<br />

666 = p (4) = a (4 − 1) (4 − 2) (4 − 3) = 6a, a = 111<br />

bulunacağından polinom p (x) = 111 (x − 1) (x − 2) (x − 3) olacaktır.<br />

Örnek. p (1) = 1, p (2) = 2, p (3) = 3, p (4) = 5 eşitliklerini sağlayan p (x)<br />

polinomunu bulunuz.<br />

Çözüm. Önceki örnekte çözüme elementer yöntemlerle ulaşabileceğimiz<br />

için Lagrange Interpolasyon Tekniği’ni uygulamamıştık. Ancak bu soruda bu<br />

tekniği hem uygulamasını öğreneceğiz hem de soruyu çözeceğiz. Lagrange tekniğine<br />

göre,<br />

p (x) = a (x) + 2b (x) + 3c (x) + 5d (x) ,<br />

polinomun da sırasıyla a (x) , b (x) , c (x) , d (x) birer kübik polinom olsun. Buna<br />

göre, a (1) = 1 ve a (x) polinomunun kökleri x = 2, 3, 4; b (2) = 1 ve b (x)<br />

polinomunun kökleri x = 1, 3, 4; c (3) = 1 ve c (x) polinomunun kökleri x =<br />

1, 2, 4; d (4) = 1 ve d (x) polinomunun kökleri x = 1, 2, 3 olacaktır.<br />

Bir önceki örnekte uyguladığımız tekniği yeniden burada uygularsak,<br />

(x − 2) (x − 3) (x − 4)<br />

a (x) = −<br />

6<br />

b (x) =<br />

(x − 1) (x − 3) (x − 4)<br />

2<br />

c (x) =<br />

(x − 1) (x − 2) (x − 4)<br />

−<br />

2<br />

d (x) =<br />

(x − 1) (x − 2) (x − 3)<br />

6


156 BÖLÜM 2. KONULAR<br />

polinomlarını elde ederiz. Dolayısıyla, soruda istenilen polinom<br />

p (x) = − 1 6 (x − 2) (x − 3) (x − 4)+(x − 1) (x − 3) (x − 4)−3 2 (x − 1) (x − 2) (x − 4)+5 (x − 1) (x −<br />

6<br />

olacaktır. isterseniz soruda verilen şartları sağlayıp sağlamadığını kontrol edebilirsiniz.<br />

Sırada ki örneğimize, Bilgi Üniversitesi Cahit Arf Mate<strong>mat</strong>ik Günleri VII-<br />

2008 sınavında sorulan bir polinom sorusu ile devam edelim.<br />

Örnek. n > 1 bir tamsayı ve p (x) ise derecesi (n − 1) olan bir polinom<br />

olsun. Eğer her k = 1, · · · , n için p (k) = 1/k ise p (n + 1) ifadesinin eşitini<br />

bulunuz.<br />

Çözüm. Varsayalım elimizde p (x)− 1 x şeklinde bir polinom olsun. Ancak<br />

1<br />

x<br />

ifadesi burada ki varsayımımızı bozacağından g (x) = x·p (x)−1 polinomunu<br />

seçelim. p (x) polinomunun derecesi (n − 1) ise g (x) polinomunun derecesi<br />

n. dereceden olacaktır. Ayrıca polinomunun kökleride olacaktır. Buna göre<br />

aslında g (x) polinomu<br />

g (x) = c · (x − 1) · (x − 2) · · · · · (x − n)<br />

olarak yazılabilir. Buradaki c katsayı sabitini bulabilmek için önce g (0) = −1<br />

olduğunu buluruz. Buna göre, x = 0<br />

eşitliğini kullanırsak<br />

g (0) = c · (0 − 1) · (0 − 2) · · · · · (0 − n)<br />

= c · −1 · −2 · −3 · · · · · −n<br />

= c · (−1) n · n!<br />

c =<br />

olacaktır. Bizden istenen p(n + 1) ise<br />

ve<br />

g (n + 1) = (−1)1−(n+1)<br />

(n + 1)!<br />

−1<br />

(−1) n n! = (−1)1−n<br />

n!<br />

g (n + 1) = (n + 1) p (n + 1) − 1<br />

· (n + 1 − 1) · (n + 1 − 2) · · · · · (n + 1 − n)


2.17. VİETE TEOREMI 157<br />

olacağından<br />

olacağından<br />

p (n + 1) =<br />

(n + 1) · p (n + 1) − 1 = (−1)−n<br />

(n + 1)! · n!<br />

[ (−1)<br />

n ]<br />

n + 1 + 1 1<br />

·<br />

n + 1 = (−1)n + (n + 1)<br />

(n + 1) 2<br />

olacaktır.<br />

Konu olarak Viete Teoremi ile çözülebilecek sorular, nispeten daha kolay<br />

sorulardır. Bu sebepten konu anlatımı içerisindeki sorularıda çalışma sorularına<br />

ekledik. Soruların tamamı AMC ve AIME sınavlarından derlenmiştir.<br />

2.17.3 Çalışma Soruları<br />

1. ax 2 + bx + c = 0 polinomunun köklerinin tersleri toplamını bulunuz.<br />

2. x 1 ve x 2 değerleri x 2 − 3√ 2x − 3√ 4 = 0 polinomunun kökleri ise<br />

toplamını bulunuz.<br />

x 1<br />

x 2<br />

+ x 2<br />

x 1<br />

3. b ve c birer sabit sayı <strong>olm</strong>ak üzere verilen<br />

(x + 2) (x + b) = x 2 + cx + 6<br />

eşitliğini sağlayan c değerini bulunuz.<br />

4. x 1 ≠ x 2 <strong>olm</strong>ak üzere 3x 2 1 − hx 1 = b ve 3x 2 2 − hx 2 = b eşitlikleri veriliyor.<br />

Buna göre x 1 + x 2 toplamını bulunuz.<br />

5. P (x) = x 3 +ax 2 +bx+c polinomunun köklerinin aritmetik ortası, köklerin<br />

çarpımına ve katsayılar toplamına eşittir. P (x) polinomunun grafiği<br />

y eksenini 2 noktasında kestiğine göre, b değerini bulunuz.<br />

6. x 2 +mx+n = 0 denkleminin köklerinin küpleri x 2 +px+q = 0 olduğuna<br />

göre, aşağıda verilen eşitliklerin ifadelerin doğruluğunu kontrol ediniz.


158 BÖLÜM 2. KONULAR<br />

• p = m 3 + 3mn<br />

• p = m 3 − 3mn<br />

• p = 3mn − m 3<br />

• p + q = m 3<br />

• ( )<br />

m 3<br />

n =<br />

p<br />

q<br />

7. ax 2 +bx+c = 0 polinomunun kökleri r ve s olduğuna göre, kökleri ar +b<br />

ve as + b olan ikinci dereceden denklemi yazınız.<br />

8.<br />

x 2 − px + ( p 2 − 1 ) /4 = 0<br />

denkleminin kökler farkının mutlak değerini bulunuz.<br />

9. x 2 + kx + 6 = 0 polinomunun kökleri x 1 ve x 2 olarak verildiğinde x 2 −<br />

kx + 6 = 0 polinomunun kökleri sırasıyla x 1 + 5 ve x 2 + 5 <strong>olm</strong>aktadır.<br />

Buna göre, k değerini bulunuz.<br />

10. x 2 + mx + n = 0denkleminin kökleri x 2 + px + m = 0 denkleminin köklerinin<br />

iki katı olduğuna göre, n/p değerini bulunuz.(n ve p değerlerinin<br />

her biri sıfırdan farklıdır.)<br />

11. a ve b birer reel sayı <strong>olm</strong>ak üzere,<br />

8x 3 + 4ax 2 + 2bx + a = 0<br />

denkleminin birbirinden farklı üç pozitif kökü vardır. Köklerin log 2 tabanındaki<br />

toplamları 5 olduğuna göre, a değerini bulunuz.<br />

12. x 2 − 63x + k = 0 denkleminin iki kökü de birer asal sayıdır. Buna göre<br />

k değerinin alabileceği kaç farklı değer vardır.<br />

13.<br />

4√ x =<br />

12<br />

7 − 4√ x<br />

denklemini sağlayan değerler toplamını bulunuz.


2.17. VİETE TEOREMI 159<br />

14.<br />

x 2 + 18x + 30 = 2 √ x 2 + 18x + 45<br />

denklemini sağlayan değerler çarpımını bulunuz.<br />

15. p, q ve r değerleri x 3 − x 2 + x − 2 = 0 denklemini birer kökü olduğuna<br />

göre,<br />

p 3 + r 3 + q 3<br />

toplamının eşitini bulunuz.<br />

16. x 1000 − 10x + 10 = 0 denkleminin kökleri r 1 , r 2 , r 3 , · · · , r 1000 olduğuna<br />

göre,<br />

r1 1000 + r2 1000 + r3 1000 + · · · + r1000<br />

değerini bulunuz.<br />

17. ⎧<br />

⎪⎨ a + 2b + 4c = 12<br />

ab + 4bc + 2ac = 22<br />

⎪ ⎩<br />

abc = 6<br />

denklem sistemini sağlayan (a, b, c) üçlülerini bulunuz.<br />

2.17.4 Çözümler<br />

1. ax 2 + bx + c = 0 denkleminin köklerine ve diyelim. Buna göre soruda<br />

istenilen toplam<br />

1<br />

x 1<br />

+ 1 x 2<br />

ifadesinin eşiti olacağına göre,<br />

1<br />

x 1<br />

+ 1 x 2<br />

= x 1 + x 2<br />

x 1 · x 2<br />

= −b/a<br />

c/a<br />

= −b c<br />

olarak istenilen toplam bulunur.


160 BÖLÜM 2. KONULAR<br />

2. Önceki sorudaki benzer yöntemleri kullanarak çözüme gidelim. x 1 ve x 2<br />

denklemim birer kökü olduğuna göre,<br />

olarak bulunacaktır.<br />

x 1<br />

x 2<br />

+ x 2<br />

x 1<br />

= x2 1 + x2 2<br />

x 1 · x 2<br />

= (x 1 + x 2 ) 2 − 2x 1 · x 2<br />

x 1 · x 2<br />

= −3<br />

3. Denklemin kökleri −2 ve −b olduğuna göre kökler çarpımı (−2 · −b) = 6<br />

ve kökler toplamı −b − 2 = c olacağından, soruda istenilen değerler<br />

kolaylıkla bulunur.<br />

4. Soruda verilen iki denklem birbirine eşit olduğuna göre,<br />

3x 2 1 − 3x 2 2 = hx 1 − hx 2<br />

olacaktır Burada x 1 ≠ x 2 olduğuna göre, soruda istenilen ifade x 1 +x 2 =<br />

h/3 olarak bulunacaktır.<br />

5. Varsayalım denklem kökleri x 1 , x 2 , x 3 olsun. Buna göre,<br />

x 1 + x 2 + x 3<br />

3<br />

= x 1 · x 2 · x 3 = a + b + c + 1<br />

olacaktır. Soruda ayrıca P (0) = 2 olarak verildiğine göre, c = 2 olacaktır.<br />

Buna göre yeni eşitliğimizi sağlayan değerler a = 12 ve b = −17 olarak<br />

bulunur.<br />

6. Varsayalım denklemlerimizin köklerini x 3 1 , x3 2 olarak alalım. Buna göre,<br />

x 3 1+x 3 2 = (x 1 + x 2 ) ( x 2 1 + x 2 ) (<br />

)<br />

2 − x 1 x 2 = −m (x 1 + x 2 ) 2 − 3x 1 x 2 = −m ( m 2 − 3mn )<br />

olacaktır. Buna göre,<br />

olacağından eşitliklerden<br />

olanı doğrudur.<br />

x 3 1 + x 3 2 = −p = −m 3 + 3mn<br />

m 3 − 3mn = p


2.17. VİETE TEOREMI 161<br />

7. Bu sorunun çözümü okuyucuya bırakılmıştır.<br />

8. Denklemin kökler toplamı ve kökler çarpımını kullanmaya çalışalım. Buna<br />

göre,<br />

x 1 + x 2 = p, ve x 1 · x 2 = p2 − 1<br />

4<br />

olacaktır. Diskriminantı kontrol edersek de<br />

ise denklemin kökleri,<br />

∆ = p 2 − 4 ( p 2 − 1 ) 1<br />

4 = p2 − p 2 + 1 = 1<br />

x 1 = p + 1<br />

2<br />

ve x 2 = p − 1<br />

2<br />

olacağına göre, soruda istenilen cevap,<br />

olacaktır.<br />

|x 1 − x 2 | = 2<br />

9. Bu sorunun çözümü okuyucuya bırakılmıştır.<br />

10. Bu sorunun çözümü okuyucuya bırakılmıştır.<br />

11. Bu sorunun çözümü okuyucuya bırakılmıştır.<br />

12. Bu sorunun çözümü okuyucuya bırakılmıştır.<br />

13. Soruda verilen denklemde 4√ x = a olarak alınırsa, yeni denklem<br />

a 2 − 7a + 12 = 0<br />

olacaktır. Buradan a 1 = 4 ve a 2 = 3 olacağından istenen x değerleri<br />

x = 256 ve x = 81 olarak bulunur.<br />

14. Bu sorunun çözümü okuyucuya bırakılmıştır.


162 BÖLÜM 2. KONULAR<br />

15. p, q, r denklemin kökleri olduğuna göre,<br />

p 3 + r 3 + q 3 = p 2 + q 2 + r 2 − p − q − r + 6<br />

olarak cevap bulunur.<br />

= (p + q + r) 2 − 2 (pq + pr + qr) − (p + q + r) + 6<br />

= 1 − 2 · 1 + 1 + 6 = 6<br />

16. Her bir r i değeri denklemi sağladığına göre<br />

r 100<br />

1 − 10r 1 + 10 = 0<br />

olacaktır. Eğer her bir r 1 , r 2 , · · · , r 100 değerini yerine koyalım buna göre,<br />

r1 100 = 10r 1 − 10<br />

r1 100 = 10r 2 − 10<br />

· · · = · · ·<br />

r100 100 = 10r 100 − 10<br />

eşitliklerini elde ederiz. Bu eşitlikleri alt alta toplarsak<br />

cevabı elde edilir.<br />

= 10 (r 1 + r 2 + · · · + r 100 ) − 10 · 1000<br />

= 10 · 10 − 10 · 1000<br />

= 10 · (−990)<br />

= −9900<br />

17. Eğer a, 2b ve 4c değerlerini bir denklemin kökleri olarak alalım. Buna<br />

göre denklemimiz<br />

0 = (x − a) (x − 2b) (x − 4c)<br />

= ( x 2 − 2bx − ax + 2ab ) (x − 4c)<br />

= ( x 2 − x (2b + a) + 2ab ) (x − 4c)<br />

= x 3 − x 2 (a + 2b + 4c) + x (2ab + 4ac + 8bc) − 8abc<br />

= x 3 − 12x 2 + 22x − 48


2.18.<br />

BAĞINTI SAYILARI 163<br />

olduğuna göre gerçektende a, 2b ve 4c değerleri 3. dereceden bir denklemin<br />

kökleridir. Buna göre toplam da 3 · 3 · 2 = 6 tane üçlü bulunabilir.<br />

2.18 Bağıntı Sayıları<br />

A kümesi boş kümeden farklı bir küme <strong>olm</strong>ak üzere A × A kartezyen çarpım<br />

kümesinin tüm alt kümelerine birer bağıntı denir. Varsayalım kümemiz<br />

A = {a, b, c} olsun. Önce kartezyen çarpımı ve bağıntıyı tablo ile gösterelim.<br />

Tablo çizmekteki amacımız hem kartezyen çarpımı hemde seçebileceğimiz herhangi<br />

bir bağıntıyı tablo üzerinde daha kolay açıklayabilmemiz olacaktır.<br />

a b c<br />

a ∗<br />

b<br />

c ∗ ∗<br />

Yukarıdaki tabloda verilen bağıntının ikilileri, eğer bu bağıntıya β dersek,<br />

β = {(a, b), (c, a), (c, c)}<br />

olacaktır. Yıldızları tablodaki karelere rastgele koyarak, yada hiçbir kareye<br />

koymayarak, bağıntılar elde edebiliriz. Bu noktadan sonra karşılaşabileceğimiz<br />

soruladan biriside, bu şekilde kaç bağıntının yazılabileceği olacaktır. Yukarıda<br />

çizdiğimiz tablo üzerinden düşünürek başlayalım. Eğer tablodaki her bir hücreye,<br />

birer yıldız koyarsak toplamda 3 × 3 = 9 yıldız olacaktır. Bağıntılarımız<br />

kartezyen çarpımlarımızın birer alt kümesi olduğuna göre, 2 9 tane bağıntımız<br />

olacaktır.<br />

Bu durumu genelleştirelim. Eğer s(A) = n olarak alırsak, bağıntı sayımız<br />

2 n×n = 2 n2<br />

olacaktır. Şimdi, bağıntı çeşitlerini ve sayılarını inceleyelim.<br />

Yansıyan Bağıntı. x ∈ A <strong>olm</strong>ak üzere, ∀x ∈ A için (x, x) ikilisi bağıntının<br />

bir elemanı ise bağıntımız yansıyan olacaktır. Eğer A = {a, b, c} ise,


164 BÖLÜM 2. KONULAR<br />

a<br />

b<br />

c<br />

a b c<br />

∗<br />

∗<br />

∗<br />

yukarıdaki tabloda da görüldüğü üzere, en küçük yansıyan bağıntımız<br />

β = {(a, a), (b, b), (c, c)}<br />

olacaktır. Öyleyse, bu bağıntıya ekleyeceğimiz her ikili için bağıntı hala yansıyan<br />

bağıntı olarak kalacaktır. İşaretli <strong>olm</strong>ayan 6 kutu için, kartezyen çarpımın<br />

2 6 tane alt kümesi olacağından 64 tane yansıyan bağıntı yazılabilir. Eğer A<br />

kümesinin n tane elemanı olduğunu varsayarsak, A × A kümesinin n 2 tane<br />

elemanı olacaktır. Oluşturulacak tablonun köşegeni üzerindeki n elemanı çıkarırsak<br />

kalan n 2 − n tane elemanın alt kümelerini almamız yeterlidir. Yani<br />

yansıyan bağıntı sayımız<br />

2 n2−n = 2 n·(n−1)<br />

olacaktır.<br />

İngilizce kaynaklarda Irreflexive Relation olarak geçen, bağıntıya biz Türkçe<br />

olarak Yansımaz Bağıntı diyelim. Buna göre, eğer yazılan bağıntının içinde<br />

x ∈ A için (x, x) /∈ β durumu varsa, bağıntımız bir yansımaz bağıntı olacaktır.<br />

Daha basit bir anlatımla çizilecek tablonun köşegeni üzerinde bağıntılar yazılırken<br />

hiç yıldız bulunmayacak. Demek ki, yansıyan bağıntı sayısı ile yansımaz<br />

bağıntı sayısı aynıdır. Ayrıca Yansımaz Bağıntı ile Yansıyan Olmayan Bağıntı<br />

birbirlerinden farklıdır.<br />

Simetrik Bağıntı. Bir bağıntının simetrik <strong>olm</strong>ası için, (x, y) elemanı bağıntının<br />

içindeyse, (y, x) bağıntısınında bağıntının içinde <strong>olm</strong>ası şartı aranır.<br />

Eğer tablo üzerinden düşünürsek, tablonun kşegene göre simetrik <strong>olm</strong>ası gerekir.<br />

a b c<br />

a ∗<br />

b ∗ ∗<br />

c ∗ ∗


2.18.<br />

BAĞINTI SAYILARI 165<br />

Örneğin yukarıda verilen tabloda verilen bağıntı simetriktir. Bağıntının elemanları<br />

{(a, b), (b, a), (b, c), (c, b), (a, a)}<br />

olacaktır. Şimdi durumu genelleştirelim. Eğer s(A) = n olarak alırsak, simetrik<br />

bağıntı yazacağımız için köşegen üstündeki ve üzerindeki noktaları seçip bu<br />

noktaların oluşturduğu kümenin alt kümelerini almamız yeterlidir. Durumu<br />

genelleştirelim. Tüm ikililerin sayısı n 2 , köşegen üzerindeki noktaların sayısı n<br />

ise,<br />

n 2 − n n · (n + 1)<br />

+ n =<br />

2<br />

2<br />

istediğimiz ikililerin sayısını alt kümelerin sayısı, yani Simetrik Bağıntı Sayısı<br />

olacaktır.<br />

2 n·(n+1)<br />

2<br />

Anti-Simetrik Bağıntı. x ve y birbirinden farklı elemanlar <strong>olm</strong>ak üzere<br />

(x, y) ve (y, x) aynı anda bağıntının elemanı değilse bağıntı antisimetrik bir<br />

bağıntı olur. Örneğin aşağıda tablosu verilen bağıntı bir antisimetrik bağıntıdır.<br />

Bağıntının elemanları,<br />

a b c<br />

a ∗ ∗<br />

b<br />

c ∗ ∗<br />

(a, a), (a, b), (c, a), (c, c)<br />

olacaktır. Bağıntıda (a, b) varken (b, a) ve (c, a) varken (a, c) yoktur. Ama köşegenler<br />

üzerindeki elemanlar istenildiği gibi seçilebilir. Peki, kaç tane simetrik<br />

bağıntı vardır Bu sayıyı bulmak için köşegen üstünde <strong>olm</strong>a ve <strong>olm</strong>ama durumlarını<br />

ayrı ayrı değerlendirelim. Eğer s(A) = n olarak alırsak, köşegen<br />

üzerindeki, elemanlar için 2 n tane durum vardır. Köşegen üzerinde <strong>olm</strong>ayan<br />

elemanlar için ise üç durum vardır. Bunlar,


166 BÖLÜM 2. KONULAR<br />

(x,y)<br />

içinde<br />

dışında<br />

dışında<br />

(y,x)<br />

dışında<br />

içinde<br />

içinde<br />

şeklinde olacaktır. (x, y) ve (y, x) ikilisinin ikiside zaten bağıntının içinde olamaz.<br />

Kümemizde n tane eleman olduğunu varsayarsak,<br />

( n n · (n − 1)<br />

C =<br />

2)<br />

2<br />

farklı durum vardır. Her ikili için üç farklı durum olduğuna göre,<br />

3 n·(n−1)/2<br />

farklı durum oluşacaktır. Buna göre toplam antisimetrik bağıntı sayısı<br />

2 n · 3 n·(n−1)/2<br />

olacaktır.<br />

Ayrıca antisimetrik bağıntı ile simetrik <strong>olm</strong>ayan bağıntılar birbirinden farklı<br />

durumlardır. Örneğin,<br />

{(a, b), (a, c), (c, a), (c, c)}<br />

bağıntısı simetrik değildir. Çünkü, (a, b) bağıntının elemanı iken (b, a) bağıntının<br />

elemanı değildir. Benzer biçimde bağıntı antisimetrik değildir çünkü hem<br />

(a, c) hemde (c, a) bağıntının birer elemanıdır.<br />

Anti-Simetrik Bağıntı ve Yansıyan Bağıntı. Eğer bu durumu antisimetrik<br />

bağıntı ile karşılaştırırsak, aradaki tek fark, bu durumda her iki köşegen<br />

üzerindeki tüm karelerde birer yıldız olacaktır. Ancak hala,<br />

n · (n − 1)<br />

2<br />

eleman için 3 farklı durum vardır. Bu sebepten de, yansıyan ve antisimetrik<br />

bağıntı sayısı<br />

3 n·(n−1)/2<br />

olacaktır.


2.18.<br />

BAĞINTI SAYILARI 167<br />

2.18.1 Çalışma Soruları<br />

Aşağıdaki çalışma soruları Tübitak XVI. Bilgisayar Olimpiyatları sınavında<br />

sorulmuştur. Soruların çözümlerini önce kendiniz yapmaya çalışınız.<br />

Daha sonra çözümleri okuyunuz.<br />

A = {0, 1, {1}, {1, {1}}} küme olarak tanımlanıyor.<br />

1. A kümesi üzerinde kaç farklı ikili bağıntı tanımlanabilir<br />

2. A kümesi üzerinde kaç farklı simetrik olan ikili bağıntı tanımlanabilir<br />

3. A kümesi üzerinde kaç farklı simetrik ve yansıma özeliğine sahip ikili<br />

bağıntı tanımlanabilir<br />

2.18.2 Çözümler<br />

1. A kümesinin sorudada görüldüğü üzere 4 tane elemanı vardır. Buna göre<br />

A kümesi üzerinde tanımlanabilecek bağıntı sayısı için önce kartezyen<br />

çarpımın elemanlarını bulalım. s(A × A) = 4 · 4 = 16 olacaktır. Her<br />

bir bağıntı kartezyen kümenin alt kümesi olduğuna göre, alt kümeleri<br />

bulmamız yeterli olacaktır. Buna göre istenen cevap<br />

olacaktır.<br />

2 4×4 = 2 16<br />

2. Simetrik bağıntı sayısı 2 n·(n+1)<br />

2 olduğuna göre, n = 4 için istenen cevap<br />

2 10 olacaktır.<br />

3. Bağıntımız hem yansıyan hem simetrik olacağına göre, bağıntının içinde<br />

mutlaka<br />

{(a, a), (b, b), (c, c), (d, d)}<br />

ikilileri mutlaka bulunacaktır. Buna göre bu bağıntının içine eleman olarak<br />

ekleyeceğimiz her ikili bir yansıyan bağıntı olacaktır. Bağıntımız eğer<br />

aynı anda simetrik bağıntı özeliğinide taşıyacağına göre, bağıntının içinde


168 BÖLÜM 2. KONULAR<br />

(a, b) eleman olarak bulunuyorsa, (b, a) ikilisi de eleman olarak bağıntının<br />

içinde bulunacaktır. Geriye kalan 12 eleman içindeki her bir ikili<br />

simetrikliği sağlamak için yanında bir ikiliyi daha taşıyacağından aslında<br />

bizim 6 elemanlı bir kümeden alt kümeler seçmemi gerekir. Buna göre,<br />

istenen sayı 2 6 = 64 olacaktır.<br />

2.19 L˙ıneer Denklemler˙ın Tamsayı Çözümler˙ı<br />

Genel olarak tekrarlı permütasyon konusu altında verilen bu konuda asıl<br />

amacımız<br />

x 1 + x 2 + · · · + x r = n, n ∈ Z + ∨ n ∈ Z + ∪ {0}<br />

formunda verilen bir lineer denklemin pozitif tamsayı veya negatif <strong>olm</strong>ayan<br />

tamsayılardaki çözüm sayılarını bulmak olacaktır. Önce teoremleri ve çözümlü<br />

örnekleri dikkatli bir şekilde çalışarak konu sonunda verilen çalışma sorular ile<br />

konuyu daha iyi kavramaya çalışınız. Önce bir teoremle başlayalım.<br />

Teorem (De Moivre) . n pozitif bir tamsayı <strong>olm</strong>ak üzere verilen<br />

x 1 + x 2 + x 3 + · · · + x r = n<br />

denkleminin pozitif tamsayı çözümlerinin sayısı<br />

( ) n − 1<br />

r − 1<br />

olacaktır.<br />

Kanıt . n sayısını n = 1 + 1 + · · · + 1 + 1 olarak yazalım. Bu toplamda<br />

n tane 1 ve (n − 1) tane + işaretinin olduğu açıktır. Buna göre, n toplamını<br />

r tane parçaya bölmek için bizim (r − 1) tane + işaretini seçmemiz yeterli<br />

olacaktır. Buna göre seçimimiz<br />

( ) n − 1<br />

r − 1<br />

olacağından ispat tamamlanır.


2.19. LİNEER DENKLEMLERİN TAMSAYI ÇÖZÜMLERİ 169<br />

Örnek. 9 rakamını üç pozitif tamsayının toplamı olarak kaç farklı biçimde<br />

yazabiliriz Mesela, 1 + 1 + 7 ve 7 + 1 + 1 birbirinden farklı iki toplamdır.<br />

Çözüm.<br />

Soruyu eğer denklem biçiminde yazarsak, aslında sorulan soru<br />

a + b + c = 9 a > 0, b > 0, c > 0<br />

denkleminin çözüm sayısı olacaktır. Öyleyse istenen cevap<br />

( ) ( 9 − 1 8<br />

= = 28<br />

3 − 1 2)<br />

olacaktır.<br />

Sonuç . n pozitif bir tamsayı <strong>olm</strong>ak üzere verilen<br />

y 1 + y 2 + · · · + y r = n<br />

denkleminin negatif <strong>olm</strong>ayan tamsayılardaki çözüm sayısı<br />

( ) n + r − 1<br />

olacaktır.<br />

r − 1<br />

Kanıt . Denklemde y r = x r − 1, x r ≥ 1 değişken değiştirmesini yaparsak<br />

x 1 − 1 + x 2 − 1 + · · · + x r − 1 = n<br />

olacağından<br />

x 1 + x 2 + · · · + x r = n + r<br />

olacaktır. Kanıtın bundan sonrası De Moivre teoreminin bir uygulamasına<br />

dönüştüğüne göre<br />

( ) n + r − 1<br />

çözüm sayısı olacaktır.<br />

<br />

r − 1


170 BÖLÜM 2. KONULAR<br />

Örnek.<br />

a + b + c + d = 100, a ≥ 30, b ≥ 21, c ≥ 1, d ≥ 1<br />

durumlarını sağlayan kaç farklı (a, b, c, d) tamsayı dörtlüsü seçilebilir<br />

Çözüm. Önce uygun değişken değiştermeleri yapalım. a = a ′ + 29, b =<br />

b ′ + 20 olarak alırsak yeni denklemimiz<br />

a ′ + b ′ + c + d = 50<br />

olacaktır. Bu denkleminde pozitif tamsayı çözümleride<br />

( 49<br />

3<br />

)<br />

= 18424<br />

olacaktır.<br />

Örnek. 8 katlı bir binanın asansörüne binen 5 kişi asansörden katlara<br />

kaç farklı biçimde dağılabilirler<br />

Çözüm.<br />

Aslında soruda bulunması istenen sayı<br />

x 1 + x 2 + · · · + x 8 = 5<br />

denkleminin negatif <strong>olm</strong>ayan çözüm sayısıdır. Buna göre cevap<br />

( ) ( )<br />

8 + 5 − 1 12<br />

= = 792<br />

8 − 1 7<br />

olacaktır.<br />

Örnek. a + b + c + d ≤ 2009 eşitsizliğini sağlayan kaç farklı negatif<br />

<strong>olm</strong>ayan tamsayı (a, b, c, d) dörtlüsü vardır<br />

Çözüm. a+b+c+d ≤ 2009 denkleminin negatif <strong>olm</strong>ayan tamsayı çözüm<br />

dörtlülerinin sayısı<br />

a + b + c + d + f = 2009, f ≥ 0


2.19. LİNEER DENKLEMLERİN TAMSAYI ÇÖZÜMLERİ 171<br />

denkleminin çözüm dörtlülerinin sayısına eşittir. Benzer biçimde son yazdığımız<br />

denklemin çözümlerinin sayısıda<br />

a 1 − 1 + b 1 − 1 + c 1 − 1 + d 1 − 1 + f 1 − 1 = 2009<br />

denkleminin çözüm sayısı ile aynı olacağına göre istenen cevap<br />

( ) 2013<br />

4<br />

olacaktır.<br />

Buraya kadar yaptığımız örneklerde, değişkenlere dair yaptığımız sınırlamalar<br />

hep tek yönlüydü. Sıradaki örneğimizde durum biraz daha farklı.<br />

Örnek.<br />

a + b + c + d = 100, 1 ≤ a ≤ 10, b ≥ 0, c ≥ 2, 20 ≤ d ≤ 30<br />

olamak üzere verilen denklemin tüm tamsayı çözüm dörtlülerinin sayısını bulunuz.<br />

Çözüm. Çözümü durum, durum inceleyerek sürdürelim. Eğer, a ≥ 1, b ≥<br />

0, c ≥ 2, d ≥ 20 olarak alırsak denklemimizin<br />

( ) 80<br />

= 82160<br />

3<br />

farklı çözümü olacaktır. Eğer a ≥ 11, b ≥ 0, c ≥ 2, d ≥ 20 olarak alırsak çözüm<br />

sayımız ( ) 70<br />

3<br />

kadar olacaktır. Eğer a ≥ 1, b ≥ 0, c ≥ 2, d ≥ 31 olarak alırsak çözüm sayımız<br />

( ) 69<br />

3<br />

kadar olacaktır. Bu iki durumun kesişimi ise<br />

( ) 59<br />

3


172 BÖLÜM 2. KONULAR<br />

kadar olacağından birleşim kümesinin eleman sayısı<br />

( ) ( ) ( )<br />

70 69 59<br />

+ − = 74625<br />

3 3 3<br />

olacaktır. Buna göre istenen çözüm sayısı<br />

( ) ( ) ( ) ( )<br />

80 70 69 59<br />

− − + = 7335<br />

3 3 3 3<br />

olacaktır.<br />

Genel olarak karşınıza çıkabilecek lineer denklemler ve pozitif tamsayı çözümleri<br />

örneklerle açıklandı. Şimdi bir çoğu Tübitak Bilgisayar Olimpiyatları<br />

birinci aşama sınavında çıkmış çalışma sorularını yapmaya çalışarak konuyu<br />

daha iyi kavramaya çalışınız.<br />

2.19.1 Çalışma Soruları<br />

1. 1 ile 1000 arasındaki sayılardan kaç tanesinin rakamlarının toplamı 7<br />

yapar<br />

2. (5a + 8b + 2c) 15 açıldığında kaç terim elde edilir<br />

3. 0 ≤ x 1 , x 2 , x 3 , x 4 ≤ 7 olduğunda x 1 + x 2 + x 3 + x 4 = 18 denkleminin kaç<br />

farklı tamsayı çözümü vardır<br />

4. 20 adet boş kartın her birinin üzerine kare, daire ve üçgen resmi çizilebilmektedir.<br />

Buna göre kaç farklı şekilde 20 karttan oluşan bir demet<br />

oluşturulabilir<br />

5. 100 tane birbirinin aynısı top ve 5 adet birbirinden farklı kutu bulunmaktadır.<br />

Her bir kutuda en az 6 adet top bulunacak şekilde topları kaç<br />

farklı şekilde kutulara dağıtabiliri<br />

6. 100 tane birbirinin aynısı top ve 5 adet birbirinden farklı kutu bulunmaktadır.<br />

Her bir kutuda en fazla 40 adet top bulunacak biçimde kaç<br />

farklı şekilde kutulara dağıtabiliriz<br />

7. Kırmızı, beyaz ve mavi zarların üçü birden atıldığında, kaç farlı durumda<br />

gelen sayıların toplamı 10 yapar


2.20. FONKSİYONEL DENKLEMLER 173<br />

8. a + b + c + d = 98 eşitliğini sağlayan kaç (a, b, c, d) pozitif tek tamsayı<br />

dörtlüsü vardır (AIME 1998)<br />

2.19.2 Çözümler<br />

Konu anlatımı içerisinde verilen çözümler, örnekleri çözmeniz için yeterlidir.<br />

Biraz uğraşark çözümlere ulaşabilirsiniz.<br />

2.20 Fonks˙ıyonel Denklemler<br />

Bilinmeyenlerinin birer fonksiyon olduğu denklemlere genel olarak fonksiyonel<br />

denklem denilir. Benzer biçimde bilinmeyenlerin polinomlardan oluştuğu<br />

denklem sorularınıda bu başlık altında alabiliriz. Ancak bu tür soruların<br />

ne yazıkki genel bir çözüm yöntemi yoktur. Belkide bu sebepten <strong>mat</strong>e<strong>mat</strong>ik<br />

yarışmalarında sıklıkla sorulan sorular haline gelmişlerdir. Bu başlık altında<br />

genel olarak tek değişkenli fonksiyonel denklemlerin genel çözüm teknikleri ve<br />

çok değişkenli fonksiyonel denklemler konuları ele alınmıştır.<br />

2.20.1 Tek Değişkenliler - Temel Teknikler<br />

Tek değişkenlilerin çözümlerine uygun dönüşümler yapılarak ulaşılır. Örnekleri<br />

inceleyerek devap edelim.<br />

Örnek.<br />

f (x + 1) = x 2 − 3x + 2 fonksiyonel denklemini çözünüz.<br />

Çözüm. t = x+1 ise x = t−1 olacağından f(t) = (t−1) 2 −3(t−1)+2 =<br />

t 2 − 5t + 6 eşitliğinden f(x) = x 2 − 5x + 6 olarak bulunur.<br />

Örnek.<br />

f( x + 1<br />

x ) = x2 + 1<br />

x 2 + 1 x<br />

fonksiyonel denklemini çözünüz.<br />

Çözüm.<br />

t = (x + 1)/x olarak alırsak x = 1/(t − 1) olacağından<br />

f(t) = ( 1<br />

t−1 )2 + 1<br />

( 1 + 1<br />

t−1 )2 ( 1<br />

t−1 ) = t2 − t + 1


174 BÖLÜM 2. KONULAR<br />

bulunur. Buna göre f(x) = x 2 − x + 1 olarak bulunur.<br />

Genel olarak çözümlerde uyguladığımız teknik, f[ϕ(x)] = g(x) eşitliğini f<br />

için çözmektir. Eğer ϕ fonksiyonunun tersi varsa, t = ϕ(x) olarak alabiliriz.<br />

Dolayısıyla da<br />

f(x) = g[ϕ −1 (x)]<br />

olur.<br />

Örnek.<br />

Çözüm.<br />

f(e x ) = x 3 + sin x fonksiyonel denklemini çözünüz.<br />

t = e x değişken değiştirmesini yaparsak, x = ln t olacağından<br />

f(x) = (ln |x|) 3 + sin(ln |x|)<br />

olacaktır.<br />

Örnek.<br />

a ≠ ±1 <strong>olm</strong>ak üzere verilen<br />

x<br />

f( ) = af(x) + ϕ(x)<br />

x − 1<br />

fonksiyonel denklemini çözünüz.<br />

Çözüm. t = x/(x − 1) değişken değiştirmesini yaparsak, x = t/(t − 1)<br />

olacaktır. Buna göre,<br />

t<br />

f(t) = af(<br />

t − 1 ) + ϕ( t<br />

t<br />

) = a(af(t) + ϕ(t)) + ϕ(<br />

t − 1 t − 1 )<br />

olacağından, istenilen fonksiyon<br />

f(x) =<br />

aϕ(x) + ϕ(<br />

x<br />

x−1 )<br />

1 − a 2<br />

olacaktır.<br />

Bir fonksiyonel denklem değişkenlere bağlı cebirsel ifadeler ihtiva edebilir.<br />

Bu tür sorularda uygulanacak temel teknik eşzamanlı fonksiyonlar oluşturmaktır.<br />

Şimdi aşağıdaki örnekleri inceleyelim.


2.20. FONKSİYONEL DENKLEMLER 175<br />

Örnek.<br />

fonksiyonel denklemini çözünüz.<br />

3f(x) + 2f(1/x) = 4x<br />

Çözüm.<br />

x değişkeni yerine 1/x alınırsa,<br />

3f(1/x) + 2f(x) = 4/x<br />

olacaktır. Soruda verilen fonksiyon −3/2 ile çarpılıp son bulunan denklem ile<br />

toplanırsa, istenilen denklem<br />

olarak bulunacaktır.<br />

f(x) = 12x2 − 8<br />

5x<br />

Örnek.<br />

x ≠ 0 <strong>olm</strong>ak üzere verilen<br />

fonksiyonel denklemini çözünüz.<br />

1<br />

x · f(−x) + f( 1 x ) = x<br />

Çözüm.<br />

Soruda verilen denklemde x yerine −x yazarsak,<br />

bulunur. Eğer x yerine 1 x alınırsa<br />

− 1 x f(x) + f(− 1 x ) = −x<br />

xf(− 1 x ) + f(x) = 1 x<br />

denklemi elde edilecektir. Bu iki fonksiyonel denklemden, istenilen fonksiyon<br />

f(x) = 1 2 (x2 + 1 x )<br />

olarak elde edilir.


176 BÖLÜM 2. KONULAR<br />

Fonksiyonel denklemlerin çözümü yapılırken eğer fonksiyonun özelikleride<br />

göz önüne alınırsa çözüme ulaşılması daha kolay olacaktır. Öyle ki fonksiyonun<br />

sürekliliği, monotonluğu, sınırlılığı, türevlenebilir <strong>olm</strong>ası gibi bilgilerin<br />

bilinmesi çözümlerde faydalı olacaktır.<br />

Örnek.<br />

f(x + 1) + f(x − 1) = 2x 2 − 4x<br />

fonksiyonel denklemini çözünüz.<br />

Çözüm. İki fonksiyonun toplamı ikinci dereceden olduğuna göre f(x)<br />

fonksiyonuda ikinci dereceden olacaktır. Buna göre, f(x) = ax 2 + bx + c ise<br />

2ax 2 + 2bx + 2(a + c) = 2x 2 − 4x<br />

eşitliğinden f(x) fonksiyonu f(x) = x 2 − 2x − 1 olarak bulunur.<br />

2.20.2 Çok Değişkenliler<br />

Bazı çok değişkenli fonksiyonel denklemlerin çözümlerinde simetriden faydalanılarak<br />

denklemi tek değişkenliye indirgemek çözümü oldukça kolaylaştırır.<br />

Aşağıdaki örnekler ve çözümler bu durumla alakalıdır.<br />

Örnek.<br />

(x − y)f(x + y) − (x + y)f(x − y) = 4xy(x 2 − y 2 )<br />

fonksiyonel denklemini çözünüz.<br />

Çözüm.<br />

Eğer soruda verilen eşitliği düzenlersek<br />

f(x + y)<br />

x + y<br />

−<br />

f(x − y)<br />

x − y<br />

= 4xy<br />

olacaktır. Buna göre<br />

g(x) = f(x)<br />

x


2.20. FONKSİYONEL DENKLEMLER 177<br />

fonksiyonunu alalım. Eğer<br />

g(x + y) − g(x − y) = 4xy<br />

ise<br />

g(x + y) − (x + y) 2 = g(x − y) − (x − y) 2<br />

olacağından g(x) − x 2 = k, k ∈ R ve g(x) = x 2 + k, f(x) = x 3 + kx olarak<br />

bulunur.<br />

Değişken sayısını düşürmenin bir diğer yoluda uygun değerleri değişkenlere<br />

atamaktır. Aşağıdaki örnek bu metodla çözülmüştür.<br />

Örnek.<br />

çözünüz.<br />

f(x) · f(y) − f(xy) = x + y fonksiyonel denklemini f(x) için<br />

Çözüm. Varsayalım y = 0 olsun. Buna göre, f(0)[f(x)−1] = x olacaktır.<br />

f(0) ≠ 0 olacağına göre,<br />

f(x) =<br />

x<br />

f(0) + 1<br />

eşitliği elde edilir. Benzer biçimde x = y = 0 alınırsa f(0)[f(0) − 1] = 0 ise<br />

f(0) = 1 olacağından f(x) = x + 1 olarak bulunur. Gerçektende bulduğumuz<br />

fonksiyon soruda verilen denklemi sağlamaktadır.<br />

Örnek.<br />

f fonksiyonu<br />

f(x) + f(y) = f(x + y) − xy − 1<br />

eşitliğini sağlamaktadır. Buna göre, eğer f(1) = 1 ise f(n) = n eşitliğini<br />

sağlayan negatif tamsayı değerlerini bulunuz.<br />

Çözüm. x = 1 olarak alırsak, f(y + 1) − f(y) = y + 2 ve y = 0 alırsak<br />

f(0) = −1 olacaktır. n ≥ için<br />

n−1<br />

∑<br />

n−1<br />

∑<br />

f(n+1)+1 = f(n)−f(0) = (f(y+1)−f(y)) = (y+2) =<br />

y=0<br />

y=0<br />

(n + 1)(n + 2)<br />

−1<br />

2


178 BÖLÜM 2. KONULAR<br />

olacağından<br />

f(n) = n2+ 3n+2<br />

2<br />

− 2 = n2 + 3n − 2<br />

2<br />

olacaktır. Eğer x = n ve y = −n alırsak, f(n) + f(−n) = n 2 − n ve<br />

olacaktır. Demekki,<br />

f(−n) = n 2 − 2 − f(n) = (−n)2 + 3(−n) − 2<br />

2<br />

f(n) = n2 + 3n − 2<br />

2<br />

eşitliği negatif n değerleri içinde geçerlidir. Sorudaki f(n) = n durumu için<br />

n 2 + 3n − 2<br />

2<br />

= n<br />

ise (n − 1)(n + 2) = 0 olacağından n = 1 veya n = −2 olacaktır.<br />

Fonksiyonel denklemlerin çözümleri yapılırken bazı kullanışlı sonuçların bilinmesinde<br />

fayda vardır. Örneğin aşağıda verilenler bunlardan sadece birkaçıdır.<br />

Buna göre, f sürekli bir fonksiyon <strong>olm</strong>ak üzere<br />

i. f(x + y) = f(x) + f(y) ise f(x) = c,<br />

ii. f(x + y) = f(x) · f(y) ise f(x) = c x<br />

iii. f(xy) = f(x) + f(y) ise f(x) = c ln x<br />

iv. f(xy) = f(x) · f(y) ise f(x) = x c<br />

olacaktır. Sıradaki örneğimizin çözümünde bu sonuçları kullanabiliriz.<br />

Örnek.<br />

(Jensen Fonksiyonel Denklemi) x, y ∈ R <strong>olm</strong>ak üzere verilen<br />

( ) x + y<br />

f =<br />

2<br />

f (x) + f (y)<br />

2<br />

eşitliğini sağlayan tüm sürekli f fonksiyonlarını bulunuz.


2.20. FONKSİYONEL DENKLEMLER 179<br />

Çözüm.<br />

olacaktır. Buna göre,<br />

Eğer y = 0 olarak alınırsa,<br />

( x<br />

)<br />

f (x) + f (0)<br />

f =<br />

2 2<br />

f (x) + f (y)<br />

2<br />

( ) x + y<br />

= f =<br />

2<br />

f (x + y) + f (0)<br />

2<br />

olacaktır. Yani, f(x + y) = f(x) + f(y) − f(0) olarak bulunur. Varsayalım<br />

h(x) = f(x) − f(0) ise h(x + y) = h(x) + h(y) olacağından h(x) = cx ve<br />

f(x) = cx + f(0) olarak bulunur.<br />

Örnek.<br />

bulunuz.<br />

f(x+y) = g(x)+h(y) eşitliğini sağlayan tüm sürekli fonksiyonları<br />

Çözüm. Eğer y = 0 olarak alırsak h(0) = b ise f(x) = g(x) + b ve eğer<br />

x = 0 olarak alırsak benzer biçimde f(y) = a + h(y) olacaktır. Buna göre,<br />

f(x + y) = [f(x) − b] + [f(y) − a] = f(x) + f(y) − (a + b)<br />

olacağından F (x) = f(x) − a − b ve F (x + y) = F (x) + F (y) olacaktır. F<br />

fonksiyonu toplamsal sürekli fonksiyon olduğundan F (x) = kx olacaktır. Sonuç<br />

olarak f(x) = kx + a + b, g(x) = kx + a ve h(x) = kx + b olacaktır.<br />

Olimpiyat sınavlarında özellikle TST ve kamp sınavlarında sorulan soruların<br />

çözümleri oldukça zordur. Belli teknikler kullanarak çözümeye gitmeye<br />

çalışmak her ne kadar gerekli olsada çoğu zaman yereli değildir. Ancak bazı<br />

yardımcı özelikleri araştırmak sanırız çözüm yolundaki en büyük yardımcılardan<br />

olacaktır. Bu sebepten elimizdeki fonksiyonun<br />

i. Birebir veya örtenliği,<br />

ii. Periyodikliği veya tek, çift fonksiyon <strong>olm</strong>a durumu,<br />

iii. Artan yada azalan <strong>olm</strong>a durumu,<br />

iv. Simetrikliği<br />

gibi özelliklerinin olup <strong>olm</strong>adığını bilmek, çoğu zaman çözümü ciddi biçimde<br />

kolaylaştıracaktır.


180 BÖLÜM 2. KONULAR<br />

2.20.3 Çalışma Soruları<br />

1. f : R → R <strong>olm</strong>ak üzere verilen<br />

f(f(x + y)) = f(x + y) + f(x)f(y) − xy x, y ∈ R<br />

eşitliğini sağlayan tüm f fonksiyonlarını bulunuz.<br />

2. f : Z → R ve x + y toplamı 3’ün tam katı olduğuna göre,<br />

( ) x + y f(x) + f(y)<br />

f =<br />

3<br />

2<br />

eşitliğini sağlayan f− fonksiyonlarını bulunuz.<br />

3. f : R → R <strong>olm</strong>ak üzere verilen<br />

f(xf(y) + x) = xy + f(x)<br />

eşitliğini sağlayan tüm f fonksiyonlarını bulunuz.<br />

4. f : R → R <strong>olm</strong>ak üzere verilen<br />

f(f(x) + y) = f(x 2 − y) + 4f(x)y<br />

eşitliğini sağlayan f fonksiyonlarını bulunuz.<br />

2.20.4 Çözümler<br />

1. Eğer y = 0 alırsak,<br />

f(f(x)) = [1 + f(0)]f(x)<br />

eşitliği elde edilir. Eğer x yerine x + y alırsak<br />

[1 + f(0)]f(x + y) = f(f(x + y)) = f(x + y) + f(x)f(y) − xy<br />

eşitliğinden<br />

elde edilir. Eğer y = 1 alınırsa,<br />

f(0)f(x + y) = f(x)f(y) − xy<br />

f(0)f(x + 1) = f(x)f(1) − x


2.20. FONKSİYONEL DENKLEMLER 181<br />

eşitliği elde edilir. Eğer y = −1 ve x yerine x + 1 alınırsa<br />

f(0)f(x) = f(x + 1)f(−1) = f(x + 1)f(−1) + x + 1<br />

eşitliği elde edilir. Eğer son iki denklemden f(x + 1)’i çekersek<br />

[f 2 (0) − f(1)f(−1)]f(x) = [f(0) − f(−1)]x + f(0)<br />

eşitliği elde edilir. Eğer f 2 (0) − f(1)f(−1) ≠ 0 ise f− fonksiyonu lineerdir.<br />

Eğer f 2 (0)−f(1)f(−1) = 0 ise son denklemde x = 0 alarak f(0) = 0<br />

olarak bulunur. Bu durumda fonksiyon f(x)f(y) = xy olacaktır. y = 1<br />

alırsak f(x)f(1) = x olacaktır. Dolayısıyla f(1) ≠ 0 ve f(x) lineerdir.<br />

Sonuç olarak, f(x) = ax + b olarak soruda verilen denklemde yerine<br />

koyarsak a = 1 ve b = 0 bulunur. Demek ki, fonksiyon f(x) = x’dir.<br />

2. Bütün n tamsayıları için<br />

olacağından<br />

f(0) + f(3n) = 2f(n) = f(n) + f(2n)<br />

f(n) = f(2n) =<br />

f(3n) + f(3n)<br />

2<br />

= f(3n)<br />

olacaktır. Demek ki f(n) = f(0) eşitliği vardır. Dolayısıyla tüm tüm<br />

sabit fonksiyonlar birer çözüm olacaktır.<br />

3. Eğer x = 1, y = −1 − f(1) ve a = f(y) + 1 alırsak eşitliğimiz<br />

f(a) = f(f(y) + 1) = y + f(1) = −1<br />

olacaktır. Eğer y = a ve b = f(a) olarak alırsak<br />

ve<br />

b = f(xf(a) + x) = ax + f(x)<br />

f(x) = −ax + b<br />

olacaktır. Eğer sorudaki denklemde yerine koyarsak,<br />

ax 2 y − abx − ax + b = xy − ax + b<br />

eşitliği bulunur. Polinom eşitliğinden a = ±1 ve b = 0 dolayısıyla f(x) =<br />

x veya f(x) = −x bulunur.


182 BÖLÜM 2. KONULAR<br />

4. y = x 2 olarak alırsak<br />

bulunur. y = −f(x) alınırsa<br />

f(f(x) + x 2 ) = f(0) + 4x 2 f(x)<br />

f(0) = f(f(x) + x 2 ) + 4f(x) 2<br />

eşitsizliği elde edilir. Bulduğumuz bu iki eşitliği karşılaştırırsak, her bir<br />

x değeri için f(x) = 0 veya f(x) = x 2 <strong>olm</strong>alıdır. Varsayalım f(a) = 0<br />

olsun. x = a olarak alırsak<br />

bulunur. y ≠ a 2 /2 için<br />

f(y) = f(a 2 − y)<br />

y 2 ≠ (a 2 − y) 2<br />

ise f(y) = 0 olacaktır. Son olarak x = 2a ve y = a 2 /2 alırsak<br />

f(a 2 /2) = f(7a 2 /2) = 0<br />

olacaktır. Demek ki, f(x) = 0 veya f(x) = x 2 <strong>olm</strong>alıdır.

Hooray! Your file is uploaded and ready to be published.

Saved successfully!

Ooh no, something went wrong!